diff --git a/TST_sti2d/DM/2102_DM1/01_2102_DM1.tex b/TST_sti2d/DM/2102_DM1/01_2102_DM1.tex new file mode 100644 index 0000000..fd69c5b --- /dev/null +++ b/TST_sti2d/DM/2102_DM1/01_2102_DM1.tex @@ -0,0 +1,121 @@ +\documentclass[a4paper,10pt]{article} +\usepackage{myXsim} +\usepackage{tasks} + +% Title Page +\title{DM1 \hfill BAHBAH Zakaria} +\tribe{TST sti2d} +\date{\hfillÀ render pour le jeudi 25 février} + +\xsimsetup{ + solution/print = false +} + +\begin{document} +\maketitle + +\begin{exercise}[subtitle={Complexes}] + \begin{enumerate} + \item Mettre le nombre complexe suivant sous forme algébrique $z_1 = \dfrac{7 + 2 i}{-5 + 3 i} $ + \item Mettre le complexe suivante sous forme exponentielle $z_2 = -7 + 7 \sqrt{3} i$ + \item Mettre le complexe suivante sous forme exponentielle $z_3 = 4 + 4 \sqrt{3} i$ + \item Calculer le produit $z_4=z_2\times z_3$ donner le résultat sous forme exponentielle puis algébrique. + \item Calculer le quotient $z_5=\frac{z_2}{z_3}$ donner le résultat sous forme exponentielle puis algébrique. + \end{enumerate} +\end{exercise} + +\begin{solution} + \begin{enumerate} + \item $z_1 = - \frac{29}{34} - \frac{31 i}{34}$ + \item $z_3 = 14 e^{\frac{2 i \pi}{3}}$ + \item $z_4 = 112 e^{i \pi} = -112 = -112.0$ + \item $z_5 = \frac{7}{4} e^{\frac{i \pi}{3}} = \frac{7}{8} + \frac{7 \sqrt{3} i}{8} = 0.875 + 1.52 i$ + \end{enumerate} +\end{solution} + +\begin{exercise}[subtitle={Bassin}] + Le tour d'un bassin au niveau du sol présente deux axes de symétrie : l’axe des abscisses et la droite d’équation $x=4$. Il est obtenu par symétrie de la courbe $\mathcal{C_f}$ sur $\intFF{0}{4}$ où $f$ est la fonction définie par + + \[ + f(x) = \left(- x^{2} + 2.4 x - 6.2\right) e^{- x} + 6.2 + \] + On admet que sur $\intFF{0}{4}$ la fonction $f$ est positive. + \begin{enumerate} + \item Sur un repère, tracer l'allure de la courbe $\mathcal{C}_f$, les axes de symétries puis compléter pour dessiner la forme du bassin. + \item Montrer que la fonction $f$ admet comme primitive sur $\R$ la fonction $F$ définie par + \[ + F(x) = 6.2 x + \left( x^{2} - 0.4 x + 5.8\right) e^{- x} + \] + \item Calculer la quantité $\ds \int_0^4 f(x) \; dx$, vous donnerez le résultat sous forme exacte. Interpréter le résultat et reportez cette quantité sur le graphique. + \item On considère que l'échelle de votre graphique est de 1unité pour 15m. Calculer l'aire du bassin. Vous donnerez un résultat arrondi au $m^2$ près. + \end{enumerate} +\end{exercise} + +\begin{solution} + \begin{enumerate} + \item + \begin{tikzpicture}[baseline=(a.north), xscale=1, yscale=0.5] + \tkzInit[xmin=0,xmax=5,xstep=1, + ymin=0,ymax=10,ystep=1] + \tkzGrid + \tkzAxeXY + \tkzFct[domain=0:10,color=red,very thick]% + { (-x**2 + 2.4*x - 6.2)*exp(-x) + 6.2 }; + \end{tikzpicture} + \item Il faut dériver $F(x)$ et vérifier que $F'(x) = f(x)$. + \item $\ds \int_0^4 f(x) \; dx = F(4) - F(0) = \frac{20.2}{e^{4}} + 19.0$ + \item La quantité calculée à la question précédente se retrouve 4fois pour former le bassin. Il faut ensuite prendre en compte l'échelle, comme 1unité de longueur correspond à 15m, une unité d'air correspond à $15\times15 = 225m^2$. Ainsi l'aire du bassin est égale à + \[ + (\frac{20.2}{e^{4}} + 19.0)\times 4 \times 15^2 = 17433.00000 + \] + + \end{enumerate} +\end{solution} + +\begin{exercise}[subtitle={Bassin}] + Le clinker est un constituant du ciment qui résulte de la cuisson d'un mélange composé de calcaire et d'argile. La fabrication du clinker nécessite des fours à très haute température qui libèrent dans l'air une grande quantité de dioxyde de carbone (CO$_2$). + + Dans une cimenterie, la fabrication du clinker s'effectue de 7 h 30 à 20 h, dans une pièce de volume \np{900000}~dm$^3$. + + À 20 h, après une journée de travail, le taux volumique de CO$_2$ dans la pièce est de 0.9\,\%. + \begin{enumerate} + \item Justifier que le volume de CO$_2$ présent dans cette pièce à 20 h est de \np{8100}~dm$^3$ . + \item On modélise le volume de CO$_2$ présent dans la pièce par une fonction du temps $t$ écoulé après 20h (exprimé en minutes) qui pour formule $V(t) = V_0e^{-0.06t} + 560$ + \begin{enumerate} + \item Démontrer que $V_0$ est égale à \np{7540}. + \item Quel sera, au dm$^3$ près, le volume de CO$_2$ dans cette pièce à 23 h ? + \item Démontrer que $V'(t) = - 452.4 e^{- 0.06 t}$. + \item Étudier le signe de $V'(t)$ puis en déduire le sens de variation de $V(t)$. + \item Que peut-on dire du volume de CO$_2$ quand $t$ devient grand? + \end{enumerate} + \end{enumerate} +\end{exercise} + +\begin{solution} + \begin{enumerate} + \item Volume à 20h: $900000\times 0.009000000000000001 = 8100$ + \item + \begin{enumerate} + \item $t=0$ correspond à 20h. + + Donc $V(0) = 8100 = V_0e^{-0.06\times 0} + 560 = V_0 + 560$ + + Donc $V_0 = 8100 - 560 = 7540$ + \item Il faut calculer $V(t)$ pour $t = 3$ donc + \[ + V(3) = 6857.94 + \] + \item Pas de correction pour cette question. + \item Pas de correction pour cette question. + \item Pas de correction pour cette question. + \end{enumerate} + \end{enumerate} +\end{solution} + + +\end{document} + +%%% Local Variables: +%%% mode: latex +%%% TeX-master: "master" +%%% End: diff --git a/TST_sti2d/DM/2102_DM1/02_2102_DM1.tex b/TST_sti2d/DM/2102_DM1/02_2102_DM1.tex new file mode 100644 index 0000000..3591aa3 --- /dev/null +++ b/TST_sti2d/DM/2102_DM1/02_2102_DM1.tex @@ -0,0 +1,121 @@ +\documentclass[a4paper,10pt]{article} +\usepackage{myXsim} +\usepackage{tasks} + +% Title Page +\title{DM1 \hfill BENALI Ilyas} +\tribe{TST sti2d} +\date{\hfillÀ render pour le jeudi 25 février} + +\xsimsetup{ + solution/print = false +} + +\begin{document} +\maketitle + +\begin{exercise}[subtitle={Complexes}] + \begin{enumerate} + \item Mettre le nombre complexe suivant sous forme algébrique $z_1 = \dfrac{9 + 3 i}{-9 + 8 i} $ + \item Mettre le complexe suivante sous forme exponentielle $z_2 = 2 \sqrt{2} + 2 \sqrt{2} i$ + \item Mettre le complexe suivante sous forme exponentielle $z_3 = 4 \sqrt{2} - 4 \sqrt{2} i$ + \item Calculer le produit $z_4=z_2\times z_3$ donner le résultat sous forme exponentielle puis algébrique. + \item Calculer le quotient $z_5=\frac{z_2}{z_3}$ donner le résultat sous forme exponentielle puis algébrique. + \end{enumerate} +\end{exercise} + +\begin{solution} + \begin{enumerate} + \item $z_1 = - \frac{57}{145} - \frac{99 i}{145}$ + \item $z_3 = 4 e^{\frac{i \pi}{4}}$ + \item $z_4 = 32 e^{0} = 32 = 32.0$ + \item $z_5 = \frac{1}{2} e^{\frac{i \pi}{2}} = \frac{i}{2} = 0.5 i$ + \end{enumerate} +\end{solution} + +\begin{exercise}[subtitle={Bassin}] + Le tour d'un bassin au niveau du sol présente deux axes de symétrie : l’axe des abscisses et la droite d’équation $x=4$. Il est obtenu par symétrie de la courbe $\mathcal{C_f}$ sur $\intFF{0}{4}$ où $f$ est la fonction définie par + + \[ + f(x) = \left(- x^{2} + 8.7 x - 5.0\right) e^{- x} + 5.0 + \] + On admet que sur $\intFF{0}{4}$ la fonction $f$ est positive. + \begin{enumerate} + \item Sur un repère, tracer l'allure de la courbe $\mathcal{C}_f$, les axes de symétries puis compléter pour dessiner la forme du bassin. + \item Montrer que la fonction $f$ admet comme primitive sur $\R$ la fonction $F$ définie par + \[ + F(x) = 5.0 x + \left( x^{2} - 6.7 x - 1.7\right) e^{- x} + \] + \item Calculer la quantité $\ds \int_0^4 f(x) \; dx$, vous donnerez le résultat sous forme exacte. Interpréter le résultat et reportez cette quantité sur le graphique. + \item On considère que l'échelle de votre graphique est de 1unité pour 15m. Calculer l'aire du bassin. Vous donnerez un résultat arrondi au $m^2$ près. + \end{enumerate} +\end{exercise} + +\begin{solution} + \begin{enumerate} + \item + \begin{tikzpicture}[baseline=(a.north), xscale=1, yscale=0.5] + \tkzInit[xmin=0,xmax=5,xstep=1, + ymin=0,ymax=10,ystep=1] + \tkzGrid + \tkzAxeXY + \tkzFct[domain=0:10,color=red,very thick]% + { (-x**2 + 8.7*x - 5.0)*exp(-x) + 5.0 }; + \end{tikzpicture} + \item Il faut dériver $F(x)$ et vérifier que $F'(x) = f(x)$. + \item $\ds \int_0^4 f(x) \; dx = F(4) - F(0) = 21.7 - \frac{12.5}{e^{4}}$ + \item La quantité calculée à la question précédente se retrouve 4fois pour former le bassin. Il faut ensuite prendre en compte l'échelle, comme 1unité de longueur correspond à 15m, une unité d'air correspond à $15\times15 = 225m^2$. Ainsi l'aire du bassin est égale à + \[ + (21.7 - \frac{12.5}{e^{4}})\times 4 \times 15^2 = 19324.00000 + \] + + \end{enumerate} +\end{solution} + +\begin{exercise}[subtitle={Bassin}] + Le clinker est un constituant du ciment qui résulte de la cuisson d'un mélange composé de calcaire et d'argile. La fabrication du clinker nécessite des fours à très haute température qui libèrent dans l'air une grande quantité de dioxyde de carbone (CO$_2$). + + Dans une cimenterie, la fabrication du clinker s'effectue de 7 h 30 à 20 h, dans une pièce de volume \np{700000}~dm$^3$. + + À 20 h, après une journée de travail, le taux volumique de CO$_2$ dans la pièce est de 0.8\,\%. + \begin{enumerate} + \item Justifier que le volume de CO$_2$ présent dans cette pièce à 20 h est de \np{5600}~dm$^3$ . + \item On modélise le volume de CO$_2$ présent dans la pièce par une fonction du temps $t$ écoulé après 20h (exprimé en minutes) qui pour formule $V(t) = V_0e^{-0.09t} + 360$ + \begin{enumerate} + \item Démontrer que $V_0$ est égale à \np{5240}. + \item Quel sera, au dm$^3$ près, le volume de CO$_2$ dans cette pièce à 21 h ? + \item Démontrer que $V'(t) = - 471.6 e^{- 0.09 t}$. + \item Étudier le signe de $V'(t)$ puis en déduire le sens de variation de $V(t)$. + \item Que peut-on dire du volume de CO$_2$ quand $t$ devient grand? + \end{enumerate} + \end{enumerate} +\end{exercise} + +\begin{solution} + \begin{enumerate} + \item Volume à 20h: $700000\times 0.008 = 5600$ + \item + \begin{enumerate} + \item $t=0$ correspond à 20h. + + Donc $V(0) = 5600 = V_0e^{-0.09\times 0} + 360 = V_0 + 360$ + + Donc $V_0 = 5600 - 360 = 5240$ + \item Il faut calculer $V(t)$ pour $t = 1$ donc + \[ + V(1) = 5149.00000000000 + \] + \item Pas de correction pour cette question. + \item Pas de correction pour cette question. + \item Pas de correction pour cette question. + \end{enumerate} + \end{enumerate} +\end{solution} + + +\end{document} + +%%% Local Variables: +%%% mode: latex +%%% TeX-master: "master" +%%% End: diff --git a/TST_sti2d/DM/2102_DM1/03_2102_DM1.tex b/TST_sti2d/DM/2102_DM1/03_2102_DM1.tex new file mode 100644 index 0000000..3bcc806 --- /dev/null +++ b/TST_sti2d/DM/2102_DM1/03_2102_DM1.tex @@ -0,0 +1,121 @@ +\documentclass[a4paper,10pt]{article} +\usepackage{myXsim} +\usepackage{tasks} + +% Title Page +\title{DM1 \hfill BERNADAT Noah} +\tribe{TST sti2d} +\date{\hfillÀ render pour le jeudi 25 février} + +\xsimsetup{ + solution/print = false +} + +\begin{document} +\maketitle + +\begin{exercise}[subtitle={Complexes}] + \begin{enumerate} + \item Mettre le nombre complexe suivant sous forme algébrique $z_1 = \dfrac{2 + 3 i}{-7 + 10 i} $ + \item Mettre le complexe suivante sous forme exponentielle $z_2 = - 5 \sqrt{3} + 5 i$ + \item Mettre le complexe suivante sous forme exponentielle $z_3 = 3 \sqrt{3} - 3 i$ + \item Calculer le produit $z_4=z_2\times z_3$ donner le résultat sous forme exponentielle puis algébrique. + \item Calculer le quotient $z_5=\frac{z_2}{z_3}$ donner le résultat sous forme exponentielle puis algébrique. + \end{enumerate} +\end{exercise} + +\begin{solution} + \begin{enumerate} + \item $z_1 = \frac{16}{149} - \frac{41 i}{149}$ + \item $z_3 = 10 e^{\frac{5 i \pi}{6}}$ + \item $z_4 = 60 e^{\frac{2 i \pi}{3}} = -30 + 30 \sqrt{3} i = -30.0 + 52.0 i$ + \item $z_5 = \frac{5}{3} e^{i \pi} = - \frac{5}{3} = -1.67$ + \end{enumerate} +\end{solution} + +\begin{exercise}[subtitle={Bassin}] + Le tour d'un bassin au niveau du sol présente deux axes de symétrie : l’axe des abscisses et la droite d’équation $x=4$. Il est obtenu par symétrie de la courbe $\mathcal{C_f}$ sur $\intFF{0}{4}$ où $f$ est la fonction définie par + + \[ + f(x) = \left(- x^{2} + 4.9 x - 9.7\right) e^{- x} + 9.7 + \] + On admet que sur $\intFF{0}{4}$ la fonction $f$ est positive. + \begin{enumerate} + \item Sur un repère, tracer l'allure de la courbe $\mathcal{C}_f$, les axes de symétries puis compléter pour dessiner la forme du bassin. + \item Montrer que la fonction $f$ admet comme primitive sur $\R$ la fonction $F$ définie par + \[ + F(x) = 9.7 x + \left( x^{2} - 2.9 x + 6.8\right) e^{- x} + \] + \item Calculer la quantité $\ds \int_0^4 f(x) \; dx$, vous donnerez le résultat sous forme exacte. Interpréter le résultat et reportez cette quantité sur le graphique. + \item On considère que l'échelle de votre graphique est de 1unité pour 15m. Calculer l'aire du bassin. Vous donnerez un résultat arrondi au $m^2$ près. + \end{enumerate} +\end{exercise} + +\begin{solution} + \begin{enumerate} + \item + \begin{tikzpicture}[baseline=(a.north), xscale=1, yscale=0.5] + \tkzInit[xmin=0,xmax=5,xstep=1, + ymin=0,ymax=10,ystep=1] + \tkzGrid + \tkzAxeXY + \tkzFct[domain=0:10,color=red,very thick]% + { (-x**2 + 4.9*x - 9.7)*exp(-x) + 9.7 }; + \end{tikzpicture} + \item Il faut dériver $F(x)$ et vérifier que $F'(x) = f(x)$. + \item $\ds \int_0^4 f(x) \; dx = F(4) - F(0) = \frac{11.2}{e^{4}} + 32.0$ + \item La quantité calculée à la question précédente se retrouve 4fois pour former le bassin. Il faut ensuite prendre en compte l'échelle, comme 1unité de longueur correspond à 15m, une unité d'air correspond à $15\times15 = 225m^2$. Ainsi l'aire du bassin est égale à + \[ + (\frac{11.2}{e^{4}} + 32.0)\times 4 \times 15^2 = 28985.00000 + \] + + \end{enumerate} +\end{solution} + +\begin{exercise}[subtitle={Bassin}] + Le clinker est un constituant du ciment qui résulte de la cuisson d'un mélange composé de calcaire et d'argile. La fabrication du clinker nécessite des fours à très haute température qui libèrent dans l'air une grande quantité de dioxyde de carbone (CO$_2$). + + Dans une cimenterie, la fabrication du clinker s'effectue de 7 h 30 à 20 h, dans une pièce de volume \np{400000}~dm$^3$. + + À 20 h, après une journée de travail, le taux volumique de CO$_2$ dans la pièce est de 0.7\,\%. + \begin{enumerate} + \item Justifier que le volume de CO$_2$ présent dans cette pièce à 20 h est de \np{2800}~dm$^3$ . + \item On modélise le volume de CO$_2$ présent dans la pièce par une fonction du temps $t$ écoulé après 20h (exprimé en minutes) qui pour formule $V(t) = V_0e^{-0.06t} + 400$ + \begin{enumerate} + \item Démontrer que $V_0$ est égale à \np{2400}. + \item Quel sera, au dm$^3$ près, le volume de CO$_2$ dans cette pièce à 23 h ? + \item Démontrer que $V'(t) = - 144.0 e^{- 0.06 t}$. + \item Étudier le signe de $V'(t)$ puis en déduire le sens de variation de $V(t)$. + \item Que peut-on dire du volume de CO$_2$ quand $t$ devient grand? + \end{enumerate} + \end{enumerate} +\end{exercise} + +\begin{solution} + \begin{enumerate} + \item Volume à 20h: $400000\times 0.006999999999999999 = 2800$ + \item + \begin{enumerate} + \item $t=0$ correspond à 20h. + + Donc $V(0) = 2800 = V_0e^{-0.06\times 0} + 400 = V_0 + 400$ + + Donc $V_0 = 2800 - 400 = 2400$ + \item Il faut calculer $V(t)$ pour $t = 3$ donc + \[ + V(3) = 2404.65 + \] + \item Pas de correction pour cette question. + \item Pas de correction pour cette question. + \item Pas de correction pour cette question. + \end{enumerate} + \end{enumerate} +\end{solution} + + +\end{document} + +%%% Local Variables: +%%% mode: latex +%%% TeX-master: "master" +%%% End: diff --git a/TST_sti2d/DM/2102_DM1/04_2102_DM1.tex b/TST_sti2d/DM/2102_DM1/04_2102_DM1.tex new file mode 100644 index 0000000..ccbcdec --- /dev/null +++ b/TST_sti2d/DM/2102_DM1/04_2102_DM1.tex @@ -0,0 +1,121 @@ +\documentclass[a4paper,10pt]{article} +\usepackage{myXsim} +\usepackage{tasks} + +% Title Page +\title{DM1 \hfill BUDIN Nathan} +\tribe{TST sti2d} +\date{\hfillÀ render pour le jeudi 25 février} + +\xsimsetup{ + solution/print = false +} + +\begin{document} +\maketitle + +\begin{exercise}[subtitle={Complexes}] + \begin{enumerate} + \item Mettre le nombre complexe suivant sous forme algébrique $z_1 = \dfrac{7 + 2 i}{-2 + 3 i} $ + \item Mettre le complexe suivante sous forme exponentielle $z_2 = - 6 \sqrt{3} - 6 i$ + \item Mettre le complexe suivante sous forme exponentielle $z_3 = - 4 \sqrt{3} + 4 i$ + \item Calculer le produit $z_4=z_2\times z_3$ donner le résultat sous forme exponentielle puis algébrique. + \item Calculer le quotient $z_5=\frac{z_2}{z_3}$ donner le résultat sous forme exponentielle puis algébrique. + \end{enumerate} +\end{exercise} + +\begin{solution} + \begin{enumerate} + \item $z_1 = - \frac{8}{13} - \frac{25 i}{13}$ + \item $z_3 = 12 e^{- \frac{5 i \pi}{6}}$ + \item $z_4 = 96 e^{0} = 96 = 96.0$ + \item $z_5 = \frac{3}{2} e^{- \frac{5 i \pi}{3}} = \frac{3}{4} + \frac{3 \sqrt{3} i}{4} = 0.75 + 1.3 i$ + \end{enumerate} +\end{solution} + +\begin{exercise}[subtitle={Bassin}] + Le tour d'un bassin au niveau du sol présente deux axes de symétrie : l’axe des abscisses et la droite d’équation $x=4$. Il est obtenu par symétrie de la courbe $\mathcal{C_f}$ sur $\intFF{0}{4}$ où $f$ est la fonction définie par + + \[ + f(x) = \left(- x^{2} + 4.9 x - 6.0\right) e^{- x} + 6.0 + \] + On admet que sur $\intFF{0}{4}$ la fonction $f$ est positive. + \begin{enumerate} + \item Sur un repère, tracer l'allure de la courbe $\mathcal{C}_f$, les axes de symétries puis compléter pour dessiner la forme du bassin. + \item Montrer que la fonction $f$ admet comme primitive sur $\R$ la fonction $F$ définie par + \[ + F(x) = 6.0 x + \left( x^{2} - 2.9 x + 3.1\right) e^{- x} + \] + \item Calculer la quantité $\ds \int_0^4 f(x) \; dx$, vous donnerez le résultat sous forme exacte. Interpréter le résultat et reportez cette quantité sur le graphique. + \item On considère que l'échelle de votre graphique est de 1unité pour 15m. Calculer l'aire du bassin. Vous donnerez un résultat arrondi au $m^2$ près. + \end{enumerate} +\end{exercise} + +\begin{solution} + \begin{enumerate} + \item + \begin{tikzpicture}[baseline=(a.north), xscale=1, yscale=0.5] + \tkzInit[xmin=0,xmax=5,xstep=1, + ymin=0,ymax=10,ystep=1] + \tkzGrid + \tkzAxeXY + \tkzFct[domain=0:10,color=red,very thick]% + { (-x**2 + 4.9*x - 6.0)*exp(-x) + 6.0 }; + \end{tikzpicture} + \item Il faut dériver $F(x)$ et vérifier que $F'(x) = f(x)$. + \item $\ds \int_0^4 f(x) \; dx = F(4) - F(0) = \frac{7.5}{e^{4}} + 20.9$ + \item La quantité calculée à la question précédente se retrouve 4fois pour former le bassin. Il faut ensuite prendre en compte l'échelle, comme 1unité de longueur correspond à 15m, une unité d'air correspond à $15\times15 = 225m^2$. Ainsi l'aire du bassin est égale à + \[ + (\frac{7.5}{e^{4}} + 20.9)\times 4 \times 15^2 = 18934.00000 + \] + + \end{enumerate} +\end{solution} + +\begin{exercise}[subtitle={Bassin}] + Le clinker est un constituant du ciment qui résulte de la cuisson d'un mélange composé de calcaire et d'argile. La fabrication du clinker nécessite des fours à très haute température qui libèrent dans l'air une grande quantité de dioxyde de carbone (CO$_2$). + + Dans une cimenterie, la fabrication du clinker s'effectue de 7 h 30 à 20 h, dans une pièce de volume \np{400000}~dm$^3$. + + À 20 h, après une journée de travail, le taux volumique de CO$_2$ dans la pièce est de 0.6\,\%. + \begin{enumerate} + \item Justifier que le volume de CO$_2$ présent dans cette pièce à 20 h est de \np{2400}~dm$^3$ . + \item On modélise le volume de CO$_2$ présent dans la pièce par une fonction du temps $t$ écoulé après 20h (exprimé en minutes) qui pour formule $V(t) = V_0e^{-0.0t} + 580$ + \begin{enumerate} + \item Démontrer que $V_0$ est égale à \np{1820}. + \item Quel sera, au dm$^3$ près, le volume de CO$_2$ dans cette pièce à 22 h ? + \item Démontrer que $V'(t) = 0$. + \item Étudier le signe de $V'(t)$ puis en déduire le sens de variation de $V(t)$. + \item Que peut-on dire du volume de CO$_2$ quand $t$ devient grand? + \end{enumerate} + \end{enumerate} +\end{exercise} + +\begin{solution} + \begin{enumerate} + \item Volume à 20h: $400000\times 0.006 = 2400$ + \item + \begin{enumerate} + \item $t=0$ correspond à 20h. + + Donc $V(0) = 2400 = V_0e^{-0.0\times 0} + 580 = V_0 + 580$ + + Donc $V_0 = 2400 - 580 = 1820$ + \item Il faut calculer $V(t)$ pour $t = 2$ donc + \[ + V(2) = 2400 + \] + \item Pas de correction pour cette question. + \item Pas de correction pour cette question. + \item Pas de correction pour cette question. + \end{enumerate} + \end{enumerate} +\end{solution} + + +\end{document} + +%%% Local Variables: +%%% mode: latex +%%% TeX-master: "master" +%%% End: diff --git a/TST_sti2d/DM/2102_DM1/05_2102_DM1.tex b/TST_sti2d/DM/2102_DM1/05_2102_DM1.tex new file mode 100644 index 0000000..fc3e513 --- /dev/null +++ b/TST_sti2d/DM/2102_DM1/05_2102_DM1.tex @@ -0,0 +1,121 @@ +\documentclass[a4paper,10pt]{article} +\usepackage{myXsim} +\usepackage{tasks} + +% Title Page +\title{DM1 \hfill CHION Léa} +\tribe{TST sti2d} +\date{\hfillÀ render pour le jeudi 25 février} + +\xsimsetup{ + solution/print = false +} + +\begin{document} +\maketitle + +\begin{exercise}[subtitle={Complexes}] + \begin{enumerate} + \item Mettre le nombre complexe suivant sous forme algébrique $z_1 = \dfrac{9 + 6 i}{-3 + 5 i} $ + \item Mettre le complexe suivante sous forme exponentielle $z_2 = -7 + 7 \sqrt{3} i$ + \item Mettre le complexe suivante sous forme exponentielle $z_3 = -5 - 5 \sqrt{3} i$ + \item Calculer le produit $z_4=z_2\times z_3$ donner le résultat sous forme exponentielle puis algébrique. + \item Calculer le quotient $z_5=\frac{z_2}{z_3}$ donner le résultat sous forme exponentielle puis algébrique. + \end{enumerate} +\end{exercise} + +\begin{solution} + \begin{enumerate} + \item $z_1 = \frac{3}{34} - \frac{63 i}{34}$ + \item $z_3 = 14 e^{\frac{2 i \pi}{3}}$ + \item $z_4 = 140 e^{0} = 140 = 140.0$ + \item $z_5 = \frac{7}{5} e^{\frac{4 i \pi}{3}} = - \frac{7}{10} - \frac{7 \sqrt{3} i}{10} = -0.7 - 1.21 i$ + \end{enumerate} +\end{solution} + +\begin{exercise}[subtitle={Bassin}] + Le tour d'un bassin au niveau du sol présente deux axes de symétrie : l’axe des abscisses et la droite d’équation $x=4$. Il est obtenu par symétrie de la courbe $\mathcal{C_f}$ sur $\intFF{0}{4}$ où $f$ est la fonction définie par + + \[ + f(x) = \left(- x^{2} + 6.2 x - 7.3\right) e^{- x} + 7.3 + \] + On admet que sur $\intFF{0}{4}$ la fonction $f$ est positive. + \begin{enumerate} + \item Sur un repère, tracer l'allure de la courbe $\mathcal{C}_f$, les axes de symétries puis compléter pour dessiner la forme du bassin. + \item Montrer que la fonction $f$ admet comme primitive sur $\R$ la fonction $F$ définie par + \[ + F(x) = 7.3 x + \left( x^{2} - 4.2 x + 3.1\right) e^{- x} + \] + \item Calculer la quantité $\ds \int_0^4 f(x) \; dx$, vous donnerez le résultat sous forme exacte. Interpréter le résultat et reportez cette quantité sur le graphique. + \item On considère que l'échelle de votre graphique est de 1unité pour 15m. Calculer l'aire du bassin. Vous donnerez un résultat arrondi au $m^2$ près. + \end{enumerate} +\end{exercise} + +\begin{solution} + \begin{enumerate} + \item + \begin{tikzpicture}[baseline=(a.north), xscale=1, yscale=0.5] + \tkzInit[xmin=0,xmax=5,xstep=1, + ymin=0,ymax=10,ystep=1] + \tkzGrid + \tkzAxeXY + \tkzFct[domain=0:10,color=red,very thick]% + { (-x**2 + 6.2*x - 7.3)*exp(-x) + 7.3 }; + \end{tikzpicture} + \item Il faut dériver $F(x)$ et vérifier que $F'(x) = f(x)$. + \item $\ds \int_0^4 f(x) \; dx = F(4) - F(0) = \frac{2.3}{e^{4}} + 26.1$ + \item La quantité calculée à la question précédente se retrouve 4fois pour former le bassin. Il faut ensuite prendre en compte l'échelle, comme 1unité de longueur correspond à 15m, une unité d'air correspond à $15\times15 = 225m^2$. Ainsi l'aire du bassin est égale à + \[ + (\frac{2.3}{e^{4}} + 26.1)\times 4 \times 15^2 = 23528.00000 + \] + + \end{enumerate} +\end{solution} + +\begin{exercise}[subtitle={Bassin}] + Le clinker est un constituant du ciment qui résulte de la cuisson d'un mélange composé de calcaire et d'argile. La fabrication du clinker nécessite des fours à très haute température qui libèrent dans l'air une grande quantité de dioxyde de carbone (CO$_2$). + + Dans une cimenterie, la fabrication du clinker s'effectue de 7 h 30 à 20 h, dans une pièce de volume \np{300000}~dm$^3$. + + À 20 h, après une journée de travail, le taux volumique de CO$_2$ dans la pièce est de 0.6\,\%. + \begin{enumerate} + \item Justifier que le volume de CO$_2$ présent dans cette pièce à 20 h est de \np{1800}~dm$^3$ . + \item On modélise le volume de CO$_2$ présent dans la pièce par une fonction du temps $t$ écoulé après 20h (exprimé en minutes) qui pour formule $V(t) = V_0e^{-0.01t} + 260$ + \begin{enumerate} + \item Démontrer que $V_0$ est égale à \np{1540}. + \item Quel sera, au dm$^3$ près, le volume de CO$_2$ dans cette pièce à 22 h ? + \item Démontrer que $V'(t) = - 15.4 e^{- 0.01 t}$. + \item Étudier le signe de $V'(t)$ puis en déduire le sens de variation de $V(t)$. + \item Que peut-on dire du volume de CO$_2$ quand $t$ devient grand? + \end{enumerate} + \end{enumerate} +\end{exercise} + +\begin{solution} + \begin{enumerate} + \item Volume à 20h: $300000\times 0.006 = 1800$ + \item + \begin{enumerate} + \item $t=0$ correspond à 20h. + + Donc $V(0) = 1800 = V_0e^{-0.01\times 0} + 260 = V_0 + 260$ + + Donc $V_0 = 1800 - 260 = 1540$ + \item Il faut calculer $V(t)$ pour $t = 2$ donc + \[ + V(2) = 1769.51 + \] + \item Pas de correction pour cette question. + \item Pas de correction pour cette question. + \item Pas de correction pour cette question. + \end{enumerate} + \end{enumerate} +\end{solution} + + +\end{document} + +%%% Local Variables: +%%% mode: latex +%%% TeX-master: "master" +%%% End: diff --git a/TST_sti2d/DM/2102_DM1/06_2102_DM1.tex b/TST_sti2d/DM/2102_DM1/06_2102_DM1.tex new file mode 100644 index 0000000..0393171 --- /dev/null +++ b/TST_sti2d/DM/2102_DM1/06_2102_DM1.tex @@ -0,0 +1,121 @@ +\documentclass[a4paper,10pt]{article} +\usepackage{myXsim} +\usepackage{tasks} + +% Title Page +\title{DM1 \hfill CLAIN Avinash} +\tribe{TST sti2d} +\date{\hfillÀ render pour le jeudi 25 février} + +\xsimsetup{ + solution/print = false +} + +\begin{document} +\maketitle + +\begin{exercise}[subtitle={Complexes}] + \begin{enumerate} + \item Mettre le nombre complexe suivant sous forme algébrique $z_1 = \dfrac{4 + 10 i}{-8 + 3 i} $ + \item Mettre le complexe suivante sous forme exponentielle $z_2 = - 4 \sqrt{3} + 4 i$ + \item Mettre le complexe suivante sous forme exponentielle $z_3 = 2 \sqrt{2} - 2 \sqrt{2} i$ + \item Calculer le produit $z_4=z_2\times z_3$ donner le résultat sous forme exponentielle puis algébrique. + \item Calculer le quotient $z_5=\frac{z_2}{z_3}$ donner le résultat sous forme exponentielle puis algébrique. + \end{enumerate} +\end{exercise} + +\begin{solution} + \begin{enumerate} + \item $z_1 = - \frac{2}{73} - \frac{92 i}{73}$ + \item $z_3 = 8 e^{\frac{5 i \pi}{6}}$ + \item $z_4 = 32 e^{\frac{7 i \pi}{12}} = - 8 \sqrt{6} + 8 \sqrt{2} + i \left(8 \sqrt{2} + 8 \sqrt{6}\right) = -8.28 + 30.9 i$ + \item $z_5 = 2 e^{\frac{13 i \pi}{12}} = - \frac{\sqrt{6}}{2} - \frac{\sqrt{2}}{2} + i \left(- \frac{\sqrt{6}}{2} + \frac{\sqrt{2}}{2}\right) = -1.93 - 0.518 i$ + \end{enumerate} +\end{solution} + +\begin{exercise}[subtitle={Bassin}] + Le tour d'un bassin au niveau du sol présente deux axes de symétrie : l’axe des abscisses et la droite d’équation $x=4$. Il est obtenu par symétrie de la courbe $\mathcal{C_f}$ sur $\intFF{0}{4}$ où $f$ est la fonction définie par + + \[ + f(x) = \left(- x^{2} + 1.8 x - 4.9\right) e^{- x} + 4.9 + \] + On admet que sur $\intFF{0}{4}$ la fonction $f$ est positive. + \begin{enumerate} + \item Sur un repère, tracer l'allure de la courbe $\mathcal{C}_f$, les axes de symétries puis compléter pour dessiner la forme du bassin. + \item Montrer que la fonction $f$ admet comme primitive sur $\R$ la fonction $F$ définie par + \[ + F(x) = 4.9 x + \left( x^{2} + 0.2 x + 5.1\right) e^{- x} + \] + \item Calculer la quantité $\ds \int_0^4 f(x) \; dx$, vous donnerez le résultat sous forme exacte. Interpréter le résultat et reportez cette quantité sur le graphique. + \item On considère que l'échelle de votre graphique est de 1unité pour 15m. Calculer l'aire du bassin. Vous donnerez un résultat arrondi au $m^2$ près. + \end{enumerate} +\end{exercise} + +\begin{solution} + \begin{enumerate} + \item + \begin{tikzpicture}[baseline=(a.north), xscale=1, yscale=0.5] + \tkzInit[xmin=0,xmax=5,xstep=1, + ymin=0,ymax=10,ystep=1] + \tkzGrid + \tkzAxeXY + \tkzFct[domain=0:10,color=red,very thick]% + { (-x**2 + 1.8*x - 4.9)*exp(-x) + 4.9 }; + \end{tikzpicture} + \item Il faut dériver $F(x)$ et vérifier que $F'(x) = f(x)$. + \item $\ds \int_0^4 f(x) \; dx = F(4) - F(0) = \frac{21.9}{e^{4}} + 14.5$ + \item La quantité calculée à la question précédente se retrouve 4fois pour former le bassin. Il faut ensuite prendre en compte l'échelle, comme 1unité de longueur correspond à 15m, une unité d'air correspond à $15\times15 = 225m^2$. Ainsi l'aire du bassin est égale à + \[ + (\frac{21.9}{e^{4}} + 14.5)\times 4 \times 15^2 = 13411.00000 + \] + + \end{enumerate} +\end{solution} + +\begin{exercise}[subtitle={Bassin}] + Le clinker est un constituant du ciment qui résulte de la cuisson d'un mélange composé de calcaire et d'argile. La fabrication du clinker nécessite des fours à très haute température qui libèrent dans l'air une grande quantité de dioxyde de carbone (CO$_2$). + + Dans une cimenterie, la fabrication du clinker s'effectue de 7 h 30 à 20 h, dans une pièce de volume \np{1000000}~dm$^3$. + + À 20 h, après une journée de travail, le taux volumique de CO$_2$ dans la pièce est de 0.8\,\%. + \begin{enumerate} + \item Justifier que le volume de CO$_2$ présent dans cette pièce à 20 h est de \np{8000}~dm$^3$ . + \item On modélise le volume de CO$_2$ présent dans la pièce par une fonction du temps $t$ écoulé après 20h (exprimé en minutes) qui pour formule $V(t) = V_0e^{-0.09t} + 580$ + \begin{enumerate} + \item Démontrer que $V_0$ est égale à \np{7420}. + \item Quel sera, au dm$^3$ près, le volume de CO$_2$ dans cette pièce à 21 h ? + \item Démontrer que $V'(t) = - 667.8 e^{- 0.09 t}$. + \item Étudier le signe de $V'(t)$ puis en déduire le sens de variation de $V(t)$. + \item Que peut-on dire du volume de CO$_2$ quand $t$ devient grand? + \end{enumerate} + \end{enumerate} +\end{exercise} + +\begin{solution} + \begin{enumerate} + \item Volume à 20h: $1000000\times 0.008 = 8000$ + \item + \begin{enumerate} + \item $t=0$ correspond à 20h. + + Donc $V(0) = 8000 = V_0e^{-0.09\times 0} + 580 = V_0 + 580$ + + Donc $V_0 = 8000 - 580 = 7420$ + \item Il faut calculer $V(t)$ pour $t = 1$ donc + \[ + V(1) = 7361.37 + \] + \item Pas de correction pour cette question. + \item Pas de correction pour cette question. + \item Pas de correction pour cette question. + \end{enumerate} + \end{enumerate} +\end{solution} + + +\end{document} + +%%% Local Variables: +%%% mode: latex +%%% TeX-master: "master" +%%% End: diff --git a/TST_sti2d/DM/2102_DM1/07_2102_DM1.tex b/TST_sti2d/DM/2102_DM1/07_2102_DM1.tex new file mode 100644 index 0000000..dbe902e --- /dev/null +++ b/TST_sti2d/DM/2102_DM1/07_2102_DM1.tex @@ -0,0 +1,121 @@ +\documentclass[a4paper,10pt]{article} +\usepackage{myXsim} +\usepackage{tasks} + +% Title Page +\title{DM1 \hfill COUBAT Alexis} +\tribe{TST sti2d} +\date{\hfillÀ render pour le jeudi 25 février} + +\xsimsetup{ + solution/print = false +} + +\begin{document} +\maketitle + +\begin{exercise}[subtitle={Complexes}] + \begin{enumerate} + \item Mettre le nombre complexe suivant sous forme algébrique $z_1 = \dfrac{7 + 3 i}{-2 + 2 i} $ + \item Mettre le complexe suivante sous forme exponentielle $z_2 = 1 - \sqrt{3} i$ + \item Mettre le complexe suivante sous forme exponentielle $z_3 = - 2 \sqrt{3} + 2 i$ + \item Calculer le produit $z_4=z_2\times z_3$ donner le résultat sous forme exponentielle puis algébrique. + \item Calculer le quotient $z_5=\frac{z_2}{z_3}$ donner le résultat sous forme exponentielle puis algébrique. + \end{enumerate} +\end{exercise} + +\begin{solution} + \begin{enumerate} + \item $z_1 = -1 - \frac{5 i}{2}$ + \item $z_3 = 2 e^{- \frac{i \pi}{3}}$ + \item $z_4 = 8 e^{\frac{i \pi}{2}} = 8 i = 8.0 i$ + \item $z_5 = \frac{1}{2} e^{- \frac{7 i \pi}{6}} = - \frac{\sqrt{3}}{4} + \frac{i}{4} = -0.433 + 0.25 i$ + \end{enumerate} +\end{solution} + +\begin{exercise}[subtitle={Bassin}] + Le tour d'un bassin au niveau du sol présente deux axes de symétrie : l’axe des abscisses et la droite d’équation $x=4$. Il est obtenu par symétrie de la courbe $\mathcal{C_f}$ sur $\intFF{0}{4}$ où $f$ est la fonction définie par + + \[ + f(x) = \left(- x^{2} + 5.6 x - 4.5\right) e^{- x} + 4.5 + \] + On admet que sur $\intFF{0}{4}$ la fonction $f$ est positive. + \begin{enumerate} + \item Sur un repère, tracer l'allure de la courbe $\mathcal{C}_f$, les axes de symétries puis compléter pour dessiner la forme du bassin. + \item Montrer que la fonction $f$ admet comme primitive sur $\R$ la fonction $F$ définie par + \[ + F(x) = 4.5 x + \left( x^{2} - 3.6 x + 0.9\right) e^{- x} + \] + \item Calculer la quantité $\ds \int_0^4 f(x) \; dx$, vous donnerez le résultat sous forme exacte. Interpréter le résultat et reportez cette quantité sur le graphique. + \item On considère que l'échelle de votre graphique est de 1unité pour 15m. Calculer l'aire du bassin. Vous donnerez un résultat arrondi au $m^2$ près. + \end{enumerate} +\end{exercise} + +\begin{solution} + \begin{enumerate} + \item + \begin{tikzpicture}[baseline=(a.north), xscale=1, yscale=0.5] + \tkzInit[xmin=0,xmax=5,xstep=1, + ymin=0,ymax=10,ystep=1] + \tkzGrid + \tkzAxeXY + \tkzFct[domain=0:10,color=red,very thick]% + { (-x**2 + 5.6*x - 4.5)*exp(-x) + 4.5 }; + \end{tikzpicture} + \item Il faut dériver $F(x)$ et vérifier que $F'(x) = f(x)$. + \item $\ds \int_0^4 f(x) \; dx = F(4) - F(0) = \frac{2.5}{e^{4}} + 17.1$ + \item La quantité calculée à la question précédente se retrouve 4fois pour former le bassin. Il faut ensuite prendre en compte l'échelle, comme 1unité de longueur correspond à 15m, une unité d'air correspond à $15\times15 = 225m^2$. Ainsi l'aire du bassin est égale à + \[ + (\frac{2.5}{e^{4}} + 17.1)\times 4 \times 15^2 = 15431.00000 + \] + + \end{enumerate} +\end{solution} + +\begin{exercise}[subtitle={Bassin}] + Le clinker est un constituant du ciment qui résulte de la cuisson d'un mélange composé de calcaire et d'argile. La fabrication du clinker nécessite des fours à très haute température qui libèrent dans l'air une grande quantité de dioxyde de carbone (CO$_2$). + + Dans une cimenterie, la fabrication du clinker s'effectue de 7 h 30 à 20 h, dans une pièce de volume \np{800000}~dm$^3$. + + À 20 h, après une journée de travail, le taux volumique de CO$_2$ dans la pièce est de 0.7\,\%. + \begin{enumerate} + \item Justifier que le volume de CO$_2$ présent dans cette pièce à 20 h est de \np{5600}~dm$^3$ . + \item On modélise le volume de CO$_2$ présent dans la pièce par une fonction du temps $t$ écoulé après 20h (exprimé en minutes) qui pour formule $V(t) = V_0e^{-0.02t} + 340$ + \begin{enumerate} + \item Démontrer que $V_0$ est égale à \np{5260}. + \item Quel sera, au dm$^3$ près, le volume de CO$_2$ dans cette pièce à 24 h ? + \item Démontrer que $V'(t) = - 105.2 e^{- 0.02 t}$. + \item Étudier le signe de $V'(t)$ puis en déduire le sens de variation de $V(t)$. + \item Que peut-on dire du volume de CO$_2$ quand $t$ devient grand? + \end{enumerate} + \end{enumerate} +\end{exercise} + +\begin{solution} + \begin{enumerate} + \item Volume à 20h: $800000\times 0.006999999999999999 = 5600$ + \item + \begin{enumerate} + \item $t=0$ correspond à 20h. + + Donc $V(0) = 5600 = V_0e^{-0.02\times 0} + 340 = V_0 + 340$ + + Donc $V_0 = 5600 - 340 = 5260$ + \item Il faut calculer $V(t)$ pour $t = 4$ donc + \[ + V(4) = 5195.59 + \] + \item Pas de correction pour cette question. + \item Pas de correction pour cette question. + \item Pas de correction pour cette question. + \end{enumerate} + \end{enumerate} +\end{solution} + + +\end{document} + +%%% Local Variables: +%%% mode: latex +%%% TeX-master: "master" +%%% End: diff --git a/TST_sti2d/DM/2102_DM1/08_2102_DM1.tex b/TST_sti2d/DM/2102_DM1/08_2102_DM1.tex new file mode 100644 index 0000000..e5a1cfe --- /dev/null +++ b/TST_sti2d/DM/2102_DM1/08_2102_DM1.tex @@ -0,0 +1,121 @@ +\documentclass[a4paper,10pt]{article} +\usepackage{myXsim} +\usepackage{tasks} + +% Title Page +\title{DM1 \hfill EVRARD Jules} +\tribe{TST sti2d} +\date{\hfillÀ render pour le jeudi 25 février} + +\xsimsetup{ + solution/print = false +} + +\begin{document} +\maketitle + +\begin{exercise}[subtitle={Complexes}] + \begin{enumerate} + \item Mettre le nombre complexe suivant sous forme algébrique $z_1 = \dfrac{8 + 3 i}{-9 + 3 i} $ + \item Mettre le complexe suivante sous forme exponentielle $z_2 = 3 - 3 \sqrt{3} i$ + \item Mettre le complexe suivante sous forme exponentielle $z_3 = \sqrt{2} - \sqrt{2} i$ + \item Calculer le produit $z_4=z_2\times z_3$ donner le résultat sous forme exponentielle puis algébrique. + \item Calculer le quotient $z_5=\frac{z_2}{z_3}$ donner le résultat sous forme exponentielle puis algébrique. + \end{enumerate} +\end{exercise} + +\begin{solution} + \begin{enumerate} + \item $z_1 = - \frac{7}{10} - \frac{17 i}{30}$ + \item $z_3 = 6 e^{- \frac{i \pi}{3}}$ + \item $z_4 = 12 e^{- \frac{7 i \pi}{12}} = - 3 \sqrt{6} + 3 \sqrt{2} + i \left(- 3 \sqrt{6} - 3 \sqrt{2}\right) = -3.11 - 11.6 i$ + \item $z_5 = 3 e^{- \frac{i \pi}{12}} = \frac{3 \sqrt{2}}{4} + \frac{3 \sqrt{6}}{4} + i \left(- \frac{3 \sqrt{6}}{4} + \frac{3 \sqrt{2}}{4}\right) = 2.9 - 0.776 i$ + \end{enumerate} +\end{solution} + +\begin{exercise}[subtitle={Bassin}] + Le tour d'un bassin au niveau du sol présente deux axes de symétrie : l’axe des abscisses et la droite d’équation $x=4$. Il est obtenu par symétrie de la courbe $\mathcal{C_f}$ sur $\intFF{0}{4}$ où $f$ est la fonction définie par + + \[ + f(x) = \left(- x^{2} + 0.9 x - 3.2\right) e^{- x} + 3.2 + \] + On admet que sur $\intFF{0}{4}$ la fonction $f$ est positive. + \begin{enumerate} + \item Sur un repère, tracer l'allure de la courbe $\mathcal{C}_f$, les axes de symétries puis compléter pour dessiner la forme du bassin. + \item Montrer que la fonction $f$ admet comme primitive sur $\R$ la fonction $F$ définie par + \[ + F(x) = 3.2 x + \left( x^{2} + 1.1 x + 4.3\right) e^{- x} + \] + \item Calculer la quantité $\ds \int_0^4 f(x) \; dx$, vous donnerez le résultat sous forme exacte. Interpréter le résultat et reportez cette quantité sur le graphique. + \item On considère que l'échelle de votre graphique est de 1unité pour 15m. Calculer l'aire du bassin. Vous donnerez un résultat arrondi au $m^2$ près. + \end{enumerate} +\end{exercise} + +\begin{solution} + \begin{enumerate} + \item + \begin{tikzpicture}[baseline=(a.north), xscale=1, yscale=0.5] + \tkzInit[xmin=0,xmax=5,xstep=1, + ymin=0,ymax=10,ystep=1] + \tkzGrid + \tkzAxeXY + \tkzFct[domain=0:10,color=red,very thick]% + { (-x**2 + 0.9*x - 3.2)*exp(-x) + 3.2 }; + \end{tikzpicture} + \item Il faut dériver $F(x)$ et vérifier que $F'(x) = f(x)$. + \item $\ds \int_0^4 f(x) \; dx = F(4) - F(0) = \frac{24.7}{e^{4}} + 8.5$ + \item La quantité calculée à la question précédente se retrouve 4fois pour former le bassin. Il faut ensuite prendre en compte l'échelle, comme 1unité de longueur correspond à 15m, une unité d'air correspond à $15\times15 = 225m^2$. Ainsi l'aire du bassin est égale à + \[ + (\frac{24.7}{e^{4}} + 8.5)\times 4 \times 15^2 = 8057.000000 + \] + + \end{enumerate} +\end{solution} + +\begin{exercise}[subtitle={Bassin}] + Le clinker est un constituant du ciment qui résulte de la cuisson d'un mélange composé de calcaire et d'argile. La fabrication du clinker nécessite des fours à très haute température qui libèrent dans l'air une grande quantité de dioxyde de carbone (CO$_2$). + + Dans une cimenterie, la fabrication du clinker s'effectue de 7 h 30 à 20 h, dans une pièce de volume \np{800000}~dm$^3$. + + À 20 h, après une journée de travail, le taux volumique de CO$_2$ dans la pièce est de 1.0\,\%. + \begin{enumerate} + \item Justifier que le volume de CO$_2$ présent dans cette pièce à 20 h est de \np{8000}~dm$^3$ . + \item On modélise le volume de CO$_2$ présent dans la pièce par une fonction du temps $t$ écoulé après 20h (exprimé en minutes) qui pour formule $V(t) = V_0e^{-0.02t} + 560$ + \begin{enumerate} + \item Démontrer que $V_0$ est égale à \np{7440}. + \item Quel sera, au dm$^3$ près, le volume de CO$_2$ dans cette pièce à 23 h ? + \item Démontrer que $V'(t) = - 148.8 e^{- 0.02 t}$. + \item Étudier le signe de $V'(t)$ puis en déduire le sens de variation de $V(t)$. + \item Que peut-on dire du volume de CO$_2$ quand $t$ devient grand? + \end{enumerate} + \end{enumerate} +\end{exercise} + +\begin{solution} + \begin{enumerate} + \item Volume à 20h: $800000\times 0.01 = 8000$ + \item + \begin{enumerate} + \item $t=0$ correspond à 20h. + + Donc $V(0) = 8000 = V_0e^{-0.02\times 0} + 560 = V_0 + 560$ + + Donc $V_0 = 8000 - 560 = 7440$ + \item Il faut calculer $V(t)$ pour $t = 3$ donc + \[ + V(3) = 7566.73 + \] + \item Pas de correction pour cette question. + \item Pas de correction pour cette question. + \item Pas de correction pour cette question. + \end{enumerate} + \end{enumerate} +\end{solution} + + +\end{document} + +%%% Local Variables: +%%% mode: latex +%%% TeX-master: "master" +%%% End: diff --git a/TST_sti2d/DM/2102_DM1/09_2102_DM1.tex b/TST_sti2d/DM/2102_DM1/09_2102_DM1.tex new file mode 100644 index 0000000..8a26c0c --- /dev/null +++ b/TST_sti2d/DM/2102_DM1/09_2102_DM1.tex @@ -0,0 +1,121 @@ +\documentclass[a4paper,10pt]{article} +\usepackage{myXsim} +\usepackage{tasks} + +% Title Page +\title{DM1 \hfill HADJRAS Mohcine} +\tribe{TST sti2d} +\date{\hfillÀ render pour le jeudi 25 février} + +\xsimsetup{ + solution/print = false +} + +\begin{document} +\maketitle + +\begin{exercise}[subtitle={Complexes}] + \begin{enumerate} + \item Mettre le nombre complexe suivant sous forme algébrique $z_1 = \dfrac{5 + 3 i}{-5 + 7 i} $ + \item Mettre le complexe suivante sous forme exponentielle $z_2 = 2 \sqrt{3} + 2 i$ + \item Mettre le complexe suivante sous forme exponentielle $z_3 = -10 - 10 \sqrt{3} i$ + \item Calculer le produit $z_4=z_2\times z_3$ donner le résultat sous forme exponentielle puis algébrique. + \item Calculer le quotient $z_5=\frac{z_2}{z_3}$ donner le résultat sous forme exponentielle puis algébrique. + \end{enumerate} +\end{exercise} + +\begin{solution} + \begin{enumerate} + \item $z_1 = - \frac{2}{37} - \frac{25 i}{37}$ + \item $z_3 = 4 e^{\frac{i \pi}{6}}$ + \item $z_4 = 80 e^{- \frac{i \pi}{2}} = - 80 i = - 80.0 i$ + \item $z_5 = \frac{1}{5} e^{\frac{5 i \pi}{6}} = - \frac{\sqrt{3}}{10} + \frac{i}{10} = -0.173 + 0.1 i$ + \end{enumerate} +\end{solution} + +\begin{exercise}[subtitle={Bassin}] + Le tour d'un bassin au niveau du sol présente deux axes de symétrie : l’axe des abscisses et la droite d’équation $x=4$. Il est obtenu par symétrie de la courbe $\mathcal{C_f}$ sur $\intFF{0}{4}$ où $f$ est la fonction définie par + + \[ + f(x) = \left(- x^{2} + 7.5 x - 9.0\right) e^{- x} + 9.0 + \] + On admet que sur $\intFF{0}{4}$ la fonction $f$ est positive. + \begin{enumerate} + \item Sur un repère, tracer l'allure de la courbe $\mathcal{C}_f$, les axes de symétries puis compléter pour dessiner la forme du bassin. + \item Montrer que la fonction $f$ admet comme primitive sur $\R$ la fonction $F$ définie par + \[ + F(x) = 9.0 x + \left( x^{2} - 5.5 x + 3.5\right) e^{- x} + \] + \item Calculer la quantité $\ds \int_0^4 f(x) \; dx$, vous donnerez le résultat sous forme exacte. Interpréter le résultat et reportez cette quantité sur le graphique. + \item On considère que l'échelle de votre graphique est de 1unité pour 15m. Calculer l'aire du bassin. Vous donnerez un résultat arrondi au $m^2$ près. + \end{enumerate} +\end{exercise} + +\begin{solution} + \begin{enumerate} + \item + \begin{tikzpicture}[baseline=(a.north), xscale=1, yscale=0.5] + \tkzInit[xmin=0,xmax=5,xstep=1, + ymin=0,ymax=10,ystep=1] + \tkzGrid + \tkzAxeXY + \tkzFct[domain=0:10,color=red,very thick]% + { (-x**2 + 7.5*x - 9.0)*exp(-x) + 9.0 }; + \end{tikzpicture} + \item Il faut dériver $F(x)$ et vérifier que $F'(x) = f(x)$. + \item $\ds \int_0^4 f(x) \; dx = F(4) - F(0) = 32.5 - \frac{2.5}{e^{4}}$ + \item La quantité calculée à la question précédente se retrouve 4fois pour former le bassin. Il faut ensuite prendre en compte l'échelle, comme 1unité de longueur correspond à 15m, une unité d'air correspond à $15\times15 = 225m^2$. Ainsi l'aire du bassin est égale à + \[ + (32.5 - \frac{2.5}{e^{4}})\times 4 \times 15^2 = 29209.00000 + \] + + \end{enumerate} +\end{solution} + +\begin{exercise}[subtitle={Bassin}] + Le clinker est un constituant du ciment qui résulte de la cuisson d'un mélange composé de calcaire et d'argile. La fabrication du clinker nécessite des fours à très haute température qui libèrent dans l'air une grande quantité de dioxyde de carbone (CO$_2$). + + Dans une cimenterie, la fabrication du clinker s'effectue de 7 h 30 à 20 h, dans une pièce de volume \np{1000000}~dm$^3$. + + À 20 h, après une journée de travail, le taux volumique de CO$_2$ dans la pièce est de 0.9\,\%. + \begin{enumerate} + \item Justifier que le volume de CO$_2$ présent dans cette pièce à 20 h est de \np{9000}~dm$^3$ . + \item On modélise le volume de CO$_2$ présent dans la pièce par une fonction du temps $t$ écoulé après 20h (exprimé en minutes) qui pour formule $V(t) = V_0e^{-0.01t} + 330$ + \begin{enumerate} + \item Démontrer que $V_0$ est égale à \np{8670}. + \item Quel sera, au dm$^3$ près, le volume de CO$_2$ dans cette pièce à 24 h ? + \item Démontrer que $V'(t) = - 86.7 e^{- 0.01 t}$. + \item Étudier le signe de $V'(t)$ puis en déduire le sens de variation de $V(t)$. + \item Que peut-on dire du volume de CO$_2$ quand $t$ devient grand? + \end{enumerate} + \end{enumerate} +\end{exercise} + +\begin{solution} + \begin{enumerate} + \item Volume à 20h: $1000000\times 0.009000000000000001 = 9000$ + \item + \begin{enumerate} + \item $t=0$ correspond à 20h. + + Donc $V(0) = 9000 = V_0e^{-0.01\times 0} + 330 = V_0 + 330$ + + Donc $V_0 = 9000 - 330 = 8670$ + \item Il faut calculer $V(t)$ pour $t = 4$ donc + \[ + V(4) = 8660.04 + \] + \item Pas de correction pour cette question. + \item Pas de correction pour cette question. + \item Pas de correction pour cette question. + \end{enumerate} + \end{enumerate} +\end{solution} + + +\end{document} + +%%% Local Variables: +%%% mode: latex +%%% TeX-master: "master" +%%% End: diff --git a/TST_sti2d/DM/2102_DM1/10_2102_DM1.tex b/TST_sti2d/DM/2102_DM1/10_2102_DM1.tex new file mode 100644 index 0000000..c9e59c5 --- /dev/null +++ b/TST_sti2d/DM/2102_DM1/10_2102_DM1.tex @@ -0,0 +1,121 @@ +\documentclass[a4paper,10pt]{article} +\usepackage{myXsim} +\usepackage{tasks} + +% Title Page +\title{DM1 \hfill HENRIST Maxime} +\tribe{TST sti2d} +\date{\hfillÀ render pour le jeudi 25 février} + +\xsimsetup{ + solution/print = false +} + +\begin{document} +\maketitle + +\begin{exercise}[subtitle={Complexes}] + \begin{enumerate} + \item Mettre le nombre complexe suivant sous forme algébrique $z_1 = \dfrac{6 + 6 i}{-2 + 7 i} $ + \item Mettre le complexe suivante sous forme exponentielle $z_2 = 1 - \sqrt{3} i$ + \item Mettre le complexe suivante sous forme exponentielle $z_3 = 3 \sqrt{2} + 3 \sqrt{2} i$ + \item Calculer le produit $z_4=z_2\times z_3$ donner le résultat sous forme exponentielle puis algébrique. + \item Calculer le quotient $z_5=\frac{z_2}{z_3}$ donner le résultat sous forme exponentielle puis algébrique. + \end{enumerate} +\end{exercise} + +\begin{solution} + \begin{enumerate} + \item $z_1 = \frac{30}{53} - \frac{54 i}{53}$ + \item $z_3 = 2 e^{- \frac{i \pi}{3}}$ + \item $z_4 = 12 e^{- \frac{i \pi}{12}} = 3 \sqrt{2} + 3 \sqrt{6} + i \left(- 3 \sqrt{6} + 3 \sqrt{2}\right) = 11.6 - 3.11 i$ + \item $z_5 = \frac{1}{3} e^{- \frac{7 i \pi}{12}} = - \frac{\sqrt{6}}{12} + \frac{\sqrt{2}}{12} + i \left(- \frac{\sqrt{6}}{12} - \frac{\sqrt{2}}{12}\right) = -0.0863 - 0.322 i$ + \end{enumerate} +\end{solution} + +\begin{exercise}[subtitle={Bassin}] + Le tour d'un bassin au niveau du sol présente deux axes de symétrie : l’axe des abscisses et la droite d’équation $x=4$. Il est obtenu par symétrie de la courbe $\mathcal{C_f}$ sur $\intFF{0}{4}$ où $f$ est la fonction définie par + + \[ + f(x) = \left(- x^{2} + 1.3 x - 8.0\right) e^{- x} + 8.0 + \] + On admet que sur $\intFF{0}{4}$ la fonction $f$ est positive. + \begin{enumerate} + \item Sur un repère, tracer l'allure de la courbe $\mathcal{C}_f$, les axes de symétries puis compléter pour dessiner la forme du bassin. + \item Montrer que la fonction $f$ admet comme primitive sur $\R$ la fonction $F$ définie par + \[ + F(x) = 8.0 x + \left( x^{2} + 0.7 x + 8.7\right) e^{- x} + \] + \item Calculer la quantité $\ds \int_0^4 f(x) \; dx$, vous donnerez le résultat sous forme exacte. Interpréter le résultat et reportez cette quantité sur le graphique. + \item On considère que l'échelle de votre graphique est de 1unité pour 15m. Calculer l'aire du bassin. Vous donnerez un résultat arrondi au $m^2$ près. + \end{enumerate} +\end{exercise} + +\begin{solution} + \begin{enumerate} + \item + \begin{tikzpicture}[baseline=(a.north), xscale=1, yscale=0.5] + \tkzInit[xmin=0,xmax=5,xstep=1, + ymin=0,ymax=10,ystep=1] + \tkzGrid + \tkzAxeXY + \tkzFct[domain=0:10,color=red,very thick]% + { (-x**2 + 1.3*x - 8.0)*exp(-x) + 8.0 }; + \end{tikzpicture} + \item Il faut dériver $F(x)$ et vérifier que $F'(x) = f(x)$. + \item $\ds \int_0^4 f(x) \; dx = F(4) - F(0) = \frac{27.5}{e^{4}} + 23.3$ + \item La quantité calculée à la question précédente se retrouve 4fois pour former le bassin. Il faut ensuite prendre en compte l'échelle, comme 1unité de longueur correspond à 15m, une unité d'air correspond à $15\times15 = 225m^2$. Ainsi l'aire du bassin est égale à + \[ + (\frac{27.5}{e^{4}} + 23.3)\times 4 \times 15^2 = 21423.00000 + \] + + \end{enumerate} +\end{solution} + +\begin{exercise}[subtitle={Bassin}] + Le clinker est un constituant du ciment qui résulte de la cuisson d'un mélange composé de calcaire et d'argile. La fabrication du clinker nécessite des fours à très haute température qui libèrent dans l'air une grande quantité de dioxyde de carbone (CO$_2$). + + Dans une cimenterie, la fabrication du clinker s'effectue de 7 h 30 à 20 h, dans une pièce de volume \np{900000}~dm$^3$. + + À 20 h, après une journée de travail, le taux volumique de CO$_2$ dans la pièce est de 0.7\,\%. + \begin{enumerate} + \item Justifier que le volume de CO$_2$ présent dans cette pièce à 20 h est de \np{6300}~dm$^3$ . + \item On modélise le volume de CO$_2$ présent dans la pièce par une fonction du temps $t$ écoulé après 20h (exprimé en minutes) qui pour formule $V(t) = V_0e^{-0.1t} + 390$ + \begin{enumerate} + \item Démontrer que $V_0$ est égale à \np{5910}. + \item Quel sera, au dm$^3$ près, le volume de CO$_2$ dans cette pièce à 22 h ? + \item Démontrer que $V'(t) = - 591.0 e^{- 0.1 t}$. + \item Étudier le signe de $V'(t)$ puis en déduire le sens de variation de $V(t)$. + \item Que peut-on dire du volume de CO$_2$ quand $t$ devient grand? + \end{enumerate} + \end{enumerate} +\end{exercise} + +\begin{solution} + \begin{enumerate} + \item Volume à 20h: $900000\times 0.006999999999999999 = 6300$ + \item + \begin{enumerate} + \item $t=0$ correspond à 20h. + + Donc $V(0) = 6300 = V_0e^{-0.1\times 0} + 390 = V_0 + 390$ + + Donc $V_0 = 6300 - 390 = 5910$ + \item Il faut calculer $V(t)$ pour $t = 2$ donc + \[ + V(2) = 5228.70 + \] + \item Pas de correction pour cette question. + \item Pas de correction pour cette question. + \item Pas de correction pour cette question. + \end{enumerate} + \end{enumerate} +\end{solution} + + +\end{document} + +%%% Local Variables: +%%% mode: latex +%%% TeX-master: "master" +%%% End: diff --git a/TST_sti2d/DM/2102_DM1/11_2102_DM1.tex b/TST_sti2d/DM/2102_DM1/11_2102_DM1.tex new file mode 100644 index 0000000..793088d --- /dev/null +++ b/TST_sti2d/DM/2102_DM1/11_2102_DM1.tex @@ -0,0 +1,121 @@ +\documentclass[a4paper,10pt]{article} +\usepackage{myXsim} +\usepackage{tasks} + +% Title Page +\title{DM1 \hfill HUMBERT Rayan} +\tribe{TST sti2d} +\date{\hfillÀ render pour le jeudi 25 février} + +\xsimsetup{ + solution/print = false +} + +\begin{document} +\maketitle + +\begin{exercise}[subtitle={Complexes}] + \begin{enumerate} + \item Mettre le nombre complexe suivant sous forme algébrique $z_1 = \dfrac{5 + 10 i}{-5 + 2 i} $ + \item Mettre le complexe suivante sous forme exponentielle $z_2 = - 5 \sqrt{3} - 5 i$ + \item Mettre le complexe suivante sous forme exponentielle $z_3 = 9 - 9 \sqrt{3} i$ + \item Calculer le produit $z_4=z_2\times z_3$ donner le résultat sous forme exponentielle puis algébrique. + \item Calculer le quotient $z_5=\frac{z_2}{z_3}$ donner le résultat sous forme exponentielle puis algébrique. + \end{enumerate} +\end{exercise} + +\begin{solution} + \begin{enumerate} + \item $z_1 = - \frac{5}{29} - \frac{60 i}{29}$ + \item $z_3 = 10 e^{- \frac{5 i \pi}{6}}$ + \item $z_4 = 180 e^{- \frac{7 i \pi}{6}} = - 90 \sqrt{3} + 90 i = -156.0 + 90.0 i$ + \item $z_5 = \frac{5}{9} e^{- \frac{i \pi}{2}} = - \frac{5 i}{9} = - 0.556 i$ + \end{enumerate} +\end{solution} + +\begin{exercise}[subtitle={Bassin}] + Le tour d'un bassin au niveau du sol présente deux axes de symétrie : l’axe des abscisses et la droite d’équation $x=4$. Il est obtenu par symétrie de la courbe $\mathcal{C_f}$ sur $\intFF{0}{4}$ où $f$ est la fonction définie par + + \[ + f(x) = \left(- x^{2} + 0.3 x - 9.7\right) e^{- x} + 9.7 + \] + On admet que sur $\intFF{0}{4}$ la fonction $f$ est positive. + \begin{enumerate} + \item Sur un repère, tracer l'allure de la courbe $\mathcal{C}_f$, les axes de symétries puis compléter pour dessiner la forme du bassin. + \item Montrer que la fonction $f$ admet comme primitive sur $\R$ la fonction $F$ définie par + \[ + F(x) = 9.7 x + \left( x^{2} + 1.7 x + 11.4\right) e^{- x} + \] + \item Calculer la quantité $\ds \int_0^4 f(x) \; dx$, vous donnerez le résultat sous forme exacte. Interpréter le résultat et reportez cette quantité sur le graphique. + \item On considère que l'échelle de votre graphique est de 1unité pour 15m. Calculer l'aire du bassin. Vous donnerez un résultat arrondi au $m^2$ près. + \end{enumerate} +\end{exercise} + +\begin{solution} + \begin{enumerate} + \item + \begin{tikzpicture}[baseline=(a.north), xscale=1, yscale=0.5] + \tkzInit[xmin=0,xmax=5,xstep=1, + ymin=0,ymax=10,ystep=1] + \tkzGrid + \tkzAxeXY + \tkzFct[domain=0:10,color=red,very thick]% + { (-x**2 + 0.3*x - 9.7)*exp(-x) + 9.7 }; + \end{tikzpicture} + \item Il faut dériver $F(x)$ et vérifier que $F'(x) = f(x)$. + \item $\ds \int_0^4 f(x) \; dx = F(4) - F(0) = \frac{34.2}{e^{4}} + 27.4$ + \item La quantité calculée à la question précédente se retrouve 4fois pour former le bassin. Il faut ensuite prendre en compte l'échelle, comme 1unité de longueur correspond à 15m, une unité d'air correspond à $15\times15 = 225m^2$. Ainsi l'aire du bassin est égale à + \[ + (\frac{34.2}{e^{4}} + 27.4)\times 4 \times 15^2 = 25224.00000 + \] + + \end{enumerate} +\end{solution} + +\begin{exercise}[subtitle={Bassin}] + Le clinker est un constituant du ciment qui résulte de la cuisson d'un mélange composé de calcaire et d'argile. La fabrication du clinker nécessite des fours à très haute température qui libèrent dans l'air une grande quantité de dioxyde de carbone (CO$_2$). + + Dans une cimenterie, la fabrication du clinker s'effectue de 7 h 30 à 20 h, dans une pièce de volume \np{600000}~dm$^3$. + + À 20 h, après une journée de travail, le taux volumique de CO$_2$ dans la pièce est de 0.6\,\%. + \begin{enumerate} + \item Justifier que le volume de CO$_2$ présent dans cette pièce à 20 h est de \np{3600}~dm$^3$ . + \item On modélise le volume de CO$_2$ présent dans la pièce par une fonction du temps $t$ écoulé après 20h (exprimé en minutes) qui pour formule $V(t) = V_0e^{-0.06t} + 360$ + \begin{enumerate} + \item Démontrer que $V_0$ est égale à \np{3240}. + \item Quel sera, au dm$^3$ près, le volume de CO$_2$ dans cette pièce à 24 h ? + \item Démontrer que $V'(t) = - 194.4 e^{- 0.06 t}$. + \item Étudier le signe de $V'(t)$ puis en déduire le sens de variation de $V(t)$. + \item Que peut-on dire du volume de CO$_2$ quand $t$ devient grand? + \end{enumerate} + \end{enumerate} +\end{exercise} + +\begin{solution} + \begin{enumerate} + \item Volume à 20h: $600000\times 0.006 = 3600$ + \item + \begin{enumerate} + \item $t=0$ correspond à 20h. + + Donc $V(0) = 3600 = V_0e^{-0.06\times 0} + 360 = V_0 + 360$ + + Donc $V_0 = 3600 - 360 = 3240$ + \item Il faut calculer $V(t)$ pour $t = 4$ donc + \[ + V(4) = 2908.67 + \] + \item Pas de correction pour cette question. + \item Pas de correction pour cette question. + \item Pas de correction pour cette question. + \end{enumerate} + \end{enumerate} +\end{solution} + + +\end{document} + +%%% Local Variables: +%%% mode: latex +%%% TeX-master: "master" +%%% End: diff --git a/TST_sti2d/DM/2102_DM1/12_2102_DM1.tex b/TST_sti2d/DM/2102_DM1/12_2102_DM1.tex new file mode 100644 index 0000000..59fb0f8 --- /dev/null +++ b/TST_sti2d/DM/2102_DM1/12_2102_DM1.tex @@ -0,0 +1,121 @@ +\documentclass[a4paper,10pt]{article} +\usepackage{myXsim} +\usepackage{tasks} + +% Title Page +\title{DM1 \hfill KILINC Suleyman} +\tribe{TST sti2d} +\date{\hfillÀ render pour le jeudi 25 février} + +\xsimsetup{ + solution/print = false +} + +\begin{document} +\maketitle + +\begin{exercise}[subtitle={Complexes}] + \begin{enumerate} + \item Mettre le nombre complexe suivant sous forme algébrique $z_1 = \dfrac{2 + 7 i}{-8 + 6 i} $ + \item Mettre le complexe suivante sous forme exponentielle $z_2 = \sqrt{2} + \sqrt{2} i$ + \item Mettre le complexe suivante sous forme exponentielle $z_3 = 8 \sqrt{3} - 8 i$ + \item Calculer le produit $z_4=z_2\times z_3$ donner le résultat sous forme exponentielle puis algébrique. + \item Calculer le quotient $z_5=\frac{z_2}{z_3}$ donner le résultat sous forme exponentielle puis algébrique. + \end{enumerate} +\end{exercise} + +\begin{solution} + \begin{enumerate} + \item $z_1 = \frac{13}{50} - \frac{17 i}{25}$ + \item $z_3 = 2 e^{\frac{i \pi}{4}}$ + \item $z_4 = 32 e^{\frac{i \pi}{12}} = 8 \sqrt{2} + 8 \sqrt{6} + i \left(- 8 \sqrt{2} + 8 \sqrt{6}\right) = 30.9 + 8.28 i$ + \item $z_5 = \frac{1}{8} e^{\frac{5 i \pi}{12}} = - \frac{\sqrt{2}}{32} + \frac{\sqrt{6}}{32} + i \left(\frac{\sqrt{2}}{32} + \frac{\sqrt{6}}{32}\right) = 0.0323 + 0.121 i$ + \end{enumerate} +\end{solution} + +\begin{exercise}[subtitle={Bassin}] + Le tour d'un bassin au niveau du sol présente deux axes de symétrie : l’axe des abscisses et la droite d’équation $x=4$. Il est obtenu par symétrie de la courbe $\mathcal{C_f}$ sur $\intFF{0}{4}$ où $f$ est la fonction définie par + + \[ + f(x) = \left(- x^{2} + 5.8 x - 4.6\right) e^{- x} + 4.6 + \] + On admet que sur $\intFF{0}{4}$ la fonction $f$ est positive. + \begin{enumerate} + \item Sur un repère, tracer l'allure de la courbe $\mathcal{C}_f$, les axes de symétries puis compléter pour dessiner la forme du bassin. + \item Montrer que la fonction $f$ admet comme primitive sur $\R$ la fonction $F$ définie par + \[ + F(x) = 4.6 x + \left( x^{2} - 3.8 x + 0.8\right) e^{- x} + \] + \item Calculer la quantité $\ds \int_0^4 f(x) \; dx$, vous donnerez le résultat sous forme exacte. Interpréter le résultat et reportez cette quantité sur le graphique. + \item On considère que l'échelle de votre graphique est de 1unité pour 15m. Calculer l'aire du bassin. Vous donnerez un résultat arrondi au $m^2$ près. + \end{enumerate} +\end{exercise} + +\begin{solution} + \begin{enumerate} + \item + \begin{tikzpicture}[baseline=(a.north), xscale=1, yscale=0.5] + \tkzInit[xmin=0,xmax=5,xstep=1, + ymin=0,ymax=10,ystep=1] + \tkzGrid + \tkzAxeXY + \tkzFct[domain=0:10,color=red,very thick]% + { (-x**2 + 5.8*x - 4.6)*exp(-x) + 4.6 }; + \end{tikzpicture} + \item Il faut dériver $F(x)$ et vérifier que $F'(x) = f(x)$. + \item $\ds \int_0^4 f(x) \; dx = F(4) - F(0) = \frac{1.6}{e^{4}} + 17.6$ + \item La quantité calculée à la question précédente se retrouve 4fois pour former le bassin. Il faut ensuite prendre en compte l'échelle, comme 1unité de longueur correspond à 15m, une unité d'air correspond à $15\times15 = 225m^2$. Ainsi l'aire du bassin est égale à + \[ + (\frac{1.6}{e^{4}} + 17.6)\times 4 \times 15^2 = 15866.00000 + \] + + \end{enumerate} +\end{solution} + +\begin{exercise}[subtitle={Bassin}] + Le clinker est un constituant du ciment qui résulte de la cuisson d'un mélange composé de calcaire et d'argile. La fabrication du clinker nécessite des fours à très haute température qui libèrent dans l'air une grande quantité de dioxyde de carbone (CO$_2$). + + Dans une cimenterie, la fabrication du clinker s'effectue de 7 h 30 à 20 h, dans une pièce de volume \np{700000}~dm$^3$. + + À 20 h, après une journée de travail, le taux volumique de CO$_2$ dans la pièce est de 1.0\,\%. + \begin{enumerate} + \item Justifier que le volume de CO$_2$ présent dans cette pièce à 20 h est de \np{7000}~dm$^3$ . + \item On modélise le volume de CO$_2$ présent dans la pièce par une fonction du temps $t$ écoulé après 20h (exprimé en minutes) qui pour formule $V(t) = V_0e^{-0.07t} + 520$ + \begin{enumerate} + \item Démontrer que $V_0$ est égale à \np{6480}. + \item Quel sera, au dm$^3$ près, le volume de CO$_2$ dans cette pièce à 24 h ? + \item Démontrer que $V'(t) = - 453.6 e^{- 0.07 t}$. + \item Étudier le signe de $V'(t)$ puis en déduire le sens de variation de $V(t)$. + \item Que peut-on dire du volume de CO$_2$ quand $t$ devient grand? + \end{enumerate} + \end{enumerate} +\end{exercise} + +\begin{solution} + \begin{enumerate} + \item Volume à 20h: $700000\times 0.01 = 7000$ + \item + \begin{enumerate} + \item $t=0$ correspond à 20h. + + Donc $V(0) = 7000 = V_0e^{-0.07\times 0} + 520 = V_0 + 520$ + + Donc $V_0 = 7000 - 520 = 6480$ + \item Il faut calculer $V(t)$ pour $t = 4$ donc + \[ + V(4) = 5417.48 + \] + \item Pas de correction pour cette question. + \item Pas de correction pour cette question. + \item Pas de correction pour cette question. + \end{enumerate} + \end{enumerate} +\end{solution} + + +\end{document} + +%%% Local Variables: +%%% mode: latex +%%% TeX-master: "master" +%%% End: diff --git a/TST_sti2d/DM/2102_DM1/13_2102_DM1.tex b/TST_sti2d/DM/2102_DM1/13_2102_DM1.tex new file mode 100644 index 0000000..11ec645 --- /dev/null +++ b/TST_sti2d/DM/2102_DM1/13_2102_DM1.tex @@ -0,0 +1,121 @@ +\documentclass[a4paper,10pt]{article} +\usepackage{myXsim} +\usepackage{tasks} + +% Title Page +\title{DM1 \hfill M'BAREK HASNAOUI Bilal} +\tribe{TST sti2d} +\date{\hfillÀ render pour le jeudi 25 février} + +\xsimsetup{ + solution/print = false +} + +\begin{document} +\maketitle + +\begin{exercise}[subtitle={Complexes}] + \begin{enumerate} + \item Mettre le nombre complexe suivant sous forme algébrique $z_1 = \dfrac{4 + 7 i}{-10 + 7 i} $ + \item Mettre le complexe suivante sous forme exponentielle $z_2 = 6 - 6 \sqrt{3} i$ + \item Mettre le complexe suivante sous forme exponentielle $z_3 = 6 \sqrt{2} + 6 \sqrt{2} i$ + \item Calculer le produit $z_4=z_2\times z_3$ donner le résultat sous forme exponentielle puis algébrique. + \item Calculer le quotient $z_5=\frac{z_2}{z_3}$ donner le résultat sous forme exponentielle puis algébrique. + \end{enumerate} +\end{exercise} + +\begin{solution} + \begin{enumerate} + \item $z_1 = \frac{9}{149} - \frac{98 i}{149}$ + \item $z_3 = 12 e^{- \frac{i \pi}{3}}$ + \item $z_4 = 144 e^{- \frac{i \pi}{12}} = 36 \sqrt{2} + 36 \sqrt{6} + i \left(- 36 \sqrt{6} + 36 \sqrt{2}\right) = 139.0 - 37.3 i$ + \item $z_5 = 1 e^{- \frac{7 i \pi}{12}} = - \frac{\sqrt{6}}{4} + \frac{\sqrt{2}}{4} + i \left(- \frac{\sqrt{6}}{4} - \frac{\sqrt{2}}{4}\right) = -0.259 - 0.966 i$ + \end{enumerate} +\end{solution} + +\begin{exercise}[subtitle={Bassin}] + Le tour d'un bassin au niveau du sol présente deux axes de symétrie : l’axe des abscisses et la droite d’équation $x=4$. Il est obtenu par symétrie de la courbe $\mathcal{C_f}$ sur $\intFF{0}{4}$ où $f$ est la fonction définie par + + \[ + f(x) = \left(- x^{2} + 2.2 x - 9.7\right) e^{- x} + 9.7 + \] + On admet que sur $\intFF{0}{4}$ la fonction $f$ est positive. + \begin{enumerate} + \item Sur un repère, tracer l'allure de la courbe $\mathcal{C}_f$, les axes de symétries puis compléter pour dessiner la forme du bassin. + \item Montrer que la fonction $f$ admet comme primitive sur $\R$ la fonction $F$ définie par + \[ + F(x) = 9.7 x + \left( x^{2} - 0.2 x + 9.5\right) e^{- x} + \] + \item Calculer la quantité $\ds \int_0^4 f(x) \; dx$, vous donnerez le résultat sous forme exacte. Interpréter le résultat et reportez cette quantité sur le graphique. + \item On considère que l'échelle de votre graphique est de 1unité pour 15m. Calculer l'aire du bassin. Vous donnerez un résultat arrondi au $m^2$ près. + \end{enumerate} +\end{exercise} + +\begin{solution} + \begin{enumerate} + \item + \begin{tikzpicture}[baseline=(a.north), xscale=1, yscale=0.5] + \tkzInit[xmin=0,xmax=5,xstep=1, + ymin=0,ymax=10,ystep=1] + \tkzGrid + \tkzAxeXY + \tkzFct[domain=0:10,color=red,very thick]% + { (-x**2 + 2.2*x - 9.7)*exp(-x) + 9.7 }; + \end{tikzpicture} + \item Il faut dériver $F(x)$ et vérifier que $F'(x) = f(x)$. + \item $\ds \int_0^4 f(x) \; dx = F(4) - F(0) = \frac{24.7}{e^{4}} + 29.3$ + \item La quantité calculée à la question précédente se retrouve 4fois pour former le bassin. Il faut ensuite prendre en compte l'échelle, comme 1unité de longueur correspond à 15m, une unité d'air correspond à $15\times15 = 225m^2$. Ainsi l'aire du bassin est égale à + \[ + (\frac{24.7}{e^{4}} + 29.3)\times 4 \times 15^2 = 26777.00000 + \] + + \end{enumerate} +\end{solution} + +\begin{exercise}[subtitle={Bassin}] + Le clinker est un constituant du ciment qui résulte de la cuisson d'un mélange composé de calcaire et d'argile. La fabrication du clinker nécessite des fours à très haute température qui libèrent dans l'air une grande quantité de dioxyde de carbone (CO$_2$). + + Dans une cimenterie, la fabrication du clinker s'effectue de 7 h 30 à 20 h, dans une pièce de volume \np{300000}~dm$^3$. + + À 20 h, après une journée de travail, le taux volumique de CO$_2$ dans la pièce est de 0.6\,\%. + \begin{enumerate} + \item Justifier que le volume de CO$_2$ présent dans cette pièce à 20 h est de \np{1800}~dm$^3$ . + \item On modélise le volume de CO$_2$ présent dans la pièce par une fonction du temps $t$ écoulé après 20h (exprimé en minutes) qui pour formule $V(t) = V_0e^{-0.1t} + 430$ + \begin{enumerate} + \item Démontrer que $V_0$ est égale à \np{1370}. + \item Quel sera, au dm$^3$ près, le volume de CO$_2$ dans cette pièce à 22 h ? + \item Démontrer que $V'(t) = - 137.0 e^{- 0.1 t}$. + \item Étudier le signe de $V'(t)$ puis en déduire le sens de variation de $V(t)$. + \item Que peut-on dire du volume de CO$_2$ quand $t$ devient grand? + \end{enumerate} + \end{enumerate} +\end{exercise} + +\begin{solution} + \begin{enumerate} + \item Volume à 20h: $300000\times 0.006 = 1800$ + \item + \begin{enumerate} + \item $t=0$ correspond à 20h. + + Donc $V(0) = 1800 = V_0e^{-0.1\times 0} + 430 = V_0 + 430$ + + Donc $V_0 = 1800 - 430 = 1370$ + \item Il faut calculer $V(t)$ pour $t = 2$ donc + \[ + V(2) = 1551.66 + \] + \item Pas de correction pour cette question. + \item Pas de correction pour cette question. + \item Pas de correction pour cette question. + \end{enumerate} + \end{enumerate} +\end{solution} + + +\end{document} + +%%% Local Variables: +%%% mode: latex +%%% TeX-master: "master" +%%% End: diff --git a/TST_sti2d/DM/2102_DM1/14_2102_DM1.tex b/TST_sti2d/DM/2102_DM1/14_2102_DM1.tex new file mode 100644 index 0000000..aa7f5fb --- /dev/null +++ b/TST_sti2d/DM/2102_DM1/14_2102_DM1.tex @@ -0,0 +1,121 @@ +\documentclass[a4paper,10pt]{article} +\usepackage{myXsim} +\usepackage{tasks} + +% Title Page +\title{DM1 \hfill MERCIER Almandin} +\tribe{TST sti2d} +\date{\hfillÀ render pour le jeudi 25 février} + +\xsimsetup{ + solution/print = false +} + +\begin{document} +\maketitle + +\begin{exercise}[subtitle={Complexes}] + \begin{enumerate} + \item Mettre le nombre complexe suivant sous forme algébrique $z_1 = \dfrac{2 + 4 i}{-6 + 6 i} $ + \item Mettre le complexe suivante sous forme exponentielle $z_2 = 4 \sqrt{3} - 4 i$ + \item Mettre le complexe suivante sous forme exponentielle $z_3 = - 10 \sqrt{2} + 10 \sqrt{2} i$ + \item Calculer le produit $z_4=z_2\times z_3$ donner le résultat sous forme exponentielle puis algébrique. + \item Calculer le quotient $z_5=\frac{z_2}{z_3}$ donner le résultat sous forme exponentielle puis algébrique. + \end{enumerate} +\end{exercise} + +\begin{solution} + \begin{enumerate} + \item $z_1 = \frac{1}{6} - \frac{i}{2}$ + \item $z_3 = 8 e^{- \frac{i \pi}{6}}$ + \item $z_4 = 160 e^{\frac{7 i \pi}{12}} = - 40 \sqrt{6} + 40 \sqrt{2} + i \left(40 \sqrt{2} + 40 \sqrt{6}\right) = -41.4 + 155.0 i$ + \item $z_5 = \frac{2}{5} e^{- \frac{11 i \pi}{12}} = - \frac{\sqrt{6}}{10} - \frac{\sqrt{2}}{10} + i \left(- \frac{\sqrt{6}}{10} + \frac{\sqrt{2}}{10}\right) = -0.386 - 0.104 i$ + \end{enumerate} +\end{solution} + +\begin{exercise}[subtitle={Bassin}] + Le tour d'un bassin au niveau du sol présente deux axes de symétrie : l’axe des abscisses et la droite d’équation $x=4$. Il est obtenu par symétrie de la courbe $\mathcal{C_f}$ sur $\intFF{0}{4}$ où $f$ est la fonction définie par + + \[ + f(x) = \left(- x^{2} + 1.4 x - 4.9\right) e^{- x} + 4.9 + \] + On admet que sur $\intFF{0}{4}$ la fonction $f$ est positive. + \begin{enumerate} + \item Sur un repère, tracer l'allure de la courbe $\mathcal{C}_f$, les axes de symétries puis compléter pour dessiner la forme du bassin. + \item Montrer que la fonction $f$ admet comme primitive sur $\R$ la fonction $F$ définie par + \[ + F(x) = 4.9 x + \left( x^{2} + 0.6 x + 5.5\right) e^{- x} + \] + \item Calculer la quantité $\ds \int_0^4 f(x) \; dx$, vous donnerez le résultat sous forme exacte. Interpréter le résultat et reportez cette quantité sur le graphique. + \item On considère que l'échelle de votre graphique est de 1unité pour 15m. Calculer l'aire du bassin. Vous donnerez un résultat arrondi au $m^2$ près. + \end{enumerate} +\end{exercise} + +\begin{solution} + \begin{enumerate} + \item + \begin{tikzpicture}[baseline=(a.north), xscale=1, yscale=0.5] + \tkzInit[xmin=0,xmax=5,xstep=1, + ymin=0,ymax=10,ystep=1] + \tkzGrid + \tkzAxeXY + \tkzFct[domain=0:10,color=red,very thick]% + { (-x**2 + 1.4*x - 4.9)*exp(-x) + 4.9 }; + \end{tikzpicture} + \item Il faut dériver $F(x)$ et vérifier que $F'(x) = f(x)$. + \item $\ds \int_0^4 f(x) \; dx = F(4) - F(0) = \frac{23.9}{e^{4}} + 14.1$ + \item La quantité calculée à la question précédente se retrouve 4fois pour former le bassin. Il faut ensuite prendre en compte l'échelle, comme 1unité de longueur correspond à 15m, une unité d'air correspond à $15\times15 = 225m^2$. Ainsi l'aire du bassin est égale à + \[ + (\frac{23.9}{e^{4}} + 14.1)\times 4 \times 15^2 = 13084.00000 + \] + + \end{enumerate} +\end{solution} + +\begin{exercise}[subtitle={Bassin}] + Le clinker est un constituant du ciment qui résulte de la cuisson d'un mélange composé de calcaire et d'argile. La fabrication du clinker nécessite des fours à très haute température qui libèrent dans l'air une grande quantité de dioxyde de carbone (CO$_2$). + + Dans une cimenterie, la fabrication du clinker s'effectue de 7 h 30 à 20 h, dans une pièce de volume \np{400000}~dm$^3$. + + À 20 h, après une journée de travail, le taux volumique de CO$_2$ dans la pièce est de 0.7\,\%. + \begin{enumerate} + \item Justifier que le volume de CO$_2$ présent dans cette pièce à 20 h est de \np{2800}~dm$^3$ . + \item On modélise le volume de CO$_2$ présent dans la pièce par une fonction du temps $t$ écoulé après 20h (exprimé en minutes) qui pour formule $V(t) = V_0e^{-0.07t} + 360$ + \begin{enumerate} + \item Démontrer que $V_0$ est égale à \np{2440}. + \item Quel sera, au dm$^3$ près, le volume de CO$_2$ dans cette pièce à 23 h ? + \item Démontrer que $V'(t) = - 170.8 e^{- 0.07 t}$. + \item Étudier le signe de $V'(t)$ puis en déduire le sens de variation de $V(t)$. + \item Que peut-on dire du volume de CO$_2$ quand $t$ devient grand? + \end{enumerate} + \end{enumerate} +\end{exercise} + +\begin{solution} + \begin{enumerate} + \item Volume à 20h: $400000\times 0.006999999999999999 = 2800$ + \item + \begin{enumerate} + \item $t=0$ correspond à 20h. + + Donc $V(0) = 2800 = V_0e^{-0.07\times 0} + 360 = V_0 + 360$ + + Donc $V_0 = 2800 - 360 = 2440$ + \item Il faut calculer $V(t)$ pour $t = 3$ donc + \[ + V(3) = 2337.83 + \] + \item Pas de correction pour cette question. + \item Pas de correction pour cette question. + \item Pas de correction pour cette question. + \end{enumerate} + \end{enumerate} +\end{solution} + + +\end{document} + +%%% Local Variables: +%%% mode: latex +%%% TeX-master: "master" +%%% End: diff --git a/TST_sti2d/DM/2102_DM1/15_2102_DM1.tex b/TST_sti2d/DM/2102_DM1/15_2102_DM1.tex new file mode 100644 index 0000000..edcc5fb --- /dev/null +++ b/TST_sti2d/DM/2102_DM1/15_2102_DM1.tex @@ -0,0 +1,121 @@ +\documentclass[a4paper,10pt]{article} +\usepackage{myXsim} +\usepackage{tasks} + +% Title Page +\title{DM1 \hfill MOUFAQ Amine} +\tribe{TST sti2d} +\date{\hfillÀ render pour le jeudi 25 février} + +\xsimsetup{ + solution/print = false +} + +\begin{document} +\maketitle + +\begin{exercise}[subtitle={Complexes}] + \begin{enumerate} + \item Mettre le nombre complexe suivant sous forme algébrique $z_1 = \dfrac{6 + 9 i}{-9 + 10 i} $ + \item Mettre le complexe suivante sous forme exponentielle $z_2 = 10 \sqrt{3} + 10 i$ + \item Mettre le complexe suivante sous forme exponentielle $z_3 = 9 \sqrt{2} + 9 \sqrt{2} i$ + \item Calculer le produit $z_4=z_2\times z_3$ donner le résultat sous forme exponentielle puis algébrique. + \item Calculer le quotient $z_5=\frac{z_2}{z_3}$ donner le résultat sous forme exponentielle puis algébrique. + \end{enumerate} +\end{exercise} + +\begin{solution} + \begin{enumerate} + \item $z_1 = \frac{36}{181} - \frac{141 i}{181}$ + \item $z_3 = 20 e^{\frac{i \pi}{6}}$ + \item $z_4 = 360 e^{\frac{5 i \pi}{12}} = - 90 \sqrt{2} + 90 \sqrt{6} + i \left(90 \sqrt{2} + 90 \sqrt{6}\right) = 93.2 + 348.0 i$ + \item $z_5 = \frac{10}{9} e^{- \frac{i \pi}{12}} = \frac{5 \sqrt{2}}{18} + \frac{5 \sqrt{6}}{18} + i \left(- \frac{5 \sqrt{6}}{18} + \frac{5 \sqrt{2}}{18}\right) = 1.07 - 0.288 i$ + \end{enumerate} +\end{solution} + +\begin{exercise}[subtitle={Bassin}] + Le tour d'un bassin au niveau du sol présente deux axes de symétrie : l’axe des abscisses et la droite d’équation $x=4$. Il est obtenu par symétrie de la courbe $\mathcal{C_f}$ sur $\intFF{0}{4}$ où $f$ est la fonction définie par + + \[ + f(x) = \left(- x^{2} + 5.5 x - 9.8\right) e^{- x} + 9.8 + \] + On admet que sur $\intFF{0}{4}$ la fonction $f$ est positive. + \begin{enumerate} + \item Sur un repère, tracer l'allure de la courbe $\mathcal{C}_f$, les axes de symétries puis compléter pour dessiner la forme du bassin. + \item Montrer que la fonction $f$ admet comme primitive sur $\R$ la fonction $F$ définie par + \[ + F(x) = 9.8 x + \left( x^{2} - 3.5 x + 6.3\right) e^{- x} + \] + \item Calculer la quantité $\ds \int_0^4 f(x) \; dx$, vous donnerez le résultat sous forme exacte. Interpréter le résultat et reportez cette quantité sur le graphique. + \item On considère que l'échelle de votre graphique est de 1unité pour 15m. Calculer l'aire du bassin. Vous donnerez un résultat arrondi au $m^2$ près. + \end{enumerate} +\end{exercise} + +\begin{solution} + \begin{enumerate} + \item + \begin{tikzpicture}[baseline=(a.north), xscale=1, yscale=0.5] + \tkzInit[xmin=0,xmax=5,xstep=1, + ymin=0,ymax=10,ystep=1] + \tkzGrid + \tkzAxeXY + \tkzFct[domain=0:10,color=red,very thick]% + { (-x**2 + 5.5*x - 9.8)*exp(-x) + 9.8 }; + \end{tikzpicture} + \item Il faut dériver $F(x)$ et vérifier que $F'(x) = f(x)$. + \item $\ds \int_0^4 f(x) \; dx = F(4) - F(0) = \frac{8.3}{e^{4}} + 32.9$ + \item La quantité calculée à la question précédente se retrouve 4fois pour former le bassin. Il faut ensuite prendre en compte l'échelle, comme 1unité de longueur correspond à 15m, une unité d'air correspond à $15\times15 = 225m^2$. Ainsi l'aire du bassin est égale à + \[ + (\frac{8.3}{e^{4}} + 32.9)\times 4 \times 15^2 = 29747.00000 + \] + + \end{enumerate} +\end{solution} + +\begin{exercise}[subtitle={Bassin}] + Le clinker est un constituant du ciment qui résulte de la cuisson d'un mélange composé de calcaire et d'argile. La fabrication du clinker nécessite des fours à très haute température qui libèrent dans l'air une grande quantité de dioxyde de carbone (CO$_2$). + + Dans une cimenterie, la fabrication du clinker s'effectue de 7 h 30 à 20 h, dans une pièce de volume \np{600000}~dm$^3$. + + À 20 h, après une journée de travail, le taux volumique de CO$_2$ dans la pièce est de 0.9\,\%. + \begin{enumerate} + \item Justifier que le volume de CO$_2$ présent dans cette pièce à 20 h est de \np{5400}~dm$^3$ . + \item On modélise le volume de CO$_2$ présent dans la pièce par une fonction du temps $t$ écoulé après 20h (exprimé en minutes) qui pour formule $V(t) = V_0e^{-0.06t} + 340$ + \begin{enumerate} + \item Démontrer que $V_0$ est égale à \np{5060}. + \item Quel sera, au dm$^3$ près, le volume de CO$_2$ dans cette pièce à 24 h ? + \item Démontrer que $V'(t) = - 303.6 e^{- 0.06 t}$. + \item Étudier le signe de $V'(t)$ puis en déduire le sens de variation de $V(t)$. + \item Que peut-on dire du volume de CO$_2$ quand $t$ devient grand? + \end{enumerate} + \end{enumerate} +\end{exercise} + +\begin{solution} + \begin{enumerate} + \item Volume à 20h: $600000\times 0.009000000000000001 = 5400$ + \item + \begin{enumerate} + \item $t=0$ correspond à 20h. + + Donc $V(0) = 5400 = V_0e^{-0.06\times 0} + 340 = V_0 + 340$ + + Donc $V_0 = 5400 - 340 = 5060$ + \item Il faut calculer $V(t)$ pour $t = 4$ donc + \[ + V(4) = 4320.34 + \] + \item Pas de correction pour cette question. + \item Pas de correction pour cette question. + \item Pas de correction pour cette question. + \end{enumerate} + \end{enumerate} +\end{solution} + + +\end{document} + +%%% Local Variables: +%%% mode: latex +%%% TeX-master: "master" +%%% End: diff --git a/TST_sti2d/DM/2102_DM1/16_2102_DM1.tex b/TST_sti2d/DM/2102_DM1/16_2102_DM1.tex new file mode 100644 index 0000000..05e7ffe --- /dev/null +++ b/TST_sti2d/DM/2102_DM1/16_2102_DM1.tex @@ -0,0 +1,121 @@ +\documentclass[a4paper,10pt]{article} +\usepackage{myXsim} +\usepackage{tasks} + +% Title Page +\title{DM1 \hfill NARDINI Kakary} +\tribe{TST sti2d} +\date{\hfillÀ render pour le jeudi 25 février} + +\xsimsetup{ + solution/print = false +} + +\begin{document} +\maketitle + +\begin{exercise}[subtitle={Complexes}] + \begin{enumerate} + \item Mettre le nombre complexe suivant sous forme algébrique $z_1 = \dfrac{7 + 5 i}{-4 + 5 i} $ + \item Mettre le complexe suivante sous forme exponentielle $z_2 = - 8 \sqrt{2} + 8 \sqrt{2} i$ + \item Mettre le complexe suivante sous forme exponentielle $z_3 = \sqrt{3} + i$ + \item Calculer le produit $z_4=z_2\times z_3$ donner le résultat sous forme exponentielle puis algébrique. + \item Calculer le quotient $z_5=\frac{z_2}{z_3}$ donner le résultat sous forme exponentielle puis algébrique. + \end{enumerate} +\end{exercise} + +\begin{solution} + \begin{enumerate} + \item $z_1 = - \frac{3}{41} - \frac{55 i}{41}$ + \item $z_3 = 16 e^{\frac{3 i \pi}{4}}$ + \item $z_4 = 32 e^{\frac{11 i \pi}{12}} = - 8 \sqrt{6} - 8 \sqrt{2} + i \left(- 8 \sqrt{2} + 8 \sqrt{6}\right) = -30.9 + 8.28 i$ + \item $z_5 = 8 e^{\frac{7 i \pi}{12}} = - 2 \sqrt{6} + 2 \sqrt{2} + i \left(2 \sqrt{2} + 2 \sqrt{6}\right) = -2.07 + 7.73 i$ + \end{enumerate} +\end{solution} + +\begin{exercise}[subtitle={Bassin}] + Le tour d'un bassin au niveau du sol présente deux axes de symétrie : l’axe des abscisses et la droite d’équation $x=4$. Il est obtenu par symétrie de la courbe $\mathcal{C_f}$ sur $\intFF{0}{4}$ où $f$ est la fonction définie par + + \[ + f(x) = \left(- x^{2} + 2.3 x - 1.7\right) e^{- x} + 1.7 + \] + On admet que sur $\intFF{0}{4}$ la fonction $f$ est positive. + \begin{enumerate} + \item Sur un repère, tracer l'allure de la courbe $\mathcal{C}_f$, les axes de symétries puis compléter pour dessiner la forme du bassin. + \item Montrer que la fonction $f$ admet comme primitive sur $\R$ la fonction $F$ définie par + \[ + F(x) = 1.7 x + \left( x^{2} - 0.3 x + 1.4\right) e^{- x} + \] + \item Calculer la quantité $\ds \int_0^4 f(x) \; dx$, vous donnerez le résultat sous forme exacte. Interpréter le résultat et reportez cette quantité sur le graphique. + \item On considère que l'échelle de votre graphique est de 1unité pour 15m. Calculer l'aire du bassin. Vous donnerez un résultat arrondi au $m^2$ près. + \end{enumerate} +\end{exercise} + +\begin{solution} + \begin{enumerate} + \item + \begin{tikzpicture}[baseline=(a.north), xscale=1, yscale=0.5] + \tkzInit[xmin=0,xmax=5,xstep=1, + ymin=0,ymax=10,ystep=1] + \tkzGrid + \tkzAxeXY + \tkzFct[domain=0:10,color=red,very thick]% + { (-x**2 + 2.3*x - 1.7)*exp(-x) + 1.7 }; + \end{tikzpicture} + \item Il faut dériver $F(x)$ et vérifier que $F'(x) = f(x)$. + \item $\ds \int_0^4 f(x) \; dx = F(4) - F(0) = \frac{16.2}{e^{4}} + 5.4$ + \item La quantité calculée à la question précédente se retrouve 4fois pour former le bassin. Il faut ensuite prendre en compte l'échelle, comme 1unité de longueur correspond à 15m, une unité d'air correspond à $15\times15 = 225m^2$. Ainsi l'aire du bassin est égale à + \[ + (\frac{16.2}{e^{4}} + 5.4)\times 4 \times 15^2 = 5127.000000 + \] + + \end{enumerate} +\end{solution} + +\begin{exercise}[subtitle={Bassin}] + Le clinker est un constituant du ciment qui résulte de la cuisson d'un mélange composé de calcaire et d'argile. La fabrication du clinker nécessite des fours à très haute température qui libèrent dans l'air une grande quantité de dioxyde de carbone (CO$_2$). + + Dans une cimenterie, la fabrication du clinker s'effectue de 7 h 30 à 20 h, dans une pièce de volume \np{600000}~dm$^3$. + + À 20 h, après une journée de travail, le taux volumique de CO$_2$ dans la pièce est de 0.9\,\%. + \begin{enumerate} + \item Justifier que le volume de CO$_2$ présent dans cette pièce à 20 h est de \np{5400}~dm$^3$ . + \item On modélise le volume de CO$_2$ présent dans la pièce par une fonction du temps $t$ écoulé après 20h (exprimé en minutes) qui pour formule $V(t) = V_0e^{-0.09t} + 360$ + \begin{enumerate} + \item Démontrer que $V_0$ est égale à \np{5040}. + \item Quel sera, au dm$^3$ près, le volume de CO$_2$ dans cette pièce à 22 h ? + \item Démontrer que $V'(t) = - 453.6 e^{- 0.09 t}$. + \item Étudier le signe de $V'(t)$ puis en déduire le sens de variation de $V(t)$. + \item Que peut-on dire du volume de CO$_2$ quand $t$ devient grand? + \end{enumerate} + \end{enumerate} +\end{exercise} + +\begin{solution} + \begin{enumerate} + \item Volume à 20h: $600000\times 0.009000000000000001 = 5400$ + \item + \begin{enumerate} + \item $t=0$ correspond à 20h. + + Donc $V(0) = 5400 = V_0e^{-0.09\times 0} + 360 = V_0 + 360$ + + Donc $V_0 = 5400 - 360 = 5040$ + \item Il faut calculer $V(t)$ pour $t = 2$ donc + \[ + V(2) = 4569.76 + \] + \item Pas de correction pour cette question. + \item Pas de correction pour cette question. + \item Pas de correction pour cette question. + \end{enumerate} + \end{enumerate} +\end{solution} + + +\end{document} + +%%% Local Variables: +%%% mode: latex +%%% TeX-master: "master" +%%% End: diff --git a/TST_sti2d/DM/2102_DM1/17_2102_DM1.tex b/TST_sti2d/DM/2102_DM1/17_2102_DM1.tex new file mode 100644 index 0000000..3de4df8 --- /dev/null +++ b/TST_sti2d/DM/2102_DM1/17_2102_DM1.tex @@ -0,0 +1,121 @@ +\documentclass[a4paper,10pt]{article} +\usepackage{myXsim} +\usepackage{tasks} + +% Title Page +\title{DM1 \hfill ONAL Yakub} +\tribe{TST sti2d} +\date{\hfillÀ render pour le jeudi 25 février} + +\xsimsetup{ + solution/print = false +} + +\begin{document} +\maketitle + +\begin{exercise}[subtitle={Complexes}] + \begin{enumerate} + \item Mettre le nombre complexe suivant sous forme algébrique $z_1 = \dfrac{8 + 7 i}{-5 + 6 i} $ + \item Mettre le complexe suivante sous forme exponentielle $z_2 = 7 + 7 \sqrt{3} i$ + \item Mettre le complexe suivante sous forme exponentielle $z_3 = \sqrt{3} + i$ + \item Calculer le produit $z_4=z_2\times z_3$ donner le résultat sous forme exponentielle puis algébrique. + \item Calculer le quotient $z_5=\frac{z_2}{z_3}$ donner le résultat sous forme exponentielle puis algébrique. + \end{enumerate} +\end{exercise} + +\begin{solution} + \begin{enumerate} + \item $z_1 = \frac{2}{61} - \frac{83 i}{61}$ + \item $z_3 = 14 e^{\frac{i \pi}{3}}$ + \item $z_4 = 28 e^{\frac{i \pi}{2}} = 28 i = 28.0 i$ + \item $z_5 = 7 e^{\frac{i \pi}{6}} = \frac{7 \sqrt{3}}{2} + \frac{7 i}{2} = 6.06 + 3.5 i$ + \end{enumerate} +\end{solution} + +\begin{exercise}[subtitle={Bassin}] + Le tour d'un bassin au niveau du sol présente deux axes de symétrie : l’axe des abscisses et la droite d’équation $x=4$. Il est obtenu par symétrie de la courbe $\mathcal{C_f}$ sur $\intFF{0}{4}$ où $f$ est la fonction définie par + + \[ + f(x) = \left(- x^{2} + 3.6 x - 8.6\right) e^{- x} + 8.6 + \] + On admet que sur $\intFF{0}{4}$ la fonction $f$ est positive. + \begin{enumerate} + \item Sur un repère, tracer l'allure de la courbe $\mathcal{C}_f$, les axes de symétries puis compléter pour dessiner la forme du bassin. + \item Montrer que la fonction $f$ admet comme primitive sur $\R$ la fonction $F$ définie par + \[ + F(x) = 8.6 x + \left( x^{2} - 1.6 x + 7.0\right) e^{- x} + \] + \item Calculer la quantité $\ds \int_0^4 f(x) \; dx$, vous donnerez le résultat sous forme exacte. Interpréter le résultat et reportez cette quantité sur le graphique. + \item On considère que l'échelle de votre graphique est de 1unité pour 15m. Calculer l'aire du bassin. Vous donnerez un résultat arrondi au $m^2$ près. + \end{enumerate} +\end{exercise} + +\begin{solution} + \begin{enumerate} + \item + \begin{tikzpicture}[baseline=(a.north), xscale=1, yscale=0.5] + \tkzInit[xmin=0,xmax=5,xstep=1, + ymin=0,ymax=10,ystep=1] + \tkzGrid + \tkzAxeXY + \tkzFct[domain=0:10,color=red,very thick]% + { (-x**2 + 3.6*x - 8.6)*exp(-x) + 8.6 }; + \end{tikzpicture} + \item Il faut dériver $F(x)$ et vérifier que $F'(x) = f(x)$. + \item $\ds \int_0^4 f(x) \; dx = F(4) - F(0) = \frac{16.6}{e^{4}} + 27.4$ + \item La quantité calculée à la question précédente se retrouve 4fois pour former le bassin. Il faut ensuite prendre en compte l'échelle, comme 1unité de longueur correspond à 15m, une unité d'air correspond à $15\times15 = 225m^2$. Ainsi l'aire du bassin est égale à + \[ + (\frac{16.6}{e^{4}} + 27.4)\times 4 \times 15^2 = 24934.00000 + \] + + \end{enumerate} +\end{solution} + +\begin{exercise}[subtitle={Bassin}] + Le clinker est un constituant du ciment qui résulte de la cuisson d'un mélange composé de calcaire et d'argile. La fabrication du clinker nécessite des fours à très haute température qui libèrent dans l'air une grande quantité de dioxyde de carbone (CO$_2$). + + Dans une cimenterie, la fabrication du clinker s'effectue de 7 h 30 à 20 h, dans une pièce de volume \np{300000}~dm$^3$. + + À 20 h, après une journée de travail, le taux volumique de CO$_2$ dans la pièce est de 0.6\,\%. + \begin{enumerate} + \item Justifier que le volume de CO$_2$ présent dans cette pièce à 20 h est de \np{1800}~dm$^3$ . + \item On modélise le volume de CO$_2$ présent dans la pièce par une fonction du temps $t$ écoulé après 20h (exprimé en minutes) qui pour formule $V(t) = V_0e^{-0.08t} + 450$ + \begin{enumerate} + \item Démontrer que $V_0$ est égale à \np{1350}. + \item Quel sera, au dm$^3$ près, le volume de CO$_2$ dans cette pièce à 22 h ? + \item Démontrer que $V'(t) = - 108.0 e^{- 0.08 t}$. + \item Étudier le signe de $V'(t)$ puis en déduire le sens de variation de $V(t)$. + \item Que peut-on dire du volume de CO$_2$ quand $t$ devient grand? + \end{enumerate} + \end{enumerate} +\end{exercise} + +\begin{solution} + \begin{enumerate} + \item Volume à 20h: $300000\times 0.006 = 1800$ + \item + \begin{enumerate} + \item $t=0$ correspond à 20h. + + Donc $V(0) = 1800 = V_0e^{-0.08\times 0} + 450 = V_0 + 450$ + + Donc $V_0 = 1800 - 450 = 1350$ + \item Il faut calculer $V(t)$ pour $t = 2$ donc + \[ + V(2) = 1600.39 + \] + \item Pas de correction pour cette question. + \item Pas de correction pour cette question. + \item Pas de correction pour cette question. + \end{enumerate} + \end{enumerate} +\end{solution} + + +\end{document} + +%%% Local Variables: +%%% mode: latex +%%% TeX-master: "master" +%%% End: diff --git a/TST_sti2d/DM/2102_DM1/18_2102_DM1.tex b/TST_sti2d/DM/2102_DM1/18_2102_DM1.tex new file mode 100644 index 0000000..adaad22 --- /dev/null +++ b/TST_sti2d/DM/2102_DM1/18_2102_DM1.tex @@ -0,0 +1,121 @@ +\documentclass[a4paper,10pt]{article} +\usepackage{myXsim} +\usepackage{tasks} + +% Title Page +\title{DM1 \hfill RADOUAA Saleh} +\tribe{TST sti2d} +\date{\hfillÀ render pour le jeudi 25 février} + +\xsimsetup{ + solution/print = false +} + +\begin{document} +\maketitle + +\begin{exercise}[subtitle={Complexes}] + \begin{enumerate} + \item Mettre le nombre complexe suivant sous forme algébrique $z_1 = \dfrac{4 + 2 i}{-8 + 10 i} $ + \item Mettre le complexe suivante sous forme exponentielle $z_2 = 10 - 10 \sqrt{3} i$ + \item Mettre le complexe suivante sous forme exponentielle $z_3 = 9 \sqrt{3} + 9 i$ + \item Calculer le produit $z_4=z_2\times z_3$ donner le résultat sous forme exponentielle puis algébrique. + \item Calculer le quotient $z_5=\frac{z_2}{z_3}$ donner le résultat sous forme exponentielle puis algébrique. + \end{enumerate} +\end{exercise} + +\begin{solution} + \begin{enumerate} + \item $z_1 = - \frac{3}{41} - \frac{14 i}{41}$ + \item $z_3 = 20 e^{- \frac{i \pi}{3}}$ + \item $z_4 = 360 e^{- \frac{i \pi}{6}} = 180 \sqrt{3} - 180 i = 312.0 - 180.0 i$ + \item $z_5 = \frac{10}{9} e^{- \frac{i \pi}{2}} = - \frac{10 i}{9} = - 1.11 i$ + \end{enumerate} +\end{solution} + +\begin{exercise}[subtitle={Bassin}] + Le tour d'un bassin au niveau du sol présente deux axes de symétrie : l’axe des abscisses et la droite d’équation $x=4$. Il est obtenu par symétrie de la courbe $\mathcal{C_f}$ sur $\intFF{0}{4}$ où $f$ est la fonction définie par + + \[ + f(x) = \left(- x^{2} + 0.6 x - 2.3\right) e^{- x} + 2.3 + \] + On admet que sur $\intFF{0}{4}$ la fonction $f$ est positive. + \begin{enumerate} + \item Sur un repère, tracer l'allure de la courbe $\mathcal{C}_f$, les axes de symétries puis compléter pour dessiner la forme du bassin. + \item Montrer que la fonction $f$ admet comme primitive sur $\R$ la fonction $F$ définie par + \[ + F(x) = 2.3 x + \left( x^{2} + 1.4 x + 3.7\right) e^{- x} + \] + \item Calculer la quantité $\ds \int_0^4 f(x) \; dx$, vous donnerez le résultat sous forme exacte. Interpréter le résultat et reportez cette quantité sur le graphique. + \item On considère que l'échelle de votre graphique est de 1unité pour 15m. Calculer l'aire du bassin. Vous donnerez un résultat arrondi au $m^2$ près. + \end{enumerate} +\end{exercise} + +\begin{solution} + \begin{enumerate} + \item + \begin{tikzpicture}[baseline=(a.north), xscale=1, yscale=0.5] + \tkzInit[xmin=0,xmax=5,xstep=1, + ymin=0,ymax=10,ystep=1] + \tkzGrid + \tkzAxeXY + \tkzFct[domain=0:10,color=red,very thick]% + { (-x**2 + 0.6*x - 2.3)*exp(-x) + 2.3 }; + \end{tikzpicture} + \item Il faut dériver $F(x)$ et vérifier que $F'(x) = f(x)$. + \item $\ds \int_0^4 f(x) \; dx = F(4) - F(0) = \frac{25.3}{e^{4}} + 5.5$ + \item La quantité calculée à la question précédente se retrouve 4fois pour former le bassin. Il faut ensuite prendre en compte l'échelle, comme 1unité de longueur correspond à 15m, une unité d'air correspond à $15\times15 = 225m^2$. Ainsi l'aire du bassin est égale à + \[ + (\frac{25.3}{e^{4}} + 5.5)\times 4 \times 15^2 = 5367.000000 + \] + + \end{enumerate} +\end{solution} + +\begin{exercise}[subtitle={Bassin}] + Le clinker est un constituant du ciment qui résulte de la cuisson d'un mélange composé de calcaire et d'argile. La fabrication du clinker nécessite des fours à très haute température qui libèrent dans l'air une grande quantité de dioxyde de carbone (CO$_2$). + + Dans une cimenterie, la fabrication du clinker s'effectue de 7 h 30 à 20 h, dans une pièce de volume \np{500000}~dm$^3$. + + À 20 h, après une journée de travail, le taux volumique de CO$_2$ dans la pièce est de 0.9\,\%. + \begin{enumerate} + \item Justifier que le volume de CO$_2$ présent dans cette pièce à 20 h est de \np{4500}~dm$^3$ . + \item On modélise le volume de CO$_2$ présent dans la pièce par une fonction du temps $t$ écoulé après 20h (exprimé en minutes) qui pour formule $V(t) = V_0e^{-0.06t} + 440$ + \begin{enumerate} + \item Démontrer que $V_0$ est égale à \np{4060}. + \item Quel sera, au dm$^3$ près, le volume de CO$_2$ dans cette pièce à 24 h ? + \item Démontrer que $V'(t) = - 243.6 e^{- 0.06 t}$. + \item Étudier le signe de $V'(t)$ puis en déduire le sens de variation de $V(t)$. + \item Que peut-on dire du volume de CO$_2$ quand $t$ devient grand? + \end{enumerate} + \end{enumerate} +\end{exercise} + +\begin{solution} + \begin{enumerate} + \item Volume à 20h: $500000\times 0.009000000000000001 = 4500$ + \item + \begin{enumerate} + \item $t=0$ correspond à 20h. + + Donc $V(0) = 4500 = V_0e^{-0.06\times 0} + 440 = V_0 + 440$ + + Donc $V_0 = 4500 - 440 = 4060$ + \item Il faut calculer $V(t)$ pour $t = 4$ donc + \[ + V(4) = 3633.71 + \] + \item Pas de correction pour cette question. + \item Pas de correction pour cette question. + \item Pas de correction pour cette question. + \end{enumerate} + \end{enumerate} +\end{solution} + + +\end{document} + +%%% Local Variables: +%%% mode: latex +%%% TeX-master: "master" +%%% End: diff --git a/TST_sti2d/DM/2102_DM1/19_2102_DM1.tex b/TST_sti2d/DM/2102_DM1/19_2102_DM1.tex new file mode 100644 index 0000000..4376d23 --- /dev/null +++ b/TST_sti2d/DM/2102_DM1/19_2102_DM1.tex @@ -0,0 +1,121 @@ +\documentclass[a4paper,10pt]{article} +\usepackage{myXsim} +\usepackage{tasks} + +% Title Page +\title{DM1 \hfill TAVERNIER Joanny} +\tribe{TST sti2d} +\date{\hfillÀ render pour le jeudi 25 février} + +\xsimsetup{ + solution/print = false +} + +\begin{document} +\maketitle + +\begin{exercise}[subtitle={Complexes}] + \begin{enumerate} + \item Mettre le nombre complexe suivant sous forme algébrique $z_1 = \dfrac{10 + 6 i}{-7 + 3 i} $ + \item Mettre le complexe suivante sous forme exponentielle $z_2 = 6 - 6 \sqrt{3} i$ + \item Mettre le complexe suivante sous forme exponentielle $z_3 = 5 \sqrt{3} - 5 i$ + \item Calculer le produit $z_4=z_2\times z_3$ donner le résultat sous forme exponentielle puis algébrique. + \item Calculer le quotient $z_5=\frac{z_2}{z_3}$ donner le résultat sous forme exponentielle puis algébrique. + \end{enumerate} +\end{exercise} + +\begin{solution} + \begin{enumerate} + \item $z_1 = - \frac{26}{29} - \frac{36 i}{29}$ + \item $z_3 = 12 e^{- \frac{i \pi}{3}}$ + \item $z_4 = 120 e^{- \frac{i \pi}{2}} = - 120 i = - 120.0 i$ + \item $z_5 = \frac{6}{5} e^{- \frac{i \pi}{6}} = \frac{3 \sqrt{3}}{5} - \frac{3 i}{5} = 1.04 - 0.6 i$ + \end{enumerate} +\end{solution} + +\begin{exercise}[subtitle={Bassin}] + Le tour d'un bassin au niveau du sol présente deux axes de symétrie : l’axe des abscisses et la droite d’équation $x=4$. Il est obtenu par symétrie de la courbe $\mathcal{C_f}$ sur $\intFF{0}{4}$ où $f$ est la fonction définie par + + \[ + f(x) = \left(- x^{2} + 8.2 x - 3.6\right) e^{- x} + 3.6 + \] + On admet que sur $\intFF{0}{4}$ la fonction $f$ est positive. + \begin{enumerate} + \item Sur un repère, tracer l'allure de la courbe $\mathcal{C}_f$, les axes de symétries puis compléter pour dessiner la forme du bassin. + \item Montrer que la fonction $f$ admet comme primitive sur $\R$ la fonction $F$ définie par + \[ + F(x) = 3.6 x + \left( x^{2} - 6.2 x - 2.6\right) e^{- x} + \] + \item Calculer la quantité $\ds \int_0^4 f(x) \; dx$, vous donnerez le résultat sous forme exacte. Interpréter le résultat et reportez cette quantité sur le graphique. + \item On considère que l'échelle de votre graphique est de 1unité pour 15m. Calculer l'aire du bassin. Vous donnerez un résultat arrondi au $m^2$ près. + \end{enumerate} +\end{exercise} + +\begin{solution} + \begin{enumerate} + \item + \begin{tikzpicture}[baseline=(a.north), xscale=1, yscale=0.5] + \tkzInit[xmin=0,xmax=5,xstep=1, + ymin=0,ymax=10,ystep=1] + \tkzGrid + \tkzAxeXY + \tkzFct[domain=0:10,color=red,very thick]% + { (-x**2 + 8.2*x - 3.6)*exp(-x) + 3.6 }; + \end{tikzpicture} + \item Il faut dériver $F(x)$ et vérifier que $F'(x) = f(x)$. + \item $\ds \int_0^4 f(x) \; dx = F(4) - F(0) = 17.0 - \frac{11.4}{e^{4}}$ + \item La quantité calculée à la question précédente se retrouve 4fois pour former le bassin. Il faut ensuite prendre en compte l'échelle, comme 1unité de longueur correspond à 15m, une unité d'air correspond à $15\times15 = 225m^2$. Ainsi l'aire du bassin est égale à + \[ + (17.0 - \frac{11.4}{e^{4}})\times 4 \times 15^2 = 15112.00000 + \] + + \end{enumerate} +\end{solution} + +\begin{exercise}[subtitle={Bassin}] + Le clinker est un constituant du ciment qui résulte de la cuisson d'un mélange composé de calcaire et d'argile. La fabrication du clinker nécessite des fours à très haute température qui libèrent dans l'air une grande quantité de dioxyde de carbone (CO$_2$). + + Dans une cimenterie, la fabrication du clinker s'effectue de 7 h 30 à 20 h, dans une pièce de volume \np{100000}~dm$^3$. + + À 20 h, après une journée de travail, le taux volumique de CO$_2$ dans la pièce est de 0.8\,\%. + \begin{enumerate} + \item Justifier que le volume de CO$_2$ présent dans cette pièce à 20 h est de \np{800}~dm$^3$ . + \item On modélise le volume de CO$_2$ présent dans la pièce par une fonction du temps $t$ écoulé après 20h (exprimé en minutes) qui pour formule $V(t) = V_0e^{-0.01t} + 560$ + \begin{enumerate} + \item Démontrer que $V_0$ est égale à \np{240}. + \item Quel sera, au dm$^3$ près, le volume de CO$_2$ dans cette pièce à 22 h ? + \item Démontrer que $V'(t) = - 2.4 e^{- 0.01 t}$. + \item Étudier le signe de $V'(t)$ puis en déduire le sens de variation de $V(t)$. + \item Que peut-on dire du volume de CO$_2$ quand $t$ devient grand? + \end{enumerate} + \end{enumerate} +\end{exercise} + +\begin{solution} + \begin{enumerate} + \item Volume à 20h: $100000\times 0.008 = 800$ + \item + \begin{enumerate} + \item $t=0$ correspond à 20h. + + Donc $V(0) = 800 = V_0e^{-0.01\times 0} + 560 = V_0 + 560$ + + Donc $V_0 = 800 - 560 = 240$ + \item Il faut calculer $V(t)$ pour $t = 2$ donc + \[ + V(2) = 795.25 + \] + \item Pas de correction pour cette question. + \item Pas de correction pour cette question. + \item Pas de correction pour cette question. + \end{enumerate} + \end{enumerate} +\end{solution} + + +\end{document} + +%%% Local Variables: +%%% mode: latex +%%% TeX-master: "master" +%%% End: diff --git a/TST_sti2d/DM/2102_DM1/20_2102_DM1.tex b/TST_sti2d/DM/2102_DM1/20_2102_DM1.tex new file mode 100644 index 0000000..0ffaa8e --- /dev/null +++ b/TST_sti2d/DM/2102_DM1/20_2102_DM1.tex @@ -0,0 +1,121 @@ +\documentclass[a4paper,10pt]{article} +\usepackage{myXsim} +\usepackage{tasks} + +% Title Page +\title{DM1 \hfill ZAHORE Zahiri} +\tribe{TST sti2d} +\date{\hfillÀ render pour le jeudi 25 février} + +\xsimsetup{ + solution/print = false +} + +\begin{document} +\maketitle + +\begin{exercise}[subtitle={Complexes}] + \begin{enumerate} + \item Mettre le nombre complexe suivant sous forme algébrique $z_1 = \dfrac{2 + 4 i}{-4 + 9 i} $ + \item Mettre le complexe suivante sous forme exponentielle $z_2 = - 2 \sqrt{3} - 2 i$ + \item Mettre le complexe suivante sous forme exponentielle $z_3 = 6 + 6 \sqrt{3} i$ + \item Calculer le produit $z_4=z_2\times z_3$ donner le résultat sous forme exponentielle puis algébrique. + \item Calculer le quotient $z_5=\frac{z_2}{z_3}$ donner le résultat sous forme exponentielle puis algébrique. + \end{enumerate} +\end{exercise} + +\begin{solution} + \begin{enumerate} + \item $z_1 = \frac{28}{97} - \frac{34 i}{97}$ + \item $z_3 = 4 e^{- \frac{5 i \pi}{6}}$ + \item $z_4 = 48 e^{- \frac{i \pi}{2}} = - 48 i = - 48.0 i$ + \item $z_5 = \frac{1}{3} e^{- \frac{7 i \pi}{6}} = - \frac{\sqrt{3}}{6} + \frac{i}{6} = -0.289 + 0.167 i$ + \end{enumerate} +\end{solution} + +\begin{exercise}[subtitle={Bassin}] + Le tour d'un bassin au niveau du sol présente deux axes de symétrie : l’axe des abscisses et la droite d’équation $x=4$. Il est obtenu par symétrie de la courbe $\mathcal{C_f}$ sur $\intFF{0}{4}$ où $f$ est la fonction définie par + + \[ + f(x) = \left(- x^{2} + 4.8 x - 3.0\right) e^{- x} + 3.0 + \] + On admet que sur $\intFF{0}{4}$ la fonction $f$ est positive. + \begin{enumerate} + \item Sur un repère, tracer l'allure de la courbe $\mathcal{C}_f$, les axes de symétries puis compléter pour dessiner la forme du bassin. + \item Montrer que la fonction $f$ admet comme primitive sur $\R$ la fonction $F$ définie par + \[ + F(x) = 3.0 x + \left( x^{2} - 2.8 x + 0.2\right) e^{- x} + \] + \item Calculer la quantité $\ds \int_0^4 f(x) \; dx$, vous donnerez le résultat sous forme exacte. Interpréter le résultat et reportez cette quantité sur le graphique. + \item On considère que l'échelle de votre graphique est de 1unité pour 15m. Calculer l'aire du bassin. Vous donnerez un résultat arrondi au $m^2$ près. + \end{enumerate} +\end{exercise} + +\begin{solution} + \begin{enumerate} + \item + \begin{tikzpicture}[baseline=(a.north), xscale=1, yscale=0.5] + \tkzInit[xmin=0,xmax=5,xstep=1, + ymin=0,ymax=10,ystep=1] + \tkzGrid + \tkzAxeXY + \tkzFct[domain=0:10,color=red,very thick]% + { (-x**2 + 4.8*x - 3.0)*exp(-x) + 3.0 }; + \end{tikzpicture} + \item Il faut dériver $F(x)$ et vérifier que $F'(x) = f(x)$. + \item $\ds \int_0^4 f(x) \; dx = F(4) - F(0) = \frac{5.0}{e^{4}} + 11.8$ + \item La quantité calculée à la question précédente se retrouve 4fois pour former le bassin. Il faut ensuite prendre en compte l'échelle, comme 1unité de longueur correspond à 15m, une unité d'air correspond à $15\times15 = 225m^2$. Ainsi l'aire du bassin est égale à + \[ + (\frac{5.0}{e^{4}} + 11.8)\times 4 \times 15^2 = 10702.00000 + \] + + \end{enumerate} +\end{solution} + +\begin{exercise}[subtitle={Bassin}] + Le clinker est un constituant du ciment qui résulte de la cuisson d'un mélange composé de calcaire et d'argile. La fabrication du clinker nécessite des fours à très haute température qui libèrent dans l'air une grande quantité de dioxyde de carbone (CO$_2$). + + Dans une cimenterie, la fabrication du clinker s'effectue de 7 h 30 à 20 h, dans une pièce de volume \np{500000}~dm$^3$. + + À 20 h, après une journée de travail, le taux volumique de CO$_2$ dans la pièce est de 0.8\,\%. + \begin{enumerate} + \item Justifier que le volume de CO$_2$ présent dans cette pièce à 20 h est de \np{4000}~dm$^3$ . + \item On modélise le volume de CO$_2$ présent dans la pièce par une fonction du temps $t$ écoulé après 20h (exprimé en minutes) qui pour formule $V(t) = V_0e^{-0.05t} + 230$ + \begin{enumerate} + \item Démontrer que $V_0$ est égale à \np{3770}. + \item Quel sera, au dm$^3$ près, le volume de CO$_2$ dans cette pièce à 24 h ? + \item Démontrer que $V'(t) = - 188.5 e^{- 0.05 t}$. + \item Étudier le signe de $V'(t)$ puis en déduire le sens de variation de $V(t)$. + \item Que peut-on dire du volume de CO$_2$ quand $t$ devient grand? + \end{enumerate} + \end{enumerate} +\end{exercise} + +\begin{solution} + \begin{enumerate} + \item Volume à 20h: $500000\times 0.008 = 4000$ + \item + \begin{enumerate} + \item $t=0$ correspond à 20h. + + Donc $V(0) = 4000 = V_0e^{-0.05\times 0} + 230 = V_0 + 230$ + + Donc $V_0 = 4000 - 230 = 3770$ + \item Il faut calculer $V(t)$ pour $t = 4$ donc + \[ + V(4) = 3316.61 + \] + \item Pas de correction pour cette question. + \item Pas de correction pour cette question. + \item Pas de correction pour cette question. + \end{enumerate} + \end{enumerate} +\end{solution} + + +\end{document} + +%%% Local Variables: +%%% mode: latex +%%% TeX-master: "master" +%%% End: diff --git a/TST_sti2d/DM/2102_DM1/all_2102_DM1.pdf b/TST_sti2d/DM/2102_DM1/all_2102_DM1.pdf new file mode 100644 index 0000000..5d63ded Binary files /dev/null and b/TST_sti2d/DM/2102_DM1/all_2102_DM1.pdf differ diff --git a/TST_sti2d/DM/2102_DM1/corr_01_2102_DM1.tex b/TST_sti2d/DM/2102_DM1/corr_01_2102_DM1.tex new file mode 100644 index 0000000..5c72692 --- /dev/null +++ b/TST_sti2d/DM/2102_DM1/corr_01_2102_DM1.tex @@ -0,0 +1,121 @@ +\documentclass[a4paper,10pt]{article} +\usepackage{myXsim} +\usepackage{tasks} + +% Title Page +\title{DM1 \hfill BAHBAH Zakaria} +\tribe{TST sti2d} +\date{\hfillÀ render pour le jeudi 25 février} + +\xsimsetup{ + solution/print = true +} + +\begin{document} +\maketitle + +\begin{exercise}[subtitle={Complexes}] + \begin{enumerate} + \item Mettre le nombre complexe suivant sous forme algébrique $z_1 = \dfrac{7 + 2 i}{-5 + 3 i} $ + \item Mettre le complexe suivante sous forme exponentielle $z_2 = -7 + 7 \sqrt{3} i$ + \item Mettre le complexe suivante sous forme exponentielle $z_3 = 4 + 4 \sqrt{3} i$ + \item Calculer le produit $z_4=z_2\times z_3$ donner le résultat sous forme exponentielle puis algébrique. + \item Calculer le quotient $z_5=\frac{z_2}{z_3}$ donner le résultat sous forme exponentielle puis algébrique. + \end{enumerate} +\end{exercise} + +\begin{solution} + \begin{enumerate} + \item $z_1 = - \frac{29}{34} - \frac{31 i}{34}$ + \item $z_3 = 14 e^{\frac{2 i \pi}{3}}$ + \item $z_4 = 112 e^{i \pi} = -112 = -112.0$ + \item $z_5 = \frac{7}{4} e^{\frac{i \pi}{3}} = \frac{7}{8} + \frac{7 \sqrt{3} i}{8} = 0.875 + 1.52 i$ + \end{enumerate} +\end{solution} + +\begin{exercise}[subtitle={Bassin}] + Le tour d'un bassin au niveau du sol présente deux axes de symétrie : l’axe des abscisses et la droite d’équation $x=4$. Il est obtenu par symétrie de la courbe $\mathcal{C_f}$ sur $\intFF{0}{4}$ où $f$ est la fonction définie par + + \[ + f(x) = \left(- x^{2} + 2.4 x - 6.2\right) e^{- x} + 6.2 + \] + On admet que sur $\intFF{0}{4}$ la fonction $f$ est positive. + \begin{enumerate} + \item Sur un repère, tracer l'allure de la courbe $\mathcal{C}_f$, les axes de symétries puis compléter pour dessiner la forme du bassin. + \item Montrer que la fonction $f$ admet comme primitive sur $\R$ la fonction $F$ définie par + \[ + F(x) = 6.2 x + \left( x^{2} - 0.4 x + 5.8\right) e^{- x} + \] + \item Calculer la quantité $\ds \int_0^4 f(x) \; dx$, vous donnerez le résultat sous forme exacte. Interpréter le résultat et reportez cette quantité sur le graphique. + \item On considère que l'échelle de votre graphique est de 1unité pour 15m. Calculer l'aire du bassin. Vous donnerez un résultat arrondi au $m^2$ près. + \end{enumerate} +\end{exercise} + +\begin{solution} + \begin{enumerate} + \item + \begin{tikzpicture}[baseline=(a.north), xscale=1, yscale=0.5] + \tkzInit[xmin=0,xmax=5,xstep=1, + ymin=0,ymax=10,ystep=1] + \tkzGrid + \tkzAxeXY + \tkzFct[domain=0:10,color=red,very thick]% + { (-x**2 + 2.4*x - 6.2)*exp(-x) + 6.2 }; + \end{tikzpicture} + \item Il faut dériver $F(x)$ et vérifier que $F'(x) = f(x)$. + \item $\ds \int_0^4 f(x) \; dx = F(4) - F(0) = \frac{20.2}{e^{4}} + 19.0$ + \item La quantité calculée à la question précédente se retrouve 4fois pour former le bassin. Il faut ensuite prendre en compte l'échelle, comme 1unité de longueur correspond à 15m, une unité d'air correspond à $15\times15 = 225m^2$. Ainsi l'aire du bassin est égale à + \[ + (\frac{20.2}{e^{4}} + 19.0)\times 4 \times 15^2 = 17433.00000 + \] + + \end{enumerate} +\end{solution} + +\begin{exercise}[subtitle={Bassin}] + Le clinker est un constituant du ciment qui résulte de la cuisson d'un mélange composé de calcaire et d'argile. La fabrication du clinker nécessite des fours à très haute température qui libèrent dans l'air une grande quantité de dioxyde de carbone (CO$_2$). + + Dans une cimenterie, la fabrication du clinker s'effectue de 7 h 30 à 20 h, dans une pièce de volume \np{900000}~dm$^3$. + + À 20 h, après une journée de travail, le taux volumique de CO$_2$ dans la pièce est de 0.9\,\%. + \begin{enumerate} + \item Justifier que le volume de CO$_2$ présent dans cette pièce à 20 h est de \np{8100}~dm$^3$ . + \item On modélise le volume de CO$_2$ présent dans la pièce par une fonction du temps $t$ écoulé après 20h (exprimé en minutes) qui pour formule $V(t) = V_0e^{-0.06t} + 560$ + \begin{enumerate} + \item Démontrer que $V_0$ est égale à \np{7540}. + \item Quel sera, au dm$^3$ près, le volume de CO$_2$ dans cette pièce à 23 h ? + \item Démontrer que $V'(t) = - 452.4 e^{- 0.06 t}$. + \item Étudier le signe de $V'(t)$ puis en déduire le sens de variation de $V(t)$. + \item Que peut-on dire du volume de CO$_2$ quand $t$ devient grand? + \end{enumerate} + \end{enumerate} +\end{exercise} + +\begin{solution} + \begin{enumerate} + \item Volume à 20h: $900000\times 0.009000000000000001 = 8100$ + \item + \begin{enumerate} + \item $t=0$ correspond à 20h. + + Donc $V(0) = 8100 = V_0e^{-0.06\times 0} + 560 = V_0 + 560$ + + Donc $V_0 = 8100 - 560 = 7540$ + \item Il faut calculer $V(t)$ pour $t = 3$ donc + \[ + V(3) = 6857.94 + \] + \item Pas de correction pour cette question. + \item Pas de correction pour cette question. + \item Pas de correction pour cette question. + \end{enumerate} + \end{enumerate} +\end{solution} + + +\end{document} + +%%% Local Variables: +%%% mode: latex +%%% TeX-master: "master" +%%% End: diff --git a/TST_sti2d/DM/2102_DM1/corr_02_2102_DM1.tex b/TST_sti2d/DM/2102_DM1/corr_02_2102_DM1.tex new file mode 100644 index 0000000..b7bb09f --- /dev/null +++ b/TST_sti2d/DM/2102_DM1/corr_02_2102_DM1.tex @@ -0,0 +1,121 @@ +\documentclass[a4paper,10pt]{article} +\usepackage{myXsim} +\usepackage{tasks} + +% Title Page +\title{DM1 \hfill BENALI Ilyas} +\tribe{TST sti2d} +\date{\hfillÀ render pour le jeudi 25 février} + +\xsimsetup{ + solution/print = true +} + +\begin{document} +\maketitle + +\begin{exercise}[subtitle={Complexes}] + \begin{enumerate} + \item Mettre le nombre complexe suivant sous forme algébrique $z_1 = \dfrac{9 + 3 i}{-9 + 8 i} $ + \item Mettre le complexe suivante sous forme exponentielle $z_2 = 2 \sqrt{2} + 2 \sqrt{2} i$ + \item Mettre le complexe suivante sous forme exponentielle $z_3 = 4 \sqrt{2} - 4 \sqrt{2} i$ + \item Calculer le produit $z_4=z_2\times z_3$ donner le résultat sous forme exponentielle puis algébrique. + \item Calculer le quotient $z_5=\frac{z_2}{z_3}$ donner le résultat sous forme exponentielle puis algébrique. + \end{enumerate} +\end{exercise} + +\begin{solution} + \begin{enumerate} + \item $z_1 = - \frac{57}{145} - \frac{99 i}{145}$ + \item $z_3 = 4 e^{\frac{i \pi}{4}}$ + \item $z_4 = 32 e^{0} = 32 = 32.0$ + \item $z_5 = \frac{1}{2} e^{\frac{i \pi}{2}} = \frac{i}{2} = 0.5 i$ + \end{enumerate} +\end{solution} + +\begin{exercise}[subtitle={Bassin}] + Le tour d'un bassin au niveau du sol présente deux axes de symétrie : l’axe des abscisses et la droite d’équation $x=4$. Il est obtenu par symétrie de la courbe $\mathcal{C_f}$ sur $\intFF{0}{4}$ où $f$ est la fonction définie par + + \[ + f(x) = \left(- x^{2} + 8.7 x - 5.0\right) e^{- x} + 5.0 + \] + On admet que sur $\intFF{0}{4}$ la fonction $f$ est positive. + \begin{enumerate} + \item Sur un repère, tracer l'allure de la courbe $\mathcal{C}_f$, les axes de symétries puis compléter pour dessiner la forme du bassin. + \item Montrer que la fonction $f$ admet comme primitive sur $\R$ la fonction $F$ définie par + \[ + F(x) = 5.0 x + \left( x^{2} - 6.7 x - 1.7\right) e^{- x} + \] + \item Calculer la quantité $\ds \int_0^4 f(x) \; dx$, vous donnerez le résultat sous forme exacte. Interpréter le résultat et reportez cette quantité sur le graphique. + \item On considère que l'échelle de votre graphique est de 1unité pour 15m. Calculer l'aire du bassin. Vous donnerez un résultat arrondi au $m^2$ près. + \end{enumerate} +\end{exercise} + +\begin{solution} + \begin{enumerate} + \item + \begin{tikzpicture}[baseline=(a.north), xscale=1, yscale=0.5] + \tkzInit[xmin=0,xmax=5,xstep=1, + ymin=0,ymax=10,ystep=1] + \tkzGrid + \tkzAxeXY + \tkzFct[domain=0:10,color=red,very thick]% + { (-x**2 + 8.7*x - 5.0)*exp(-x) + 5.0 }; + \end{tikzpicture} + \item Il faut dériver $F(x)$ et vérifier que $F'(x) = f(x)$. + \item $\ds \int_0^4 f(x) \; dx = F(4) - F(0) = 21.7 - \frac{12.5}{e^{4}}$ + \item La quantité calculée à la question précédente se retrouve 4fois pour former le bassin. Il faut ensuite prendre en compte l'échelle, comme 1unité de longueur correspond à 15m, une unité d'air correspond à $15\times15 = 225m^2$. Ainsi l'aire du bassin est égale à + \[ + (21.7 - \frac{12.5}{e^{4}})\times 4 \times 15^2 = 19324.00000 + \] + + \end{enumerate} +\end{solution} + +\begin{exercise}[subtitle={Bassin}] + Le clinker est un constituant du ciment qui résulte de la cuisson d'un mélange composé de calcaire et d'argile. La fabrication du clinker nécessite des fours à très haute température qui libèrent dans l'air une grande quantité de dioxyde de carbone (CO$_2$). + + Dans une cimenterie, la fabrication du clinker s'effectue de 7 h 30 à 20 h, dans une pièce de volume \np{700000}~dm$^3$. + + À 20 h, après une journée de travail, le taux volumique de CO$_2$ dans la pièce est de 0.8\,\%. + \begin{enumerate} + \item Justifier que le volume de CO$_2$ présent dans cette pièce à 20 h est de \np{5600}~dm$^3$ . + \item On modélise le volume de CO$_2$ présent dans la pièce par une fonction du temps $t$ écoulé après 20h (exprimé en minutes) qui pour formule $V(t) = V_0e^{-0.09t} + 360$ + \begin{enumerate} + \item Démontrer que $V_0$ est égale à \np{5240}. + \item Quel sera, au dm$^3$ près, le volume de CO$_2$ dans cette pièce à 21 h ? + \item Démontrer que $V'(t) = - 471.6 e^{- 0.09 t}$. + \item Étudier le signe de $V'(t)$ puis en déduire le sens de variation de $V(t)$. + \item Que peut-on dire du volume de CO$_2$ quand $t$ devient grand? + \end{enumerate} + \end{enumerate} +\end{exercise} + +\begin{solution} + \begin{enumerate} + \item Volume à 20h: $700000\times 0.008 = 5600$ + \item + \begin{enumerate} + \item $t=0$ correspond à 20h. + + Donc $V(0) = 5600 = V_0e^{-0.09\times 0} + 360 = V_0 + 360$ + + Donc $V_0 = 5600 - 360 = 5240$ + \item Il faut calculer $V(t)$ pour $t = 1$ donc + \[ + V(1) = 5149.00000000000 + \] + \item Pas de correction pour cette question. + \item Pas de correction pour cette question. + \item Pas de correction pour cette question. + \end{enumerate} + \end{enumerate} +\end{solution} + + +\end{document} + +%%% Local Variables: +%%% mode: latex +%%% TeX-master: "master" +%%% End: diff --git a/TST_sti2d/DM/2102_DM1/corr_03_2102_DM1.tex b/TST_sti2d/DM/2102_DM1/corr_03_2102_DM1.tex new file mode 100644 index 0000000..abe00d9 --- /dev/null +++ b/TST_sti2d/DM/2102_DM1/corr_03_2102_DM1.tex @@ -0,0 +1,121 @@ +\documentclass[a4paper,10pt]{article} +\usepackage{myXsim} +\usepackage{tasks} + +% Title Page +\title{DM1 \hfill BERNADAT Noah} +\tribe{TST sti2d} +\date{\hfillÀ render pour le jeudi 25 février} + +\xsimsetup{ + solution/print = true +} + +\begin{document} +\maketitle + +\begin{exercise}[subtitle={Complexes}] + \begin{enumerate} + \item Mettre le nombre complexe suivant sous forme algébrique $z_1 = \dfrac{2 + 3 i}{-7 + 10 i} $ + \item Mettre le complexe suivante sous forme exponentielle $z_2 = - 5 \sqrt{3} + 5 i$ + \item Mettre le complexe suivante sous forme exponentielle $z_3 = 3 \sqrt{3} - 3 i$ + \item Calculer le produit $z_4=z_2\times z_3$ donner le résultat sous forme exponentielle puis algébrique. + \item Calculer le quotient $z_5=\frac{z_2}{z_3}$ donner le résultat sous forme exponentielle puis algébrique. + \end{enumerate} +\end{exercise} + +\begin{solution} + \begin{enumerate} + \item $z_1 = \frac{16}{149} - \frac{41 i}{149}$ + \item $z_3 = 10 e^{\frac{5 i \pi}{6}}$ + \item $z_4 = 60 e^{\frac{2 i \pi}{3}} = -30 + 30 \sqrt{3} i = -30.0 + 52.0 i$ + \item $z_5 = \frac{5}{3} e^{i \pi} = - \frac{5}{3} = -1.67$ + \end{enumerate} +\end{solution} + +\begin{exercise}[subtitle={Bassin}] + Le tour d'un bassin au niveau du sol présente deux axes de symétrie : l’axe des abscisses et la droite d’équation $x=4$. Il est obtenu par symétrie de la courbe $\mathcal{C_f}$ sur $\intFF{0}{4}$ où $f$ est la fonction définie par + + \[ + f(x) = \left(- x^{2} + 4.9 x - 9.7\right) e^{- x} + 9.7 + \] + On admet que sur $\intFF{0}{4}$ la fonction $f$ est positive. + \begin{enumerate} + \item Sur un repère, tracer l'allure de la courbe $\mathcal{C}_f$, les axes de symétries puis compléter pour dessiner la forme du bassin. + \item Montrer que la fonction $f$ admet comme primitive sur $\R$ la fonction $F$ définie par + \[ + F(x) = 9.7 x + \left( x^{2} - 2.9 x + 6.8\right) e^{- x} + \] + \item Calculer la quantité $\ds \int_0^4 f(x) \; dx$, vous donnerez le résultat sous forme exacte. Interpréter le résultat et reportez cette quantité sur le graphique. + \item On considère que l'échelle de votre graphique est de 1unité pour 15m. Calculer l'aire du bassin. Vous donnerez un résultat arrondi au $m^2$ près. + \end{enumerate} +\end{exercise} + +\begin{solution} + \begin{enumerate} + \item + \begin{tikzpicture}[baseline=(a.north), xscale=1, yscale=0.5] + \tkzInit[xmin=0,xmax=5,xstep=1, + ymin=0,ymax=10,ystep=1] + \tkzGrid + \tkzAxeXY + \tkzFct[domain=0:10,color=red,very thick]% + { (-x**2 + 4.9*x - 9.7)*exp(-x) + 9.7 }; + \end{tikzpicture} + \item Il faut dériver $F(x)$ et vérifier que $F'(x) = f(x)$. + \item $\ds \int_0^4 f(x) \; dx = F(4) - F(0) = \frac{11.2}{e^{4}} + 32.0$ + \item La quantité calculée à la question précédente se retrouve 4fois pour former le bassin. Il faut ensuite prendre en compte l'échelle, comme 1unité de longueur correspond à 15m, une unité d'air correspond à $15\times15 = 225m^2$. Ainsi l'aire du bassin est égale à + \[ + (\frac{11.2}{e^{4}} + 32.0)\times 4 \times 15^2 = 28985.00000 + \] + + \end{enumerate} +\end{solution} + +\begin{exercise}[subtitle={Bassin}] + Le clinker est un constituant du ciment qui résulte de la cuisson d'un mélange composé de calcaire et d'argile. La fabrication du clinker nécessite des fours à très haute température qui libèrent dans l'air une grande quantité de dioxyde de carbone (CO$_2$). + + Dans une cimenterie, la fabrication du clinker s'effectue de 7 h 30 à 20 h, dans une pièce de volume \np{400000}~dm$^3$. + + À 20 h, après une journée de travail, le taux volumique de CO$_2$ dans la pièce est de 0.7\,\%. + \begin{enumerate} + \item Justifier que le volume de CO$_2$ présent dans cette pièce à 20 h est de \np{2800}~dm$^3$ . + \item On modélise le volume de CO$_2$ présent dans la pièce par une fonction du temps $t$ écoulé après 20h (exprimé en minutes) qui pour formule $V(t) = V_0e^{-0.06t} + 400$ + \begin{enumerate} + \item Démontrer que $V_0$ est égale à \np{2400}. + \item Quel sera, au dm$^3$ près, le volume de CO$_2$ dans cette pièce à 23 h ? + \item Démontrer que $V'(t) = - 144.0 e^{- 0.06 t}$. + \item Étudier le signe de $V'(t)$ puis en déduire le sens de variation de $V(t)$. + \item Que peut-on dire du volume de CO$_2$ quand $t$ devient grand? + \end{enumerate} + \end{enumerate} +\end{exercise} + +\begin{solution} + \begin{enumerate} + \item Volume à 20h: $400000\times 0.006999999999999999 = 2800$ + \item + \begin{enumerate} + \item $t=0$ correspond à 20h. + + Donc $V(0) = 2800 = V_0e^{-0.06\times 0} + 400 = V_0 + 400$ + + Donc $V_0 = 2800 - 400 = 2400$ + \item Il faut calculer $V(t)$ pour $t = 3$ donc + \[ + V(3) = 2404.65 + \] + \item Pas de correction pour cette question. + \item Pas de correction pour cette question. + \item Pas de correction pour cette question. + \end{enumerate} + \end{enumerate} +\end{solution} + + +\end{document} + +%%% Local Variables: +%%% mode: latex +%%% TeX-master: "master" +%%% End: diff --git a/TST_sti2d/DM/2102_DM1/corr_04_2102_DM1.tex b/TST_sti2d/DM/2102_DM1/corr_04_2102_DM1.tex new file mode 100644 index 0000000..c3009be --- /dev/null +++ b/TST_sti2d/DM/2102_DM1/corr_04_2102_DM1.tex @@ -0,0 +1,121 @@ +\documentclass[a4paper,10pt]{article} +\usepackage{myXsim} +\usepackage{tasks} + +% Title Page +\title{DM1 \hfill BUDIN Nathan} +\tribe{TST sti2d} +\date{\hfillÀ render pour le jeudi 25 février} + +\xsimsetup{ + solution/print = true +} + +\begin{document} +\maketitle + +\begin{exercise}[subtitle={Complexes}] + \begin{enumerate} + \item Mettre le nombre complexe suivant sous forme algébrique $z_1 = \dfrac{7 + 2 i}{-2 + 3 i} $ + \item Mettre le complexe suivante sous forme exponentielle $z_2 = - 6 \sqrt{3} - 6 i$ + \item Mettre le complexe suivante sous forme exponentielle $z_3 = - 4 \sqrt{3} + 4 i$ + \item Calculer le produit $z_4=z_2\times z_3$ donner le résultat sous forme exponentielle puis algébrique. + \item Calculer le quotient $z_5=\frac{z_2}{z_3}$ donner le résultat sous forme exponentielle puis algébrique. + \end{enumerate} +\end{exercise} + +\begin{solution} + \begin{enumerate} + \item $z_1 = - \frac{8}{13} - \frac{25 i}{13}$ + \item $z_3 = 12 e^{- \frac{5 i \pi}{6}}$ + \item $z_4 = 96 e^{0} = 96 = 96.0$ + \item $z_5 = \frac{3}{2} e^{- \frac{5 i \pi}{3}} = \frac{3}{4} + \frac{3 \sqrt{3} i}{4} = 0.75 + 1.3 i$ + \end{enumerate} +\end{solution} + +\begin{exercise}[subtitle={Bassin}] + Le tour d'un bassin au niveau du sol présente deux axes de symétrie : l’axe des abscisses et la droite d’équation $x=4$. Il est obtenu par symétrie de la courbe $\mathcal{C_f}$ sur $\intFF{0}{4}$ où $f$ est la fonction définie par + + \[ + f(x) = \left(- x^{2} + 4.9 x - 6.0\right) e^{- x} + 6.0 + \] + On admet que sur $\intFF{0}{4}$ la fonction $f$ est positive. + \begin{enumerate} + \item Sur un repère, tracer l'allure de la courbe $\mathcal{C}_f$, les axes de symétries puis compléter pour dessiner la forme du bassin. + \item Montrer que la fonction $f$ admet comme primitive sur $\R$ la fonction $F$ définie par + \[ + F(x) = 6.0 x + \left( x^{2} - 2.9 x + 3.1\right) e^{- x} + \] + \item Calculer la quantité $\ds \int_0^4 f(x) \; dx$, vous donnerez le résultat sous forme exacte. Interpréter le résultat et reportez cette quantité sur le graphique. + \item On considère que l'échelle de votre graphique est de 1unité pour 15m. Calculer l'aire du bassin. Vous donnerez un résultat arrondi au $m^2$ près. + \end{enumerate} +\end{exercise} + +\begin{solution} + \begin{enumerate} + \item + \begin{tikzpicture}[baseline=(a.north), xscale=1, yscale=0.5] + \tkzInit[xmin=0,xmax=5,xstep=1, + ymin=0,ymax=10,ystep=1] + \tkzGrid + \tkzAxeXY + \tkzFct[domain=0:10,color=red,very thick]% + { (-x**2 + 4.9*x - 6.0)*exp(-x) + 6.0 }; + \end{tikzpicture} + \item Il faut dériver $F(x)$ et vérifier que $F'(x) = f(x)$. + \item $\ds \int_0^4 f(x) \; dx = F(4) - F(0) = \frac{7.5}{e^{4}} + 20.9$ + \item La quantité calculée à la question précédente se retrouve 4fois pour former le bassin. Il faut ensuite prendre en compte l'échelle, comme 1unité de longueur correspond à 15m, une unité d'air correspond à $15\times15 = 225m^2$. Ainsi l'aire du bassin est égale à + \[ + (\frac{7.5}{e^{4}} + 20.9)\times 4 \times 15^2 = 18934.00000 + \] + + \end{enumerate} +\end{solution} + +\begin{exercise}[subtitle={Bassin}] + Le clinker est un constituant du ciment qui résulte de la cuisson d'un mélange composé de calcaire et d'argile. La fabrication du clinker nécessite des fours à très haute température qui libèrent dans l'air une grande quantité de dioxyde de carbone (CO$_2$). + + Dans une cimenterie, la fabrication du clinker s'effectue de 7 h 30 à 20 h, dans une pièce de volume \np{400000}~dm$^3$. + + À 20 h, après une journée de travail, le taux volumique de CO$_2$ dans la pièce est de 0.6\,\%. + \begin{enumerate} + \item Justifier que le volume de CO$_2$ présent dans cette pièce à 20 h est de \np{2400}~dm$^3$ . + \item On modélise le volume de CO$_2$ présent dans la pièce par une fonction du temps $t$ écoulé après 20h (exprimé en minutes) qui pour formule $V(t) = V_0e^{-0.0t} + 580$ + \begin{enumerate} + \item Démontrer que $V_0$ est égale à \np{1820}. + \item Quel sera, au dm$^3$ près, le volume de CO$_2$ dans cette pièce à 22 h ? + \item Démontrer que $V'(t) = 0$. + \item Étudier le signe de $V'(t)$ puis en déduire le sens de variation de $V(t)$. + \item Que peut-on dire du volume de CO$_2$ quand $t$ devient grand? + \end{enumerate} + \end{enumerate} +\end{exercise} + +\begin{solution} + \begin{enumerate} + \item Volume à 20h: $400000\times 0.006 = 2400$ + \item + \begin{enumerate} + \item $t=0$ correspond à 20h. + + Donc $V(0) = 2400 = V_0e^{-0.0\times 0} + 580 = V_0 + 580$ + + Donc $V_0 = 2400 - 580 = 1820$ + \item Il faut calculer $V(t)$ pour $t = 2$ donc + \[ + V(2) = 2400 + \] + \item Pas de correction pour cette question. + \item Pas de correction pour cette question. + \item Pas de correction pour cette question. + \end{enumerate} + \end{enumerate} +\end{solution} + + +\end{document} + +%%% Local Variables: +%%% mode: latex +%%% TeX-master: "master" +%%% End: diff --git a/TST_sti2d/DM/2102_DM1/corr_05_2102_DM1.tex b/TST_sti2d/DM/2102_DM1/corr_05_2102_DM1.tex new file mode 100644 index 0000000..eac5203 --- /dev/null +++ b/TST_sti2d/DM/2102_DM1/corr_05_2102_DM1.tex @@ -0,0 +1,121 @@ +\documentclass[a4paper,10pt]{article} +\usepackage{myXsim} +\usepackage{tasks} + +% Title Page +\title{DM1 \hfill CHION Léa} +\tribe{TST sti2d} +\date{\hfillÀ render pour le jeudi 25 février} + +\xsimsetup{ + solution/print = true +} + +\begin{document} +\maketitle + +\begin{exercise}[subtitle={Complexes}] + \begin{enumerate} + \item Mettre le nombre complexe suivant sous forme algébrique $z_1 = \dfrac{9 + 6 i}{-3 + 5 i} $ + \item Mettre le complexe suivante sous forme exponentielle $z_2 = -7 + 7 \sqrt{3} i$ + \item Mettre le complexe suivante sous forme exponentielle $z_3 = -5 - 5 \sqrt{3} i$ + \item Calculer le produit $z_4=z_2\times z_3$ donner le résultat sous forme exponentielle puis algébrique. + \item Calculer le quotient $z_5=\frac{z_2}{z_3}$ donner le résultat sous forme exponentielle puis algébrique. + \end{enumerate} +\end{exercise} + +\begin{solution} + \begin{enumerate} + \item $z_1 = \frac{3}{34} - \frac{63 i}{34}$ + \item $z_3 = 14 e^{\frac{2 i \pi}{3}}$ + \item $z_4 = 140 e^{0} = 140 = 140.0$ + \item $z_5 = \frac{7}{5} e^{\frac{4 i \pi}{3}} = - \frac{7}{10} - \frac{7 \sqrt{3} i}{10} = -0.7 - 1.21 i$ + \end{enumerate} +\end{solution} + +\begin{exercise}[subtitle={Bassin}] + Le tour d'un bassin au niveau du sol présente deux axes de symétrie : l’axe des abscisses et la droite d’équation $x=4$. Il est obtenu par symétrie de la courbe $\mathcal{C_f}$ sur $\intFF{0}{4}$ où $f$ est la fonction définie par + + \[ + f(x) = \left(- x^{2} + 6.2 x - 7.3\right) e^{- x} + 7.3 + \] + On admet que sur $\intFF{0}{4}$ la fonction $f$ est positive. + \begin{enumerate} + \item Sur un repère, tracer l'allure de la courbe $\mathcal{C}_f$, les axes de symétries puis compléter pour dessiner la forme du bassin. + \item Montrer que la fonction $f$ admet comme primitive sur $\R$ la fonction $F$ définie par + \[ + F(x) = 7.3 x + \left( x^{2} - 4.2 x + 3.1\right) e^{- x} + \] + \item Calculer la quantité $\ds \int_0^4 f(x) \; dx$, vous donnerez le résultat sous forme exacte. Interpréter le résultat et reportez cette quantité sur le graphique. + \item On considère que l'échelle de votre graphique est de 1unité pour 15m. Calculer l'aire du bassin. Vous donnerez un résultat arrondi au $m^2$ près. + \end{enumerate} +\end{exercise} + +\begin{solution} + \begin{enumerate} + \item + \begin{tikzpicture}[baseline=(a.north), xscale=1, yscale=0.5] + \tkzInit[xmin=0,xmax=5,xstep=1, + ymin=0,ymax=10,ystep=1] + \tkzGrid + \tkzAxeXY + \tkzFct[domain=0:10,color=red,very thick]% + { (-x**2 + 6.2*x - 7.3)*exp(-x) + 7.3 }; + \end{tikzpicture} + \item Il faut dériver $F(x)$ et vérifier que $F'(x) = f(x)$. + \item $\ds \int_0^4 f(x) \; dx = F(4) - F(0) = \frac{2.3}{e^{4}} + 26.1$ + \item La quantité calculée à la question précédente se retrouve 4fois pour former le bassin. Il faut ensuite prendre en compte l'échelle, comme 1unité de longueur correspond à 15m, une unité d'air correspond à $15\times15 = 225m^2$. Ainsi l'aire du bassin est égale à + \[ + (\frac{2.3}{e^{4}} + 26.1)\times 4 \times 15^2 = 23528.00000 + \] + + \end{enumerate} +\end{solution} + +\begin{exercise}[subtitle={Bassin}] + Le clinker est un constituant du ciment qui résulte de la cuisson d'un mélange composé de calcaire et d'argile. La fabrication du clinker nécessite des fours à très haute température qui libèrent dans l'air une grande quantité de dioxyde de carbone (CO$_2$). + + Dans une cimenterie, la fabrication du clinker s'effectue de 7 h 30 à 20 h, dans une pièce de volume \np{300000}~dm$^3$. + + À 20 h, après une journée de travail, le taux volumique de CO$_2$ dans la pièce est de 0.6\,\%. + \begin{enumerate} + \item Justifier que le volume de CO$_2$ présent dans cette pièce à 20 h est de \np{1800}~dm$^3$ . + \item On modélise le volume de CO$_2$ présent dans la pièce par une fonction du temps $t$ écoulé après 20h (exprimé en minutes) qui pour formule $V(t) = V_0e^{-0.01t} + 260$ + \begin{enumerate} + \item Démontrer que $V_0$ est égale à \np{1540}. + \item Quel sera, au dm$^3$ près, le volume de CO$_2$ dans cette pièce à 22 h ? + \item Démontrer que $V'(t) = - 15.4 e^{- 0.01 t}$. + \item Étudier le signe de $V'(t)$ puis en déduire le sens de variation de $V(t)$. + \item Que peut-on dire du volume de CO$_2$ quand $t$ devient grand? + \end{enumerate} + \end{enumerate} +\end{exercise} + +\begin{solution} + \begin{enumerate} + \item Volume à 20h: $300000\times 0.006 = 1800$ + \item + \begin{enumerate} + \item $t=0$ correspond à 20h. + + Donc $V(0) = 1800 = V_0e^{-0.01\times 0} + 260 = V_0 + 260$ + + Donc $V_0 = 1800 - 260 = 1540$ + \item Il faut calculer $V(t)$ pour $t = 2$ donc + \[ + V(2) = 1769.51 + \] + \item Pas de correction pour cette question. + \item Pas de correction pour cette question. + \item Pas de correction pour cette question. + \end{enumerate} + \end{enumerate} +\end{solution} + + +\end{document} + +%%% Local Variables: +%%% mode: latex +%%% TeX-master: "master" +%%% End: diff --git a/TST_sti2d/DM/2102_DM1/corr_06_2102_DM1.tex b/TST_sti2d/DM/2102_DM1/corr_06_2102_DM1.tex new file mode 100644 index 0000000..8ed27b8 --- /dev/null +++ b/TST_sti2d/DM/2102_DM1/corr_06_2102_DM1.tex @@ -0,0 +1,121 @@ +\documentclass[a4paper,10pt]{article} +\usepackage{myXsim} +\usepackage{tasks} + +% Title Page +\title{DM1 \hfill CLAIN Avinash} +\tribe{TST sti2d} +\date{\hfillÀ render pour le jeudi 25 février} + +\xsimsetup{ + solution/print = true +} + +\begin{document} +\maketitle + +\begin{exercise}[subtitle={Complexes}] + \begin{enumerate} + \item Mettre le nombre complexe suivant sous forme algébrique $z_1 = \dfrac{4 + 10 i}{-8 + 3 i} $ + \item Mettre le complexe suivante sous forme exponentielle $z_2 = - 4 \sqrt{3} + 4 i$ + \item Mettre le complexe suivante sous forme exponentielle $z_3 = 2 \sqrt{2} - 2 \sqrt{2} i$ + \item Calculer le produit $z_4=z_2\times z_3$ donner le résultat sous forme exponentielle puis algébrique. + \item Calculer le quotient $z_5=\frac{z_2}{z_3}$ donner le résultat sous forme exponentielle puis algébrique. + \end{enumerate} +\end{exercise} + +\begin{solution} + \begin{enumerate} + \item $z_1 = - \frac{2}{73} - \frac{92 i}{73}$ + \item $z_3 = 8 e^{\frac{5 i \pi}{6}}$ + \item $z_4 = 32 e^{\frac{7 i \pi}{12}} = - 8 \sqrt{6} + 8 \sqrt{2} + i \left(8 \sqrt{2} + 8 \sqrt{6}\right) = -8.28 + 30.9 i$ + \item $z_5 = 2 e^{\frac{13 i \pi}{12}} = - \frac{\sqrt{6}}{2} - \frac{\sqrt{2}}{2} + i \left(- \frac{\sqrt{6}}{2} + \frac{\sqrt{2}}{2}\right) = -1.93 - 0.518 i$ + \end{enumerate} +\end{solution} + +\begin{exercise}[subtitle={Bassin}] + Le tour d'un bassin au niveau du sol présente deux axes de symétrie : l’axe des abscisses et la droite d’équation $x=4$. Il est obtenu par symétrie de la courbe $\mathcal{C_f}$ sur $\intFF{0}{4}$ où $f$ est la fonction définie par + + \[ + f(x) = \left(- x^{2} + 1.8 x - 4.9\right) e^{- x} + 4.9 + \] + On admet que sur $\intFF{0}{4}$ la fonction $f$ est positive. + \begin{enumerate} + \item Sur un repère, tracer l'allure de la courbe $\mathcal{C}_f$, les axes de symétries puis compléter pour dessiner la forme du bassin. + \item Montrer que la fonction $f$ admet comme primitive sur $\R$ la fonction $F$ définie par + \[ + F(x) = 4.9 x + \left( x^{2} + 0.2 x + 5.1\right) e^{- x} + \] + \item Calculer la quantité $\ds \int_0^4 f(x) \; dx$, vous donnerez le résultat sous forme exacte. Interpréter le résultat et reportez cette quantité sur le graphique. + \item On considère que l'échelle de votre graphique est de 1unité pour 15m. Calculer l'aire du bassin. Vous donnerez un résultat arrondi au $m^2$ près. + \end{enumerate} +\end{exercise} + +\begin{solution} + \begin{enumerate} + \item + \begin{tikzpicture}[baseline=(a.north), xscale=1, yscale=0.5] + \tkzInit[xmin=0,xmax=5,xstep=1, + ymin=0,ymax=10,ystep=1] + \tkzGrid + \tkzAxeXY + \tkzFct[domain=0:10,color=red,very thick]% + { (-x**2 + 1.8*x - 4.9)*exp(-x) + 4.9 }; + \end{tikzpicture} + \item Il faut dériver $F(x)$ et vérifier que $F'(x) = f(x)$. + \item $\ds \int_0^4 f(x) \; dx = F(4) - F(0) = \frac{21.9}{e^{4}} + 14.5$ + \item La quantité calculée à la question précédente se retrouve 4fois pour former le bassin. Il faut ensuite prendre en compte l'échelle, comme 1unité de longueur correspond à 15m, une unité d'air correspond à $15\times15 = 225m^2$. Ainsi l'aire du bassin est égale à + \[ + (\frac{21.9}{e^{4}} + 14.5)\times 4 \times 15^2 = 13411.00000 + \] + + \end{enumerate} +\end{solution} + +\begin{exercise}[subtitle={Bassin}] + Le clinker est un constituant du ciment qui résulte de la cuisson d'un mélange composé de calcaire et d'argile. La fabrication du clinker nécessite des fours à très haute température qui libèrent dans l'air une grande quantité de dioxyde de carbone (CO$_2$). + + Dans une cimenterie, la fabrication du clinker s'effectue de 7 h 30 à 20 h, dans une pièce de volume \np{1000000}~dm$^3$. + + À 20 h, après une journée de travail, le taux volumique de CO$_2$ dans la pièce est de 0.8\,\%. + \begin{enumerate} + \item Justifier que le volume de CO$_2$ présent dans cette pièce à 20 h est de \np{8000}~dm$^3$ . + \item On modélise le volume de CO$_2$ présent dans la pièce par une fonction du temps $t$ écoulé après 20h (exprimé en minutes) qui pour formule $V(t) = V_0e^{-0.09t} + 580$ + \begin{enumerate} + \item Démontrer que $V_0$ est égale à \np{7420}. + \item Quel sera, au dm$^3$ près, le volume de CO$_2$ dans cette pièce à 21 h ? + \item Démontrer que $V'(t) = - 667.8 e^{- 0.09 t}$. + \item Étudier le signe de $V'(t)$ puis en déduire le sens de variation de $V(t)$. + \item Que peut-on dire du volume de CO$_2$ quand $t$ devient grand? + \end{enumerate} + \end{enumerate} +\end{exercise} + +\begin{solution} + \begin{enumerate} + \item Volume à 20h: $1000000\times 0.008 = 8000$ + \item + \begin{enumerate} + \item $t=0$ correspond à 20h. + + Donc $V(0) = 8000 = V_0e^{-0.09\times 0} + 580 = V_0 + 580$ + + Donc $V_0 = 8000 - 580 = 7420$ + \item Il faut calculer $V(t)$ pour $t = 1$ donc + \[ + V(1) = 7361.37 + \] + \item Pas de correction pour cette question. + \item Pas de correction pour cette question. + \item Pas de correction pour cette question. + \end{enumerate} + \end{enumerate} +\end{solution} + + +\end{document} + +%%% Local Variables: +%%% mode: latex +%%% TeX-master: "master" +%%% End: diff --git a/TST_sti2d/DM/2102_DM1/corr_07_2102_DM1.tex b/TST_sti2d/DM/2102_DM1/corr_07_2102_DM1.tex new file mode 100644 index 0000000..4844c48 --- /dev/null +++ b/TST_sti2d/DM/2102_DM1/corr_07_2102_DM1.tex @@ -0,0 +1,121 @@ +\documentclass[a4paper,10pt]{article} +\usepackage{myXsim} +\usepackage{tasks} + +% Title Page +\title{DM1 \hfill COUBAT Alexis} +\tribe{TST sti2d} +\date{\hfillÀ render pour le jeudi 25 février} + +\xsimsetup{ + solution/print = true +} + +\begin{document} +\maketitle + +\begin{exercise}[subtitle={Complexes}] + \begin{enumerate} + \item Mettre le nombre complexe suivant sous forme algébrique $z_1 = \dfrac{7 + 3 i}{-2 + 2 i} $ + \item Mettre le complexe suivante sous forme exponentielle $z_2 = 1 - \sqrt{3} i$ + \item Mettre le complexe suivante sous forme exponentielle $z_3 = - 2 \sqrt{3} + 2 i$ + \item Calculer le produit $z_4=z_2\times z_3$ donner le résultat sous forme exponentielle puis algébrique. + \item Calculer le quotient $z_5=\frac{z_2}{z_3}$ donner le résultat sous forme exponentielle puis algébrique. + \end{enumerate} +\end{exercise} + +\begin{solution} + \begin{enumerate} + \item $z_1 = -1 - \frac{5 i}{2}$ + \item $z_3 = 2 e^{- \frac{i \pi}{3}}$ + \item $z_4 = 8 e^{\frac{i \pi}{2}} = 8 i = 8.0 i$ + \item $z_5 = \frac{1}{2} e^{- \frac{7 i \pi}{6}} = - \frac{\sqrt{3}}{4} + \frac{i}{4} = -0.433 + 0.25 i$ + \end{enumerate} +\end{solution} + +\begin{exercise}[subtitle={Bassin}] + Le tour d'un bassin au niveau du sol présente deux axes de symétrie : l’axe des abscisses et la droite d’équation $x=4$. Il est obtenu par symétrie de la courbe $\mathcal{C_f}$ sur $\intFF{0}{4}$ où $f$ est la fonction définie par + + \[ + f(x) = \left(- x^{2} + 5.6 x - 4.5\right) e^{- x} + 4.5 + \] + On admet que sur $\intFF{0}{4}$ la fonction $f$ est positive. + \begin{enumerate} + \item Sur un repère, tracer l'allure de la courbe $\mathcal{C}_f$, les axes de symétries puis compléter pour dessiner la forme du bassin. + \item Montrer que la fonction $f$ admet comme primitive sur $\R$ la fonction $F$ définie par + \[ + F(x) = 4.5 x + \left( x^{2} - 3.6 x + 0.9\right) e^{- x} + \] + \item Calculer la quantité $\ds \int_0^4 f(x) \; dx$, vous donnerez le résultat sous forme exacte. Interpréter le résultat et reportez cette quantité sur le graphique. + \item On considère que l'échelle de votre graphique est de 1unité pour 15m. Calculer l'aire du bassin. Vous donnerez un résultat arrondi au $m^2$ près. + \end{enumerate} +\end{exercise} + +\begin{solution} + \begin{enumerate} + \item + \begin{tikzpicture}[baseline=(a.north), xscale=1, yscale=0.5] + \tkzInit[xmin=0,xmax=5,xstep=1, + ymin=0,ymax=10,ystep=1] + \tkzGrid + \tkzAxeXY + \tkzFct[domain=0:10,color=red,very thick]% + { (-x**2 + 5.6*x - 4.5)*exp(-x) + 4.5 }; + \end{tikzpicture} + \item Il faut dériver $F(x)$ et vérifier que $F'(x) = f(x)$. + \item $\ds \int_0^4 f(x) \; dx = F(4) - F(0) = \frac{2.5}{e^{4}} + 17.1$ + \item La quantité calculée à la question précédente se retrouve 4fois pour former le bassin. Il faut ensuite prendre en compte l'échelle, comme 1unité de longueur correspond à 15m, une unité d'air correspond à $15\times15 = 225m^2$. Ainsi l'aire du bassin est égale à + \[ + (\frac{2.5}{e^{4}} + 17.1)\times 4 \times 15^2 = 15431.00000 + \] + + \end{enumerate} +\end{solution} + +\begin{exercise}[subtitle={Bassin}] + Le clinker est un constituant du ciment qui résulte de la cuisson d'un mélange composé de calcaire et d'argile. La fabrication du clinker nécessite des fours à très haute température qui libèrent dans l'air une grande quantité de dioxyde de carbone (CO$_2$). + + Dans une cimenterie, la fabrication du clinker s'effectue de 7 h 30 à 20 h, dans une pièce de volume \np{800000}~dm$^3$. + + À 20 h, après une journée de travail, le taux volumique de CO$_2$ dans la pièce est de 0.7\,\%. + \begin{enumerate} + \item Justifier que le volume de CO$_2$ présent dans cette pièce à 20 h est de \np{5600}~dm$^3$ . + \item On modélise le volume de CO$_2$ présent dans la pièce par une fonction du temps $t$ écoulé après 20h (exprimé en minutes) qui pour formule $V(t) = V_0e^{-0.02t} + 340$ + \begin{enumerate} + \item Démontrer que $V_0$ est égale à \np{5260}. + \item Quel sera, au dm$^3$ près, le volume de CO$_2$ dans cette pièce à 24 h ? + \item Démontrer que $V'(t) = - 105.2 e^{- 0.02 t}$. + \item Étudier le signe de $V'(t)$ puis en déduire le sens de variation de $V(t)$. + \item Que peut-on dire du volume de CO$_2$ quand $t$ devient grand? + \end{enumerate} + \end{enumerate} +\end{exercise} + +\begin{solution} + \begin{enumerate} + \item Volume à 20h: $800000\times 0.006999999999999999 = 5600$ + \item + \begin{enumerate} + \item $t=0$ correspond à 20h. + + Donc $V(0) = 5600 = V_0e^{-0.02\times 0} + 340 = V_0 + 340$ + + Donc $V_0 = 5600 - 340 = 5260$ + \item Il faut calculer $V(t)$ pour $t = 4$ donc + \[ + V(4) = 5195.59 + \] + \item Pas de correction pour cette question. + \item Pas de correction pour cette question. + \item Pas de correction pour cette question. + \end{enumerate} + \end{enumerate} +\end{solution} + + +\end{document} + +%%% Local Variables: +%%% mode: latex +%%% TeX-master: "master" +%%% End: diff --git a/TST_sti2d/DM/2102_DM1/corr_08_2102_DM1.tex b/TST_sti2d/DM/2102_DM1/corr_08_2102_DM1.tex new file mode 100644 index 0000000..c063bc3 --- /dev/null +++ b/TST_sti2d/DM/2102_DM1/corr_08_2102_DM1.tex @@ -0,0 +1,121 @@ +\documentclass[a4paper,10pt]{article} +\usepackage{myXsim} +\usepackage{tasks} + +% Title Page +\title{DM1 \hfill EVRARD Jules} +\tribe{TST sti2d} +\date{\hfillÀ render pour le jeudi 25 février} + +\xsimsetup{ + solution/print = true +} + +\begin{document} +\maketitle + +\begin{exercise}[subtitle={Complexes}] + \begin{enumerate} + \item Mettre le nombre complexe suivant sous forme algébrique $z_1 = \dfrac{8 + 3 i}{-9 + 3 i} $ + \item Mettre le complexe suivante sous forme exponentielle $z_2 = 3 - 3 \sqrt{3} i$ + \item Mettre le complexe suivante sous forme exponentielle $z_3 = \sqrt{2} - \sqrt{2} i$ + \item Calculer le produit $z_4=z_2\times z_3$ donner le résultat sous forme exponentielle puis algébrique. + \item Calculer le quotient $z_5=\frac{z_2}{z_3}$ donner le résultat sous forme exponentielle puis algébrique. + \end{enumerate} +\end{exercise} + +\begin{solution} + \begin{enumerate} + \item $z_1 = - \frac{7}{10} - \frac{17 i}{30}$ + \item $z_3 = 6 e^{- \frac{i \pi}{3}}$ + \item $z_4 = 12 e^{- \frac{7 i \pi}{12}} = - 3 \sqrt{6} + 3 \sqrt{2} + i \left(- 3 \sqrt{6} - 3 \sqrt{2}\right) = -3.11 - 11.6 i$ + \item $z_5 = 3 e^{- \frac{i \pi}{12}} = \frac{3 \sqrt{2}}{4} + \frac{3 \sqrt{6}}{4} + i \left(- \frac{3 \sqrt{6}}{4} + \frac{3 \sqrt{2}}{4}\right) = 2.9 - 0.776 i$ + \end{enumerate} +\end{solution} + +\begin{exercise}[subtitle={Bassin}] + Le tour d'un bassin au niveau du sol présente deux axes de symétrie : l’axe des abscisses et la droite d’équation $x=4$. Il est obtenu par symétrie de la courbe $\mathcal{C_f}$ sur $\intFF{0}{4}$ où $f$ est la fonction définie par + + \[ + f(x) = \left(- x^{2} + 0.9 x - 3.2\right) e^{- x} + 3.2 + \] + On admet que sur $\intFF{0}{4}$ la fonction $f$ est positive. + \begin{enumerate} + \item Sur un repère, tracer l'allure de la courbe $\mathcal{C}_f$, les axes de symétries puis compléter pour dessiner la forme du bassin. + \item Montrer que la fonction $f$ admet comme primitive sur $\R$ la fonction $F$ définie par + \[ + F(x) = 3.2 x + \left( x^{2} + 1.1 x + 4.3\right) e^{- x} + \] + \item Calculer la quantité $\ds \int_0^4 f(x) \; dx$, vous donnerez le résultat sous forme exacte. Interpréter le résultat et reportez cette quantité sur le graphique. + \item On considère que l'échelle de votre graphique est de 1unité pour 15m. Calculer l'aire du bassin. Vous donnerez un résultat arrondi au $m^2$ près. + \end{enumerate} +\end{exercise} + +\begin{solution} + \begin{enumerate} + \item + \begin{tikzpicture}[baseline=(a.north), xscale=1, yscale=0.5] + \tkzInit[xmin=0,xmax=5,xstep=1, + ymin=0,ymax=10,ystep=1] + \tkzGrid + \tkzAxeXY + \tkzFct[domain=0:10,color=red,very thick]% + { (-x**2 + 0.9*x - 3.2)*exp(-x) + 3.2 }; + \end{tikzpicture} + \item Il faut dériver $F(x)$ et vérifier que $F'(x) = f(x)$. + \item $\ds \int_0^4 f(x) \; dx = F(4) - F(0) = \frac{24.7}{e^{4}} + 8.5$ + \item La quantité calculée à la question précédente se retrouve 4fois pour former le bassin. Il faut ensuite prendre en compte l'échelle, comme 1unité de longueur correspond à 15m, une unité d'air correspond à $15\times15 = 225m^2$. Ainsi l'aire du bassin est égale à + \[ + (\frac{24.7}{e^{4}} + 8.5)\times 4 \times 15^2 = 8057.000000 + \] + + \end{enumerate} +\end{solution} + +\begin{exercise}[subtitle={Bassin}] + Le clinker est un constituant du ciment qui résulte de la cuisson d'un mélange composé de calcaire et d'argile. La fabrication du clinker nécessite des fours à très haute température qui libèrent dans l'air une grande quantité de dioxyde de carbone (CO$_2$). + + Dans une cimenterie, la fabrication du clinker s'effectue de 7 h 30 à 20 h, dans une pièce de volume \np{800000}~dm$^3$. + + À 20 h, après une journée de travail, le taux volumique de CO$_2$ dans la pièce est de 1.0\,\%. + \begin{enumerate} + \item Justifier que le volume de CO$_2$ présent dans cette pièce à 20 h est de \np{8000}~dm$^3$ . + \item On modélise le volume de CO$_2$ présent dans la pièce par une fonction du temps $t$ écoulé après 20h (exprimé en minutes) qui pour formule $V(t) = V_0e^{-0.02t} + 560$ + \begin{enumerate} + \item Démontrer que $V_0$ est égale à \np{7440}. + \item Quel sera, au dm$^3$ près, le volume de CO$_2$ dans cette pièce à 23 h ? + \item Démontrer que $V'(t) = - 148.8 e^{- 0.02 t}$. + \item Étudier le signe de $V'(t)$ puis en déduire le sens de variation de $V(t)$. + \item Que peut-on dire du volume de CO$_2$ quand $t$ devient grand? + \end{enumerate} + \end{enumerate} +\end{exercise} + +\begin{solution} + \begin{enumerate} + \item Volume à 20h: $800000\times 0.01 = 8000$ + \item + \begin{enumerate} + \item $t=0$ correspond à 20h. + + Donc $V(0) = 8000 = V_0e^{-0.02\times 0} + 560 = V_0 + 560$ + + Donc $V_0 = 8000 - 560 = 7440$ + \item Il faut calculer $V(t)$ pour $t = 3$ donc + \[ + V(3) = 7566.73 + \] + \item Pas de correction pour cette question. + \item Pas de correction pour cette question. + \item Pas de correction pour cette question. + \end{enumerate} + \end{enumerate} +\end{solution} + + +\end{document} + +%%% Local Variables: +%%% mode: latex +%%% TeX-master: "master" +%%% End: diff --git a/TST_sti2d/DM/2102_DM1/corr_09_2102_DM1.tex b/TST_sti2d/DM/2102_DM1/corr_09_2102_DM1.tex new file mode 100644 index 0000000..bf045e6 --- /dev/null +++ b/TST_sti2d/DM/2102_DM1/corr_09_2102_DM1.tex @@ -0,0 +1,121 @@ +\documentclass[a4paper,10pt]{article} +\usepackage{myXsim} +\usepackage{tasks} + +% Title Page +\title{DM1 \hfill HADJRAS Mohcine} +\tribe{TST sti2d} +\date{\hfillÀ render pour le jeudi 25 février} + +\xsimsetup{ + solution/print = true +} + +\begin{document} +\maketitle + +\begin{exercise}[subtitle={Complexes}] + \begin{enumerate} + \item Mettre le nombre complexe suivant sous forme algébrique $z_1 = \dfrac{5 + 3 i}{-5 + 7 i} $ + \item Mettre le complexe suivante sous forme exponentielle $z_2 = 2 \sqrt{3} + 2 i$ + \item Mettre le complexe suivante sous forme exponentielle $z_3 = -10 - 10 \sqrt{3} i$ + \item Calculer le produit $z_4=z_2\times z_3$ donner le résultat sous forme exponentielle puis algébrique. + \item Calculer le quotient $z_5=\frac{z_2}{z_3}$ donner le résultat sous forme exponentielle puis algébrique. + \end{enumerate} +\end{exercise} + +\begin{solution} + \begin{enumerate} + \item $z_1 = - \frac{2}{37} - \frac{25 i}{37}$ + \item $z_3 = 4 e^{\frac{i \pi}{6}}$ + \item $z_4 = 80 e^{- \frac{i \pi}{2}} = - 80 i = - 80.0 i$ + \item $z_5 = \frac{1}{5} e^{\frac{5 i \pi}{6}} = - \frac{\sqrt{3}}{10} + \frac{i}{10} = -0.173 + 0.1 i$ + \end{enumerate} +\end{solution} + +\begin{exercise}[subtitle={Bassin}] + Le tour d'un bassin au niveau du sol présente deux axes de symétrie : l’axe des abscisses et la droite d’équation $x=4$. Il est obtenu par symétrie de la courbe $\mathcal{C_f}$ sur $\intFF{0}{4}$ où $f$ est la fonction définie par + + \[ + f(x) = \left(- x^{2} + 7.5 x - 9.0\right) e^{- x} + 9.0 + \] + On admet que sur $\intFF{0}{4}$ la fonction $f$ est positive. + \begin{enumerate} + \item Sur un repère, tracer l'allure de la courbe $\mathcal{C}_f$, les axes de symétries puis compléter pour dessiner la forme du bassin. + \item Montrer que la fonction $f$ admet comme primitive sur $\R$ la fonction $F$ définie par + \[ + F(x) = 9.0 x + \left( x^{2} - 5.5 x + 3.5\right) e^{- x} + \] + \item Calculer la quantité $\ds \int_0^4 f(x) \; dx$, vous donnerez le résultat sous forme exacte. Interpréter le résultat et reportez cette quantité sur le graphique. + \item On considère que l'échelle de votre graphique est de 1unité pour 15m. Calculer l'aire du bassin. Vous donnerez un résultat arrondi au $m^2$ près. + \end{enumerate} +\end{exercise} + +\begin{solution} + \begin{enumerate} + \item + \begin{tikzpicture}[baseline=(a.north), xscale=1, yscale=0.5] + \tkzInit[xmin=0,xmax=5,xstep=1, + ymin=0,ymax=10,ystep=1] + \tkzGrid + \tkzAxeXY + \tkzFct[domain=0:10,color=red,very thick]% + { (-x**2 + 7.5*x - 9.0)*exp(-x) + 9.0 }; + \end{tikzpicture} + \item Il faut dériver $F(x)$ et vérifier que $F'(x) = f(x)$. + \item $\ds \int_0^4 f(x) \; dx = F(4) - F(0) = 32.5 - \frac{2.5}{e^{4}}$ + \item La quantité calculée à la question précédente se retrouve 4fois pour former le bassin. Il faut ensuite prendre en compte l'échelle, comme 1unité de longueur correspond à 15m, une unité d'air correspond à $15\times15 = 225m^2$. Ainsi l'aire du bassin est égale à + \[ + (32.5 - \frac{2.5}{e^{4}})\times 4 \times 15^2 = 29209.00000 + \] + + \end{enumerate} +\end{solution} + +\begin{exercise}[subtitle={Bassin}] + Le clinker est un constituant du ciment qui résulte de la cuisson d'un mélange composé de calcaire et d'argile. La fabrication du clinker nécessite des fours à très haute température qui libèrent dans l'air une grande quantité de dioxyde de carbone (CO$_2$). + + Dans une cimenterie, la fabrication du clinker s'effectue de 7 h 30 à 20 h, dans une pièce de volume \np{1000000}~dm$^3$. + + À 20 h, après une journée de travail, le taux volumique de CO$_2$ dans la pièce est de 0.9\,\%. + \begin{enumerate} + \item Justifier que le volume de CO$_2$ présent dans cette pièce à 20 h est de \np{9000}~dm$^3$ . + \item On modélise le volume de CO$_2$ présent dans la pièce par une fonction du temps $t$ écoulé après 20h (exprimé en minutes) qui pour formule $V(t) = V_0e^{-0.01t} + 330$ + \begin{enumerate} + \item Démontrer que $V_0$ est égale à \np{8670}. + \item Quel sera, au dm$^3$ près, le volume de CO$_2$ dans cette pièce à 24 h ? + \item Démontrer que $V'(t) = - 86.7 e^{- 0.01 t}$. + \item Étudier le signe de $V'(t)$ puis en déduire le sens de variation de $V(t)$. + \item Que peut-on dire du volume de CO$_2$ quand $t$ devient grand? + \end{enumerate} + \end{enumerate} +\end{exercise} + +\begin{solution} + \begin{enumerate} + \item Volume à 20h: $1000000\times 0.009000000000000001 = 9000$ + \item + \begin{enumerate} + \item $t=0$ correspond à 20h. + + Donc $V(0) = 9000 = V_0e^{-0.01\times 0} + 330 = V_0 + 330$ + + Donc $V_0 = 9000 - 330 = 8670$ + \item Il faut calculer $V(t)$ pour $t = 4$ donc + \[ + V(4) = 8660.04 + \] + \item Pas de correction pour cette question. + \item Pas de correction pour cette question. + \item Pas de correction pour cette question. + \end{enumerate} + \end{enumerate} +\end{solution} + + +\end{document} + +%%% Local Variables: +%%% mode: latex +%%% TeX-master: "master" +%%% End: diff --git a/TST_sti2d/DM/2102_DM1/corr_10_2102_DM1.tex b/TST_sti2d/DM/2102_DM1/corr_10_2102_DM1.tex new file mode 100644 index 0000000..332b4c5 --- /dev/null +++ b/TST_sti2d/DM/2102_DM1/corr_10_2102_DM1.tex @@ -0,0 +1,121 @@ +\documentclass[a4paper,10pt]{article} +\usepackage{myXsim} +\usepackage{tasks} + +% Title Page +\title{DM1 \hfill HENRIST Maxime} +\tribe{TST sti2d} +\date{\hfillÀ render pour le jeudi 25 février} + +\xsimsetup{ + solution/print = true +} + +\begin{document} +\maketitle + +\begin{exercise}[subtitle={Complexes}] + \begin{enumerate} + \item Mettre le nombre complexe suivant sous forme algébrique $z_1 = \dfrac{6 + 6 i}{-2 + 7 i} $ + \item Mettre le complexe suivante sous forme exponentielle $z_2 = 1 - \sqrt{3} i$ + \item Mettre le complexe suivante sous forme exponentielle $z_3 = 3 \sqrt{2} + 3 \sqrt{2} i$ + \item Calculer le produit $z_4=z_2\times z_3$ donner le résultat sous forme exponentielle puis algébrique. + \item Calculer le quotient $z_5=\frac{z_2}{z_3}$ donner le résultat sous forme exponentielle puis algébrique. + \end{enumerate} +\end{exercise} + +\begin{solution} + \begin{enumerate} + \item $z_1 = \frac{30}{53} - \frac{54 i}{53}$ + \item $z_3 = 2 e^{- \frac{i \pi}{3}}$ + \item $z_4 = 12 e^{- \frac{i \pi}{12}} = 3 \sqrt{2} + 3 \sqrt{6} + i \left(- 3 \sqrt{6} + 3 \sqrt{2}\right) = 11.6 - 3.11 i$ + \item $z_5 = \frac{1}{3} e^{- \frac{7 i \pi}{12}} = - \frac{\sqrt{6}}{12} + \frac{\sqrt{2}}{12} + i \left(- \frac{\sqrt{6}}{12} - \frac{\sqrt{2}}{12}\right) = -0.0863 - 0.322 i$ + \end{enumerate} +\end{solution} + +\begin{exercise}[subtitle={Bassin}] + Le tour d'un bassin au niveau du sol présente deux axes de symétrie : l’axe des abscisses et la droite d’équation $x=4$. Il est obtenu par symétrie de la courbe $\mathcal{C_f}$ sur $\intFF{0}{4}$ où $f$ est la fonction définie par + + \[ + f(x) = \left(- x^{2} + 1.3 x - 8.0\right) e^{- x} + 8.0 + \] + On admet que sur $\intFF{0}{4}$ la fonction $f$ est positive. + \begin{enumerate} + \item Sur un repère, tracer l'allure de la courbe $\mathcal{C}_f$, les axes de symétries puis compléter pour dessiner la forme du bassin. + \item Montrer que la fonction $f$ admet comme primitive sur $\R$ la fonction $F$ définie par + \[ + F(x) = 8.0 x + \left( x^{2} + 0.7 x + 8.7\right) e^{- x} + \] + \item Calculer la quantité $\ds \int_0^4 f(x) \; dx$, vous donnerez le résultat sous forme exacte. Interpréter le résultat et reportez cette quantité sur le graphique. + \item On considère que l'échelle de votre graphique est de 1unité pour 15m. Calculer l'aire du bassin. Vous donnerez un résultat arrondi au $m^2$ près. + \end{enumerate} +\end{exercise} + +\begin{solution} + \begin{enumerate} + \item + \begin{tikzpicture}[baseline=(a.north), xscale=1, yscale=0.5] + \tkzInit[xmin=0,xmax=5,xstep=1, + ymin=0,ymax=10,ystep=1] + \tkzGrid + \tkzAxeXY + \tkzFct[domain=0:10,color=red,very thick]% + { (-x**2 + 1.3*x - 8.0)*exp(-x) + 8.0 }; + \end{tikzpicture} + \item Il faut dériver $F(x)$ et vérifier que $F'(x) = f(x)$. + \item $\ds \int_0^4 f(x) \; dx = F(4) - F(0) = \frac{27.5}{e^{4}} + 23.3$ + \item La quantité calculée à la question précédente se retrouve 4fois pour former le bassin. Il faut ensuite prendre en compte l'échelle, comme 1unité de longueur correspond à 15m, une unité d'air correspond à $15\times15 = 225m^2$. Ainsi l'aire du bassin est égale à + \[ + (\frac{27.5}{e^{4}} + 23.3)\times 4 \times 15^2 = 21423.00000 + \] + + \end{enumerate} +\end{solution} + +\begin{exercise}[subtitle={Bassin}] + Le clinker est un constituant du ciment qui résulte de la cuisson d'un mélange composé de calcaire et d'argile. La fabrication du clinker nécessite des fours à très haute température qui libèrent dans l'air une grande quantité de dioxyde de carbone (CO$_2$). + + Dans une cimenterie, la fabrication du clinker s'effectue de 7 h 30 à 20 h, dans une pièce de volume \np{900000}~dm$^3$. + + À 20 h, après une journée de travail, le taux volumique de CO$_2$ dans la pièce est de 0.7\,\%. + \begin{enumerate} + \item Justifier que le volume de CO$_2$ présent dans cette pièce à 20 h est de \np{6300}~dm$^3$ . + \item On modélise le volume de CO$_2$ présent dans la pièce par une fonction du temps $t$ écoulé après 20h (exprimé en minutes) qui pour formule $V(t) = V_0e^{-0.1t} + 390$ + \begin{enumerate} + \item Démontrer que $V_0$ est égale à \np{5910}. + \item Quel sera, au dm$^3$ près, le volume de CO$_2$ dans cette pièce à 22 h ? + \item Démontrer que $V'(t) = - 591.0 e^{- 0.1 t}$. + \item Étudier le signe de $V'(t)$ puis en déduire le sens de variation de $V(t)$. + \item Que peut-on dire du volume de CO$_2$ quand $t$ devient grand? + \end{enumerate} + \end{enumerate} +\end{exercise} + +\begin{solution} + \begin{enumerate} + \item Volume à 20h: $900000\times 0.006999999999999999 = 6300$ + \item + \begin{enumerate} + \item $t=0$ correspond à 20h. + + Donc $V(0) = 6300 = V_0e^{-0.1\times 0} + 390 = V_0 + 390$ + + Donc $V_0 = 6300 - 390 = 5910$ + \item Il faut calculer $V(t)$ pour $t = 2$ donc + \[ + V(2) = 5228.70 + \] + \item Pas de correction pour cette question. + \item Pas de correction pour cette question. + \item Pas de correction pour cette question. + \end{enumerate} + \end{enumerate} +\end{solution} + + +\end{document} + +%%% Local Variables: +%%% mode: latex +%%% TeX-master: "master" +%%% End: diff --git a/TST_sti2d/DM/2102_DM1/corr_11_2102_DM1.tex b/TST_sti2d/DM/2102_DM1/corr_11_2102_DM1.tex new file mode 100644 index 0000000..a9bebd3 --- /dev/null +++ b/TST_sti2d/DM/2102_DM1/corr_11_2102_DM1.tex @@ -0,0 +1,121 @@ +\documentclass[a4paper,10pt]{article} +\usepackage{myXsim} +\usepackage{tasks} + +% Title Page +\title{DM1 \hfill HUMBERT Rayan} +\tribe{TST sti2d} +\date{\hfillÀ render pour le jeudi 25 février} + +\xsimsetup{ + solution/print = true +} + +\begin{document} +\maketitle + +\begin{exercise}[subtitle={Complexes}] + \begin{enumerate} + \item Mettre le nombre complexe suivant sous forme algébrique $z_1 = \dfrac{5 + 10 i}{-5 + 2 i} $ + \item Mettre le complexe suivante sous forme exponentielle $z_2 = - 5 \sqrt{3} - 5 i$ + \item Mettre le complexe suivante sous forme exponentielle $z_3 = 9 - 9 \sqrt{3} i$ + \item Calculer le produit $z_4=z_2\times z_3$ donner le résultat sous forme exponentielle puis algébrique. + \item Calculer le quotient $z_5=\frac{z_2}{z_3}$ donner le résultat sous forme exponentielle puis algébrique. + \end{enumerate} +\end{exercise} + +\begin{solution} + \begin{enumerate} + \item $z_1 = - \frac{5}{29} - \frac{60 i}{29}$ + \item $z_3 = 10 e^{- \frac{5 i \pi}{6}}$ + \item $z_4 = 180 e^{- \frac{7 i \pi}{6}} = - 90 \sqrt{3} + 90 i = -156.0 + 90.0 i$ + \item $z_5 = \frac{5}{9} e^{- \frac{i \pi}{2}} = - \frac{5 i}{9} = - 0.556 i$ + \end{enumerate} +\end{solution} + +\begin{exercise}[subtitle={Bassin}] + Le tour d'un bassin au niveau du sol présente deux axes de symétrie : l’axe des abscisses et la droite d’équation $x=4$. Il est obtenu par symétrie de la courbe $\mathcal{C_f}$ sur $\intFF{0}{4}$ où $f$ est la fonction définie par + + \[ + f(x) = \left(- x^{2} + 0.3 x - 9.7\right) e^{- x} + 9.7 + \] + On admet que sur $\intFF{0}{4}$ la fonction $f$ est positive. + \begin{enumerate} + \item Sur un repère, tracer l'allure de la courbe $\mathcal{C}_f$, les axes de symétries puis compléter pour dessiner la forme du bassin. + \item Montrer que la fonction $f$ admet comme primitive sur $\R$ la fonction $F$ définie par + \[ + F(x) = 9.7 x + \left( x^{2} + 1.7 x + 11.4\right) e^{- x} + \] + \item Calculer la quantité $\ds \int_0^4 f(x) \; dx$, vous donnerez le résultat sous forme exacte. Interpréter le résultat et reportez cette quantité sur le graphique. + \item On considère que l'échelle de votre graphique est de 1unité pour 15m. Calculer l'aire du bassin. Vous donnerez un résultat arrondi au $m^2$ près. + \end{enumerate} +\end{exercise} + +\begin{solution} + \begin{enumerate} + \item + \begin{tikzpicture}[baseline=(a.north), xscale=1, yscale=0.5] + \tkzInit[xmin=0,xmax=5,xstep=1, + ymin=0,ymax=10,ystep=1] + \tkzGrid + \tkzAxeXY + \tkzFct[domain=0:10,color=red,very thick]% + { (-x**2 + 0.3*x - 9.7)*exp(-x) + 9.7 }; + \end{tikzpicture} + \item Il faut dériver $F(x)$ et vérifier que $F'(x) = f(x)$. + \item $\ds \int_0^4 f(x) \; dx = F(4) - F(0) = \frac{34.2}{e^{4}} + 27.4$ + \item La quantité calculée à la question précédente se retrouve 4fois pour former le bassin. Il faut ensuite prendre en compte l'échelle, comme 1unité de longueur correspond à 15m, une unité d'air correspond à $15\times15 = 225m^2$. Ainsi l'aire du bassin est égale à + \[ + (\frac{34.2}{e^{4}} + 27.4)\times 4 \times 15^2 = 25224.00000 + \] + + \end{enumerate} +\end{solution} + +\begin{exercise}[subtitle={Bassin}] + Le clinker est un constituant du ciment qui résulte de la cuisson d'un mélange composé de calcaire et d'argile. La fabrication du clinker nécessite des fours à très haute température qui libèrent dans l'air une grande quantité de dioxyde de carbone (CO$_2$). + + Dans une cimenterie, la fabrication du clinker s'effectue de 7 h 30 à 20 h, dans une pièce de volume \np{600000}~dm$^3$. + + À 20 h, après une journée de travail, le taux volumique de CO$_2$ dans la pièce est de 0.6\,\%. + \begin{enumerate} + \item Justifier que le volume de CO$_2$ présent dans cette pièce à 20 h est de \np{3600}~dm$^3$ . + \item On modélise le volume de CO$_2$ présent dans la pièce par une fonction du temps $t$ écoulé après 20h (exprimé en minutes) qui pour formule $V(t) = V_0e^{-0.06t} + 360$ + \begin{enumerate} + \item Démontrer que $V_0$ est égale à \np{3240}. + \item Quel sera, au dm$^3$ près, le volume de CO$_2$ dans cette pièce à 24 h ? + \item Démontrer que $V'(t) = - 194.4 e^{- 0.06 t}$. + \item Étudier le signe de $V'(t)$ puis en déduire le sens de variation de $V(t)$. + \item Que peut-on dire du volume de CO$_2$ quand $t$ devient grand? + \end{enumerate} + \end{enumerate} +\end{exercise} + +\begin{solution} + \begin{enumerate} + \item Volume à 20h: $600000\times 0.006 = 3600$ + \item + \begin{enumerate} + \item $t=0$ correspond à 20h. + + Donc $V(0) = 3600 = V_0e^{-0.06\times 0} + 360 = V_0 + 360$ + + Donc $V_0 = 3600 - 360 = 3240$ + \item Il faut calculer $V(t)$ pour $t = 4$ donc + \[ + V(4) = 2908.67 + \] + \item Pas de correction pour cette question. + \item Pas de correction pour cette question. + \item Pas de correction pour cette question. + \end{enumerate} + \end{enumerate} +\end{solution} + + +\end{document} + +%%% Local Variables: +%%% mode: latex +%%% TeX-master: "master" +%%% End: diff --git a/TST_sti2d/DM/2102_DM1/corr_12_2102_DM1.tex b/TST_sti2d/DM/2102_DM1/corr_12_2102_DM1.tex new file mode 100644 index 0000000..811ad75 --- /dev/null +++ b/TST_sti2d/DM/2102_DM1/corr_12_2102_DM1.tex @@ -0,0 +1,121 @@ +\documentclass[a4paper,10pt]{article} +\usepackage{myXsim} +\usepackage{tasks} + +% Title Page +\title{DM1 \hfill KILINC Suleyman} +\tribe{TST sti2d} +\date{\hfillÀ render pour le jeudi 25 février} + +\xsimsetup{ + solution/print = true +} + +\begin{document} +\maketitle + +\begin{exercise}[subtitle={Complexes}] + \begin{enumerate} + \item Mettre le nombre complexe suivant sous forme algébrique $z_1 = \dfrac{2 + 7 i}{-8 + 6 i} $ + \item Mettre le complexe suivante sous forme exponentielle $z_2 = \sqrt{2} + \sqrt{2} i$ + \item Mettre le complexe suivante sous forme exponentielle $z_3 = 8 \sqrt{3} - 8 i$ + \item Calculer le produit $z_4=z_2\times z_3$ donner le résultat sous forme exponentielle puis algébrique. + \item Calculer le quotient $z_5=\frac{z_2}{z_3}$ donner le résultat sous forme exponentielle puis algébrique. + \end{enumerate} +\end{exercise} + +\begin{solution} + \begin{enumerate} + \item $z_1 = \frac{13}{50} - \frac{17 i}{25}$ + \item $z_3 = 2 e^{\frac{i \pi}{4}}$ + \item $z_4 = 32 e^{\frac{i \pi}{12}} = 8 \sqrt{2} + 8 \sqrt{6} + i \left(- 8 \sqrt{2} + 8 \sqrt{6}\right) = 30.9 + 8.28 i$ + \item $z_5 = \frac{1}{8} e^{\frac{5 i \pi}{12}} = - \frac{\sqrt{2}}{32} + \frac{\sqrt{6}}{32} + i \left(\frac{\sqrt{2}}{32} + \frac{\sqrt{6}}{32}\right) = 0.0323 + 0.121 i$ + \end{enumerate} +\end{solution} + +\begin{exercise}[subtitle={Bassin}] + Le tour d'un bassin au niveau du sol présente deux axes de symétrie : l’axe des abscisses et la droite d’équation $x=4$. Il est obtenu par symétrie de la courbe $\mathcal{C_f}$ sur $\intFF{0}{4}$ où $f$ est la fonction définie par + + \[ + f(x) = \left(- x^{2} + 5.8 x - 4.6\right) e^{- x} + 4.6 + \] + On admet que sur $\intFF{0}{4}$ la fonction $f$ est positive. + \begin{enumerate} + \item Sur un repère, tracer l'allure de la courbe $\mathcal{C}_f$, les axes de symétries puis compléter pour dessiner la forme du bassin. + \item Montrer que la fonction $f$ admet comme primitive sur $\R$ la fonction $F$ définie par + \[ + F(x) = 4.6 x + \left( x^{2} - 3.8 x + 0.8\right) e^{- x} + \] + \item Calculer la quantité $\ds \int_0^4 f(x) \; dx$, vous donnerez le résultat sous forme exacte. Interpréter le résultat et reportez cette quantité sur le graphique. + \item On considère que l'échelle de votre graphique est de 1unité pour 15m. Calculer l'aire du bassin. Vous donnerez un résultat arrondi au $m^2$ près. + \end{enumerate} +\end{exercise} + +\begin{solution} + \begin{enumerate} + \item + \begin{tikzpicture}[baseline=(a.north), xscale=1, yscale=0.5] + \tkzInit[xmin=0,xmax=5,xstep=1, + ymin=0,ymax=10,ystep=1] + \tkzGrid + \tkzAxeXY + \tkzFct[domain=0:10,color=red,very thick]% + { (-x**2 + 5.8*x - 4.6)*exp(-x) + 4.6 }; + \end{tikzpicture} + \item Il faut dériver $F(x)$ et vérifier que $F'(x) = f(x)$. + \item $\ds \int_0^4 f(x) \; dx = F(4) - F(0) = \frac{1.6}{e^{4}} + 17.6$ + \item La quantité calculée à la question précédente se retrouve 4fois pour former le bassin. Il faut ensuite prendre en compte l'échelle, comme 1unité de longueur correspond à 15m, une unité d'air correspond à $15\times15 = 225m^2$. Ainsi l'aire du bassin est égale à + \[ + (\frac{1.6}{e^{4}} + 17.6)\times 4 \times 15^2 = 15866.00000 + \] + + \end{enumerate} +\end{solution} + +\begin{exercise}[subtitle={Bassin}] + Le clinker est un constituant du ciment qui résulte de la cuisson d'un mélange composé de calcaire et d'argile. La fabrication du clinker nécessite des fours à très haute température qui libèrent dans l'air une grande quantité de dioxyde de carbone (CO$_2$). + + Dans une cimenterie, la fabrication du clinker s'effectue de 7 h 30 à 20 h, dans une pièce de volume \np{700000}~dm$^3$. + + À 20 h, après une journée de travail, le taux volumique de CO$_2$ dans la pièce est de 1.0\,\%. + \begin{enumerate} + \item Justifier que le volume de CO$_2$ présent dans cette pièce à 20 h est de \np{7000}~dm$^3$ . + \item On modélise le volume de CO$_2$ présent dans la pièce par une fonction du temps $t$ écoulé après 20h (exprimé en minutes) qui pour formule $V(t) = V_0e^{-0.07t} + 520$ + \begin{enumerate} + \item Démontrer que $V_0$ est égale à \np{6480}. + \item Quel sera, au dm$^3$ près, le volume de CO$_2$ dans cette pièce à 24 h ? + \item Démontrer que $V'(t) = - 453.6 e^{- 0.07 t}$. + \item Étudier le signe de $V'(t)$ puis en déduire le sens de variation de $V(t)$. + \item Que peut-on dire du volume de CO$_2$ quand $t$ devient grand? + \end{enumerate} + \end{enumerate} +\end{exercise} + +\begin{solution} + \begin{enumerate} + \item Volume à 20h: $700000\times 0.01 = 7000$ + \item + \begin{enumerate} + \item $t=0$ correspond à 20h. + + Donc $V(0) = 7000 = V_0e^{-0.07\times 0} + 520 = V_0 + 520$ + + Donc $V_0 = 7000 - 520 = 6480$ + \item Il faut calculer $V(t)$ pour $t = 4$ donc + \[ + V(4) = 5417.48 + \] + \item Pas de correction pour cette question. + \item Pas de correction pour cette question. + \item Pas de correction pour cette question. + \end{enumerate} + \end{enumerate} +\end{solution} + + +\end{document} + +%%% Local Variables: +%%% mode: latex +%%% TeX-master: "master" +%%% End: diff --git a/TST_sti2d/DM/2102_DM1/corr_13_2102_DM1.tex b/TST_sti2d/DM/2102_DM1/corr_13_2102_DM1.tex new file mode 100644 index 0000000..585e0e5 --- /dev/null +++ b/TST_sti2d/DM/2102_DM1/corr_13_2102_DM1.tex @@ -0,0 +1,121 @@ +\documentclass[a4paper,10pt]{article} +\usepackage{myXsim} +\usepackage{tasks} + +% Title Page +\title{DM1 \hfill M'BAREK HASNAOUI Bilal} +\tribe{TST sti2d} +\date{\hfillÀ render pour le jeudi 25 février} + +\xsimsetup{ + solution/print = true +} + +\begin{document} +\maketitle + +\begin{exercise}[subtitle={Complexes}] + \begin{enumerate} + \item Mettre le nombre complexe suivant sous forme algébrique $z_1 = \dfrac{4 + 7 i}{-10 + 7 i} $ + \item Mettre le complexe suivante sous forme exponentielle $z_2 = 6 - 6 \sqrt{3} i$ + \item Mettre le complexe suivante sous forme exponentielle $z_3 = 6 \sqrt{2} + 6 \sqrt{2} i$ + \item Calculer le produit $z_4=z_2\times z_3$ donner le résultat sous forme exponentielle puis algébrique. + \item Calculer le quotient $z_5=\frac{z_2}{z_3}$ donner le résultat sous forme exponentielle puis algébrique. + \end{enumerate} +\end{exercise} + +\begin{solution} + \begin{enumerate} + \item $z_1 = \frac{9}{149} - \frac{98 i}{149}$ + \item $z_3 = 12 e^{- \frac{i \pi}{3}}$ + \item $z_4 = 144 e^{- \frac{i \pi}{12}} = 36 \sqrt{2} + 36 \sqrt{6} + i \left(- 36 \sqrt{6} + 36 \sqrt{2}\right) = 139.0 - 37.3 i$ + \item $z_5 = 1 e^{- \frac{7 i \pi}{12}} = - \frac{\sqrt{6}}{4} + \frac{\sqrt{2}}{4} + i \left(- \frac{\sqrt{6}}{4} - \frac{\sqrt{2}}{4}\right) = -0.259 - 0.966 i$ + \end{enumerate} +\end{solution} + +\begin{exercise}[subtitle={Bassin}] + Le tour d'un bassin au niveau du sol présente deux axes de symétrie : l’axe des abscisses et la droite d’équation $x=4$. Il est obtenu par symétrie de la courbe $\mathcal{C_f}$ sur $\intFF{0}{4}$ où $f$ est la fonction définie par + + \[ + f(x) = \left(- x^{2} + 2.2 x - 9.7\right) e^{- x} + 9.7 + \] + On admet que sur $\intFF{0}{4}$ la fonction $f$ est positive. + \begin{enumerate} + \item Sur un repère, tracer l'allure de la courbe $\mathcal{C}_f$, les axes de symétries puis compléter pour dessiner la forme du bassin. + \item Montrer que la fonction $f$ admet comme primitive sur $\R$ la fonction $F$ définie par + \[ + F(x) = 9.7 x + \left( x^{2} - 0.2 x + 9.5\right) e^{- x} + \] + \item Calculer la quantité $\ds \int_0^4 f(x) \; dx$, vous donnerez le résultat sous forme exacte. Interpréter le résultat et reportez cette quantité sur le graphique. + \item On considère que l'échelle de votre graphique est de 1unité pour 15m. Calculer l'aire du bassin. Vous donnerez un résultat arrondi au $m^2$ près. + \end{enumerate} +\end{exercise} + +\begin{solution} + \begin{enumerate} + \item + \begin{tikzpicture}[baseline=(a.north), xscale=1, yscale=0.5] + \tkzInit[xmin=0,xmax=5,xstep=1, + ymin=0,ymax=10,ystep=1] + \tkzGrid + \tkzAxeXY + \tkzFct[domain=0:10,color=red,very thick]% + { (-x**2 + 2.2*x - 9.7)*exp(-x) + 9.7 }; + \end{tikzpicture} + \item Il faut dériver $F(x)$ et vérifier que $F'(x) = f(x)$. + \item $\ds \int_0^4 f(x) \; dx = F(4) - F(0) = \frac{24.7}{e^{4}} + 29.3$ + \item La quantité calculée à la question précédente se retrouve 4fois pour former le bassin. Il faut ensuite prendre en compte l'échelle, comme 1unité de longueur correspond à 15m, une unité d'air correspond à $15\times15 = 225m^2$. Ainsi l'aire du bassin est égale à + \[ + (\frac{24.7}{e^{4}} + 29.3)\times 4 \times 15^2 = 26777.00000 + \] + + \end{enumerate} +\end{solution} + +\begin{exercise}[subtitle={Bassin}] + Le clinker est un constituant du ciment qui résulte de la cuisson d'un mélange composé de calcaire et d'argile. La fabrication du clinker nécessite des fours à très haute température qui libèrent dans l'air une grande quantité de dioxyde de carbone (CO$_2$). + + Dans une cimenterie, la fabrication du clinker s'effectue de 7 h 30 à 20 h, dans une pièce de volume \np{300000}~dm$^3$. + + À 20 h, après une journée de travail, le taux volumique de CO$_2$ dans la pièce est de 0.6\,\%. + \begin{enumerate} + \item Justifier que le volume de CO$_2$ présent dans cette pièce à 20 h est de \np{1800}~dm$^3$ . + \item On modélise le volume de CO$_2$ présent dans la pièce par une fonction du temps $t$ écoulé après 20h (exprimé en minutes) qui pour formule $V(t) = V_0e^{-0.1t} + 430$ + \begin{enumerate} + \item Démontrer que $V_0$ est égale à \np{1370}. + \item Quel sera, au dm$^3$ près, le volume de CO$_2$ dans cette pièce à 22 h ? + \item Démontrer que $V'(t) = - 137.0 e^{- 0.1 t}$. + \item Étudier le signe de $V'(t)$ puis en déduire le sens de variation de $V(t)$. + \item Que peut-on dire du volume de CO$_2$ quand $t$ devient grand? + \end{enumerate} + \end{enumerate} +\end{exercise} + +\begin{solution} + \begin{enumerate} + \item Volume à 20h: $300000\times 0.006 = 1800$ + \item + \begin{enumerate} + \item $t=0$ correspond à 20h. + + Donc $V(0) = 1800 = V_0e^{-0.1\times 0} + 430 = V_0 + 430$ + + Donc $V_0 = 1800 - 430 = 1370$ + \item Il faut calculer $V(t)$ pour $t = 2$ donc + \[ + V(2) = 1551.66 + \] + \item Pas de correction pour cette question. + \item Pas de correction pour cette question. + \item Pas de correction pour cette question. + \end{enumerate} + \end{enumerate} +\end{solution} + + +\end{document} + +%%% Local Variables: +%%% mode: latex +%%% TeX-master: "master" +%%% End: diff --git a/TST_sti2d/DM/2102_DM1/corr_14_2102_DM1.tex b/TST_sti2d/DM/2102_DM1/corr_14_2102_DM1.tex new file mode 100644 index 0000000..e3b378b --- /dev/null +++ b/TST_sti2d/DM/2102_DM1/corr_14_2102_DM1.tex @@ -0,0 +1,121 @@ +\documentclass[a4paper,10pt]{article} +\usepackage{myXsim} +\usepackage{tasks} + +% Title Page +\title{DM1 \hfill MERCIER Almandin} +\tribe{TST sti2d} +\date{\hfillÀ render pour le jeudi 25 février} + +\xsimsetup{ + solution/print = true +} + +\begin{document} +\maketitle + +\begin{exercise}[subtitle={Complexes}] + \begin{enumerate} + \item Mettre le nombre complexe suivant sous forme algébrique $z_1 = \dfrac{2 + 4 i}{-6 + 6 i} $ + \item Mettre le complexe suivante sous forme exponentielle $z_2 = 4 \sqrt{3} - 4 i$ + \item Mettre le complexe suivante sous forme exponentielle $z_3 = - 10 \sqrt{2} + 10 \sqrt{2} i$ + \item Calculer le produit $z_4=z_2\times z_3$ donner le résultat sous forme exponentielle puis algébrique. + \item Calculer le quotient $z_5=\frac{z_2}{z_3}$ donner le résultat sous forme exponentielle puis algébrique. + \end{enumerate} +\end{exercise} + +\begin{solution} + \begin{enumerate} + \item $z_1 = \frac{1}{6} - \frac{i}{2}$ + \item $z_3 = 8 e^{- \frac{i \pi}{6}}$ + \item $z_4 = 160 e^{\frac{7 i \pi}{12}} = - 40 \sqrt{6} + 40 \sqrt{2} + i \left(40 \sqrt{2} + 40 \sqrt{6}\right) = -41.4 + 155.0 i$ + \item $z_5 = \frac{2}{5} e^{- \frac{11 i \pi}{12}} = - \frac{\sqrt{6}}{10} - \frac{\sqrt{2}}{10} + i \left(- \frac{\sqrt{6}}{10} + \frac{\sqrt{2}}{10}\right) = -0.386 - 0.104 i$ + \end{enumerate} +\end{solution} + +\begin{exercise}[subtitle={Bassin}] + Le tour d'un bassin au niveau du sol présente deux axes de symétrie : l’axe des abscisses et la droite d’équation $x=4$. Il est obtenu par symétrie de la courbe $\mathcal{C_f}$ sur $\intFF{0}{4}$ où $f$ est la fonction définie par + + \[ + f(x) = \left(- x^{2} + 1.4 x - 4.9\right) e^{- x} + 4.9 + \] + On admet que sur $\intFF{0}{4}$ la fonction $f$ est positive. + \begin{enumerate} + \item Sur un repère, tracer l'allure de la courbe $\mathcal{C}_f$, les axes de symétries puis compléter pour dessiner la forme du bassin. + \item Montrer que la fonction $f$ admet comme primitive sur $\R$ la fonction $F$ définie par + \[ + F(x) = 4.9 x + \left( x^{2} + 0.6 x + 5.5\right) e^{- x} + \] + \item Calculer la quantité $\ds \int_0^4 f(x) \; dx$, vous donnerez le résultat sous forme exacte. Interpréter le résultat et reportez cette quantité sur le graphique. + \item On considère que l'échelle de votre graphique est de 1unité pour 15m. Calculer l'aire du bassin. Vous donnerez un résultat arrondi au $m^2$ près. + \end{enumerate} +\end{exercise} + +\begin{solution} + \begin{enumerate} + \item + \begin{tikzpicture}[baseline=(a.north), xscale=1, yscale=0.5] + \tkzInit[xmin=0,xmax=5,xstep=1, + ymin=0,ymax=10,ystep=1] + \tkzGrid + \tkzAxeXY + \tkzFct[domain=0:10,color=red,very thick]% + { (-x**2 + 1.4*x - 4.9)*exp(-x) + 4.9 }; + \end{tikzpicture} + \item Il faut dériver $F(x)$ et vérifier que $F'(x) = f(x)$. + \item $\ds \int_0^4 f(x) \; dx = F(4) - F(0) = \frac{23.9}{e^{4}} + 14.1$ + \item La quantité calculée à la question précédente se retrouve 4fois pour former le bassin. Il faut ensuite prendre en compte l'échelle, comme 1unité de longueur correspond à 15m, une unité d'air correspond à $15\times15 = 225m^2$. Ainsi l'aire du bassin est égale à + \[ + (\frac{23.9}{e^{4}} + 14.1)\times 4 \times 15^2 = 13084.00000 + \] + + \end{enumerate} +\end{solution} + +\begin{exercise}[subtitle={Bassin}] + Le clinker est un constituant du ciment qui résulte de la cuisson d'un mélange composé de calcaire et d'argile. La fabrication du clinker nécessite des fours à très haute température qui libèrent dans l'air une grande quantité de dioxyde de carbone (CO$_2$). + + Dans une cimenterie, la fabrication du clinker s'effectue de 7 h 30 à 20 h, dans une pièce de volume \np{400000}~dm$^3$. + + À 20 h, après une journée de travail, le taux volumique de CO$_2$ dans la pièce est de 0.7\,\%. + \begin{enumerate} + \item Justifier que le volume de CO$_2$ présent dans cette pièce à 20 h est de \np{2800}~dm$^3$ . + \item On modélise le volume de CO$_2$ présent dans la pièce par une fonction du temps $t$ écoulé après 20h (exprimé en minutes) qui pour formule $V(t) = V_0e^{-0.07t} + 360$ + \begin{enumerate} + \item Démontrer que $V_0$ est égale à \np{2440}. + \item Quel sera, au dm$^3$ près, le volume de CO$_2$ dans cette pièce à 23 h ? + \item Démontrer que $V'(t) = - 170.8 e^{- 0.07 t}$. + \item Étudier le signe de $V'(t)$ puis en déduire le sens de variation de $V(t)$. + \item Que peut-on dire du volume de CO$_2$ quand $t$ devient grand? + \end{enumerate} + \end{enumerate} +\end{exercise} + +\begin{solution} + \begin{enumerate} + \item Volume à 20h: $400000\times 0.006999999999999999 = 2800$ + \item + \begin{enumerate} + \item $t=0$ correspond à 20h. + + Donc $V(0) = 2800 = V_0e^{-0.07\times 0} + 360 = V_0 + 360$ + + Donc $V_0 = 2800 - 360 = 2440$ + \item Il faut calculer $V(t)$ pour $t = 3$ donc + \[ + V(3) = 2337.83 + \] + \item Pas de correction pour cette question. + \item Pas de correction pour cette question. + \item Pas de correction pour cette question. + \end{enumerate} + \end{enumerate} +\end{solution} + + +\end{document} + +%%% Local Variables: +%%% mode: latex +%%% TeX-master: "master" +%%% End: diff --git a/TST_sti2d/DM/2102_DM1/corr_15_2102_DM1.tex b/TST_sti2d/DM/2102_DM1/corr_15_2102_DM1.tex new file mode 100644 index 0000000..22b18e8 --- /dev/null +++ b/TST_sti2d/DM/2102_DM1/corr_15_2102_DM1.tex @@ -0,0 +1,121 @@ +\documentclass[a4paper,10pt]{article} +\usepackage{myXsim} +\usepackage{tasks} + +% Title Page +\title{DM1 \hfill MOUFAQ Amine} +\tribe{TST sti2d} +\date{\hfillÀ render pour le jeudi 25 février} + +\xsimsetup{ + solution/print = true +} + +\begin{document} +\maketitle + +\begin{exercise}[subtitle={Complexes}] + \begin{enumerate} + \item Mettre le nombre complexe suivant sous forme algébrique $z_1 = \dfrac{6 + 9 i}{-9 + 10 i} $ + \item Mettre le complexe suivante sous forme exponentielle $z_2 = 10 \sqrt{3} + 10 i$ + \item Mettre le complexe suivante sous forme exponentielle $z_3 = 9 \sqrt{2} + 9 \sqrt{2} i$ + \item Calculer le produit $z_4=z_2\times z_3$ donner le résultat sous forme exponentielle puis algébrique. + \item Calculer le quotient $z_5=\frac{z_2}{z_3}$ donner le résultat sous forme exponentielle puis algébrique. + \end{enumerate} +\end{exercise} + +\begin{solution} + \begin{enumerate} + \item $z_1 = \frac{36}{181} - \frac{141 i}{181}$ + \item $z_3 = 20 e^{\frac{i \pi}{6}}$ + \item $z_4 = 360 e^{\frac{5 i \pi}{12}} = - 90 \sqrt{2} + 90 \sqrt{6} + i \left(90 \sqrt{2} + 90 \sqrt{6}\right) = 93.2 + 348.0 i$ + \item $z_5 = \frac{10}{9} e^{- \frac{i \pi}{12}} = \frac{5 \sqrt{2}}{18} + \frac{5 \sqrt{6}}{18} + i \left(- \frac{5 \sqrt{6}}{18} + \frac{5 \sqrt{2}}{18}\right) = 1.07 - 0.288 i$ + \end{enumerate} +\end{solution} + +\begin{exercise}[subtitle={Bassin}] + Le tour d'un bassin au niveau du sol présente deux axes de symétrie : l’axe des abscisses et la droite d’équation $x=4$. Il est obtenu par symétrie de la courbe $\mathcal{C_f}$ sur $\intFF{0}{4}$ où $f$ est la fonction définie par + + \[ + f(x) = \left(- x^{2} + 5.5 x - 9.8\right) e^{- x} + 9.8 + \] + On admet que sur $\intFF{0}{4}$ la fonction $f$ est positive. + \begin{enumerate} + \item Sur un repère, tracer l'allure de la courbe $\mathcal{C}_f$, les axes de symétries puis compléter pour dessiner la forme du bassin. + \item Montrer que la fonction $f$ admet comme primitive sur $\R$ la fonction $F$ définie par + \[ + F(x) = 9.8 x + \left( x^{2} - 3.5 x + 6.3\right) e^{- x} + \] + \item Calculer la quantité $\ds \int_0^4 f(x) \; dx$, vous donnerez le résultat sous forme exacte. Interpréter le résultat et reportez cette quantité sur le graphique. + \item On considère que l'échelle de votre graphique est de 1unité pour 15m. Calculer l'aire du bassin. Vous donnerez un résultat arrondi au $m^2$ près. + \end{enumerate} +\end{exercise} + +\begin{solution} + \begin{enumerate} + \item + \begin{tikzpicture}[baseline=(a.north), xscale=1, yscale=0.5] + \tkzInit[xmin=0,xmax=5,xstep=1, + ymin=0,ymax=10,ystep=1] + \tkzGrid + \tkzAxeXY + \tkzFct[domain=0:10,color=red,very thick]% + { (-x**2 + 5.5*x - 9.8)*exp(-x) + 9.8 }; + \end{tikzpicture} + \item Il faut dériver $F(x)$ et vérifier que $F'(x) = f(x)$. + \item $\ds \int_0^4 f(x) \; dx = F(4) - F(0) = \frac{8.3}{e^{4}} + 32.9$ + \item La quantité calculée à la question précédente se retrouve 4fois pour former le bassin. Il faut ensuite prendre en compte l'échelle, comme 1unité de longueur correspond à 15m, une unité d'air correspond à $15\times15 = 225m^2$. Ainsi l'aire du bassin est égale à + \[ + (\frac{8.3}{e^{4}} + 32.9)\times 4 \times 15^2 = 29747.00000 + \] + + \end{enumerate} +\end{solution} + +\begin{exercise}[subtitle={Bassin}] + Le clinker est un constituant du ciment qui résulte de la cuisson d'un mélange composé de calcaire et d'argile. La fabrication du clinker nécessite des fours à très haute température qui libèrent dans l'air une grande quantité de dioxyde de carbone (CO$_2$). + + Dans une cimenterie, la fabrication du clinker s'effectue de 7 h 30 à 20 h, dans une pièce de volume \np{600000}~dm$^3$. + + À 20 h, après une journée de travail, le taux volumique de CO$_2$ dans la pièce est de 0.9\,\%. + \begin{enumerate} + \item Justifier que le volume de CO$_2$ présent dans cette pièce à 20 h est de \np{5400}~dm$^3$ . + \item On modélise le volume de CO$_2$ présent dans la pièce par une fonction du temps $t$ écoulé après 20h (exprimé en minutes) qui pour formule $V(t) = V_0e^{-0.06t} + 340$ + \begin{enumerate} + \item Démontrer que $V_0$ est égale à \np{5060}. + \item Quel sera, au dm$^3$ près, le volume de CO$_2$ dans cette pièce à 24 h ? + \item Démontrer que $V'(t) = - 303.6 e^{- 0.06 t}$. + \item Étudier le signe de $V'(t)$ puis en déduire le sens de variation de $V(t)$. + \item Que peut-on dire du volume de CO$_2$ quand $t$ devient grand? + \end{enumerate} + \end{enumerate} +\end{exercise} + +\begin{solution} + \begin{enumerate} + \item Volume à 20h: $600000\times 0.009000000000000001 = 5400$ + \item + \begin{enumerate} + \item $t=0$ correspond à 20h. + + Donc $V(0) = 5400 = V_0e^{-0.06\times 0} + 340 = V_0 + 340$ + + Donc $V_0 = 5400 - 340 = 5060$ + \item Il faut calculer $V(t)$ pour $t = 4$ donc + \[ + V(4) = 4320.34 + \] + \item Pas de correction pour cette question. + \item Pas de correction pour cette question. + \item Pas de correction pour cette question. + \end{enumerate} + \end{enumerate} +\end{solution} + + +\end{document} + +%%% Local Variables: +%%% mode: latex +%%% TeX-master: "master" +%%% End: diff --git a/TST_sti2d/DM/2102_DM1/corr_16_2102_DM1.tex b/TST_sti2d/DM/2102_DM1/corr_16_2102_DM1.tex new file mode 100644 index 0000000..35de724 --- /dev/null +++ b/TST_sti2d/DM/2102_DM1/corr_16_2102_DM1.tex @@ -0,0 +1,121 @@ +\documentclass[a4paper,10pt]{article} +\usepackage{myXsim} +\usepackage{tasks} + +% Title Page +\title{DM1 \hfill NARDINI Kakary} +\tribe{TST sti2d} +\date{\hfillÀ render pour le jeudi 25 février} + +\xsimsetup{ + solution/print = true +} + +\begin{document} +\maketitle + +\begin{exercise}[subtitle={Complexes}] + \begin{enumerate} + \item Mettre le nombre complexe suivant sous forme algébrique $z_1 = \dfrac{7 + 5 i}{-4 + 5 i} $ + \item Mettre le complexe suivante sous forme exponentielle $z_2 = - 8 \sqrt{2} + 8 \sqrt{2} i$ + \item Mettre le complexe suivante sous forme exponentielle $z_3 = \sqrt{3} + i$ + \item Calculer le produit $z_4=z_2\times z_3$ donner le résultat sous forme exponentielle puis algébrique. + \item Calculer le quotient $z_5=\frac{z_2}{z_3}$ donner le résultat sous forme exponentielle puis algébrique. + \end{enumerate} +\end{exercise} + +\begin{solution} + \begin{enumerate} + \item $z_1 = - \frac{3}{41} - \frac{55 i}{41}$ + \item $z_3 = 16 e^{\frac{3 i \pi}{4}}$ + \item $z_4 = 32 e^{\frac{11 i \pi}{12}} = - 8 \sqrt{6} - 8 \sqrt{2} + i \left(- 8 \sqrt{2} + 8 \sqrt{6}\right) = -30.9 + 8.28 i$ + \item $z_5 = 8 e^{\frac{7 i \pi}{12}} = - 2 \sqrt{6} + 2 \sqrt{2} + i \left(2 \sqrt{2} + 2 \sqrt{6}\right) = -2.07 + 7.73 i$ + \end{enumerate} +\end{solution} + +\begin{exercise}[subtitle={Bassin}] + Le tour d'un bassin au niveau du sol présente deux axes de symétrie : l’axe des abscisses et la droite d’équation $x=4$. Il est obtenu par symétrie de la courbe $\mathcal{C_f}$ sur $\intFF{0}{4}$ où $f$ est la fonction définie par + + \[ + f(x) = \left(- x^{2} + 2.3 x - 1.7\right) e^{- x} + 1.7 + \] + On admet que sur $\intFF{0}{4}$ la fonction $f$ est positive. + \begin{enumerate} + \item Sur un repère, tracer l'allure de la courbe $\mathcal{C}_f$, les axes de symétries puis compléter pour dessiner la forme du bassin. + \item Montrer que la fonction $f$ admet comme primitive sur $\R$ la fonction $F$ définie par + \[ + F(x) = 1.7 x + \left( x^{2} - 0.3 x + 1.4\right) e^{- x} + \] + \item Calculer la quantité $\ds \int_0^4 f(x) \; dx$, vous donnerez le résultat sous forme exacte. Interpréter le résultat et reportez cette quantité sur le graphique. + \item On considère que l'échelle de votre graphique est de 1unité pour 15m. Calculer l'aire du bassin. Vous donnerez un résultat arrondi au $m^2$ près. + \end{enumerate} +\end{exercise} + +\begin{solution} + \begin{enumerate} + \item + \begin{tikzpicture}[baseline=(a.north), xscale=1, yscale=0.5] + \tkzInit[xmin=0,xmax=5,xstep=1, + ymin=0,ymax=10,ystep=1] + \tkzGrid + \tkzAxeXY + \tkzFct[domain=0:10,color=red,very thick]% + { (-x**2 + 2.3*x - 1.7)*exp(-x) + 1.7 }; + \end{tikzpicture} + \item Il faut dériver $F(x)$ et vérifier que $F'(x) = f(x)$. + \item $\ds \int_0^4 f(x) \; dx = F(4) - F(0) = \frac{16.2}{e^{4}} + 5.4$ + \item La quantité calculée à la question précédente se retrouve 4fois pour former le bassin. Il faut ensuite prendre en compte l'échelle, comme 1unité de longueur correspond à 15m, une unité d'air correspond à $15\times15 = 225m^2$. Ainsi l'aire du bassin est égale à + \[ + (\frac{16.2}{e^{4}} + 5.4)\times 4 \times 15^2 = 5127.000000 + \] + + \end{enumerate} +\end{solution} + +\begin{exercise}[subtitle={Bassin}] + Le clinker est un constituant du ciment qui résulte de la cuisson d'un mélange composé de calcaire et d'argile. La fabrication du clinker nécessite des fours à très haute température qui libèrent dans l'air une grande quantité de dioxyde de carbone (CO$_2$). + + Dans une cimenterie, la fabrication du clinker s'effectue de 7 h 30 à 20 h, dans une pièce de volume \np{600000}~dm$^3$. + + À 20 h, après une journée de travail, le taux volumique de CO$_2$ dans la pièce est de 0.9\,\%. + \begin{enumerate} + \item Justifier que le volume de CO$_2$ présent dans cette pièce à 20 h est de \np{5400}~dm$^3$ . + \item On modélise le volume de CO$_2$ présent dans la pièce par une fonction du temps $t$ écoulé après 20h (exprimé en minutes) qui pour formule $V(t) = V_0e^{-0.09t} + 360$ + \begin{enumerate} + \item Démontrer que $V_0$ est égale à \np{5040}. + \item Quel sera, au dm$^3$ près, le volume de CO$_2$ dans cette pièce à 22 h ? + \item Démontrer que $V'(t) = - 453.6 e^{- 0.09 t}$. + \item Étudier le signe de $V'(t)$ puis en déduire le sens de variation de $V(t)$. + \item Que peut-on dire du volume de CO$_2$ quand $t$ devient grand? + \end{enumerate} + \end{enumerate} +\end{exercise} + +\begin{solution} + \begin{enumerate} + \item Volume à 20h: $600000\times 0.009000000000000001 = 5400$ + \item + \begin{enumerate} + \item $t=0$ correspond à 20h. + + Donc $V(0) = 5400 = V_0e^{-0.09\times 0} + 360 = V_0 + 360$ + + Donc $V_0 = 5400 - 360 = 5040$ + \item Il faut calculer $V(t)$ pour $t = 2$ donc + \[ + V(2) = 4569.76 + \] + \item Pas de correction pour cette question. + \item Pas de correction pour cette question. + \item Pas de correction pour cette question. + \end{enumerate} + \end{enumerate} +\end{solution} + + +\end{document} + +%%% Local Variables: +%%% mode: latex +%%% TeX-master: "master" +%%% End: diff --git a/TST_sti2d/DM/2102_DM1/corr_17_2102_DM1.tex b/TST_sti2d/DM/2102_DM1/corr_17_2102_DM1.tex new file mode 100644 index 0000000..9edf658 --- /dev/null +++ b/TST_sti2d/DM/2102_DM1/corr_17_2102_DM1.tex @@ -0,0 +1,121 @@ +\documentclass[a4paper,10pt]{article} +\usepackage{myXsim} +\usepackage{tasks} + +% Title Page +\title{DM1 \hfill ONAL Yakub} +\tribe{TST sti2d} +\date{\hfillÀ render pour le jeudi 25 février} + +\xsimsetup{ + solution/print = true +} + +\begin{document} +\maketitle + +\begin{exercise}[subtitle={Complexes}] + \begin{enumerate} + \item Mettre le nombre complexe suivant sous forme algébrique $z_1 = \dfrac{8 + 7 i}{-5 + 6 i} $ + \item Mettre le complexe suivante sous forme exponentielle $z_2 = 7 + 7 \sqrt{3} i$ + \item Mettre le complexe suivante sous forme exponentielle $z_3 = \sqrt{3} + i$ + \item Calculer le produit $z_4=z_2\times z_3$ donner le résultat sous forme exponentielle puis algébrique. + \item Calculer le quotient $z_5=\frac{z_2}{z_3}$ donner le résultat sous forme exponentielle puis algébrique. + \end{enumerate} +\end{exercise} + +\begin{solution} + \begin{enumerate} + \item $z_1 = \frac{2}{61} - \frac{83 i}{61}$ + \item $z_3 = 14 e^{\frac{i \pi}{3}}$ + \item $z_4 = 28 e^{\frac{i \pi}{2}} = 28 i = 28.0 i$ + \item $z_5 = 7 e^{\frac{i \pi}{6}} = \frac{7 \sqrt{3}}{2} + \frac{7 i}{2} = 6.06 + 3.5 i$ + \end{enumerate} +\end{solution} + +\begin{exercise}[subtitle={Bassin}] + Le tour d'un bassin au niveau du sol présente deux axes de symétrie : l’axe des abscisses et la droite d’équation $x=4$. Il est obtenu par symétrie de la courbe $\mathcal{C_f}$ sur $\intFF{0}{4}$ où $f$ est la fonction définie par + + \[ + f(x) = \left(- x^{2} + 3.6 x - 8.6\right) e^{- x} + 8.6 + \] + On admet que sur $\intFF{0}{4}$ la fonction $f$ est positive. + \begin{enumerate} + \item Sur un repère, tracer l'allure de la courbe $\mathcal{C}_f$, les axes de symétries puis compléter pour dessiner la forme du bassin. + \item Montrer que la fonction $f$ admet comme primitive sur $\R$ la fonction $F$ définie par + \[ + F(x) = 8.6 x + \left( x^{2} - 1.6 x + 7.0\right) e^{- x} + \] + \item Calculer la quantité $\ds \int_0^4 f(x) \; dx$, vous donnerez le résultat sous forme exacte. Interpréter le résultat et reportez cette quantité sur le graphique. + \item On considère que l'échelle de votre graphique est de 1unité pour 15m. Calculer l'aire du bassin. Vous donnerez un résultat arrondi au $m^2$ près. + \end{enumerate} +\end{exercise} + +\begin{solution} + \begin{enumerate} + \item + \begin{tikzpicture}[baseline=(a.north), xscale=1, yscale=0.5] + \tkzInit[xmin=0,xmax=5,xstep=1, + ymin=0,ymax=10,ystep=1] + \tkzGrid + \tkzAxeXY + \tkzFct[domain=0:10,color=red,very thick]% + { (-x**2 + 3.6*x - 8.6)*exp(-x) + 8.6 }; + \end{tikzpicture} + \item Il faut dériver $F(x)$ et vérifier que $F'(x) = f(x)$. + \item $\ds \int_0^4 f(x) \; dx = F(4) - F(0) = \frac{16.6}{e^{4}} + 27.4$ + \item La quantité calculée à la question précédente se retrouve 4fois pour former le bassin. Il faut ensuite prendre en compte l'échelle, comme 1unité de longueur correspond à 15m, une unité d'air correspond à $15\times15 = 225m^2$. Ainsi l'aire du bassin est égale à + \[ + (\frac{16.6}{e^{4}} + 27.4)\times 4 \times 15^2 = 24934.00000 + \] + + \end{enumerate} +\end{solution} + +\begin{exercise}[subtitle={Bassin}] + Le clinker est un constituant du ciment qui résulte de la cuisson d'un mélange composé de calcaire et d'argile. La fabrication du clinker nécessite des fours à très haute température qui libèrent dans l'air une grande quantité de dioxyde de carbone (CO$_2$). + + Dans une cimenterie, la fabrication du clinker s'effectue de 7 h 30 à 20 h, dans une pièce de volume \np{300000}~dm$^3$. + + À 20 h, après une journée de travail, le taux volumique de CO$_2$ dans la pièce est de 0.6\,\%. + \begin{enumerate} + \item Justifier que le volume de CO$_2$ présent dans cette pièce à 20 h est de \np{1800}~dm$^3$ . + \item On modélise le volume de CO$_2$ présent dans la pièce par une fonction du temps $t$ écoulé après 20h (exprimé en minutes) qui pour formule $V(t) = V_0e^{-0.08t} + 450$ + \begin{enumerate} + \item Démontrer que $V_0$ est égale à \np{1350}. + \item Quel sera, au dm$^3$ près, le volume de CO$_2$ dans cette pièce à 22 h ? + \item Démontrer que $V'(t) = - 108.0 e^{- 0.08 t}$. + \item Étudier le signe de $V'(t)$ puis en déduire le sens de variation de $V(t)$. + \item Que peut-on dire du volume de CO$_2$ quand $t$ devient grand? + \end{enumerate} + \end{enumerate} +\end{exercise} + +\begin{solution} + \begin{enumerate} + \item Volume à 20h: $300000\times 0.006 = 1800$ + \item + \begin{enumerate} + \item $t=0$ correspond à 20h. + + Donc $V(0) = 1800 = V_0e^{-0.08\times 0} + 450 = V_0 + 450$ + + Donc $V_0 = 1800 - 450 = 1350$ + \item Il faut calculer $V(t)$ pour $t = 2$ donc + \[ + V(2) = 1600.39 + \] + \item Pas de correction pour cette question. + \item Pas de correction pour cette question. + \item Pas de correction pour cette question. + \end{enumerate} + \end{enumerate} +\end{solution} + + +\end{document} + +%%% Local Variables: +%%% mode: latex +%%% TeX-master: "master" +%%% End: diff --git a/TST_sti2d/DM/2102_DM1/corr_18_2102_DM1.tex b/TST_sti2d/DM/2102_DM1/corr_18_2102_DM1.tex new file mode 100644 index 0000000..2bd4aff --- /dev/null +++ b/TST_sti2d/DM/2102_DM1/corr_18_2102_DM1.tex @@ -0,0 +1,121 @@ +\documentclass[a4paper,10pt]{article} +\usepackage{myXsim} +\usepackage{tasks} + +% Title Page +\title{DM1 \hfill RADOUAA Saleh} +\tribe{TST sti2d} +\date{\hfillÀ render pour le jeudi 25 février} + +\xsimsetup{ + solution/print = true +} + +\begin{document} +\maketitle + +\begin{exercise}[subtitle={Complexes}] + \begin{enumerate} + \item Mettre le nombre complexe suivant sous forme algébrique $z_1 = \dfrac{4 + 2 i}{-8 + 10 i} $ + \item Mettre le complexe suivante sous forme exponentielle $z_2 = 10 - 10 \sqrt{3} i$ + \item Mettre le complexe suivante sous forme exponentielle $z_3 = 9 \sqrt{3} + 9 i$ + \item Calculer le produit $z_4=z_2\times z_3$ donner le résultat sous forme exponentielle puis algébrique. + \item Calculer le quotient $z_5=\frac{z_2}{z_3}$ donner le résultat sous forme exponentielle puis algébrique. + \end{enumerate} +\end{exercise} + +\begin{solution} + \begin{enumerate} + \item $z_1 = - \frac{3}{41} - \frac{14 i}{41}$ + \item $z_3 = 20 e^{- \frac{i \pi}{3}}$ + \item $z_4 = 360 e^{- \frac{i \pi}{6}} = 180 \sqrt{3} - 180 i = 312.0 - 180.0 i$ + \item $z_5 = \frac{10}{9} e^{- \frac{i \pi}{2}} = - \frac{10 i}{9} = - 1.11 i$ + \end{enumerate} +\end{solution} + +\begin{exercise}[subtitle={Bassin}] + Le tour d'un bassin au niveau du sol présente deux axes de symétrie : l’axe des abscisses et la droite d’équation $x=4$. Il est obtenu par symétrie de la courbe $\mathcal{C_f}$ sur $\intFF{0}{4}$ où $f$ est la fonction définie par + + \[ + f(x) = \left(- x^{2} + 0.6 x - 2.3\right) e^{- x} + 2.3 + \] + On admet que sur $\intFF{0}{4}$ la fonction $f$ est positive. + \begin{enumerate} + \item Sur un repère, tracer l'allure de la courbe $\mathcal{C}_f$, les axes de symétries puis compléter pour dessiner la forme du bassin. + \item Montrer que la fonction $f$ admet comme primitive sur $\R$ la fonction $F$ définie par + \[ + F(x) = 2.3 x + \left( x^{2} + 1.4 x + 3.7\right) e^{- x} + \] + \item Calculer la quantité $\ds \int_0^4 f(x) \; dx$, vous donnerez le résultat sous forme exacte. Interpréter le résultat et reportez cette quantité sur le graphique. + \item On considère que l'échelle de votre graphique est de 1unité pour 15m. Calculer l'aire du bassin. Vous donnerez un résultat arrondi au $m^2$ près. + \end{enumerate} +\end{exercise} + +\begin{solution} + \begin{enumerate} + \item + \begin{tikzpicture}[baseline=(a.north), xscale=1, yscale=0.5] + \tkzInit[xmin=0,xmax=5,xstep=1, + ymin=0,ymax=10,ystep=1] + \tkzGrid + \tkzAxeXY + \tkzFct[domain=0:10,color=red,very thick]% + { (-x**2 + 0.6*x - 2.3)*exp(-x) + 2.3 }; + \end{tikzpicture} + \item Il faut dériver $F(x)$ et vérifier que $F'(x) = f(x)$. + \item $\ds \int_0^4 f(x) \; dx = F(4) - F(0) = \frac{25.3}{e^{4}} + 5.5$ + \item La quantité calculée à la question précédente se retrouve 4fois pour former le bassin. Il faut ensuite prendre en compte l'échelle, comme 1unité de longueur correspond à 15m, une unité d'air correspond à $15\times15 = 225m^2$. Ainsi l'aire du bassin est égale à + \[ + (\frac{25.3}{e^{4}} + 5.5)\times 4 \times 15^2 = 5367.000000 + \] + + \end{enumerate} +\end{solution} + +\begin{exercise}[subtitle={Bassin}] + Le clinker est un constituant du ciment qui résulte de la cuisson d'un mélange composé de calcaire et d'argile. La fabrication du clinker nécessite des fours à très haute température qui libèrent dans l'air une grande quantité de dioxyde de carbone (CO$_2$). + + Dans une cimenterie, la fabrication du clinker s'effectue de 7 h 30 à 20 h, dans une pièce de volume \np{500000}~dm$^3$. + + À 20 h, après une journée de travail, le taux volumique de CO$_2$ dans la pièce est de 0.9\,\%. + \begin{enumerate} + \item Justifier que le volume de CO$_2$ présent dans cette pièce à 20 h est de \np{4500}~dm$^3$ . + \item On modélise le volume de CO$_2$ présent dans la pièce par une fonction du temps $t$ écoulé après 20h (exprimé en minutes) qui pour formule $V(t) = V_0e^{-0.06t} + 440$ + \begin{enumerate} + \item Démontrer que $V_0$ est égale à \np{4060}. + \item Quel sera, au dm$^3$ près, le volume de CO$_2$ dans cette pièce à 24 h ? + \item Démontrer que $V'(t) = - 243.6 e^{- 0.06 t}$. + \item Étudier le signe de $V'(t)$ puis en déduire le sens de variation de $V(t)$. + \item Que peut-on dire du volume de CO$_2$ quand $t$ devient grand? + \end{enumerate} + \end{enumerate} +\end{exercise} + +\begin{solution} + \begin{enumerate} + \item Volume à 20h: $500000\times 0.009000000000000001 = 4500$ + \item + \begin{enumerate} + \item $t=0$ correspond à 20h. + + Donc $V(0) = 4500 = V_0e^{-0.06\times 0} + 440 = V_0 + 440$ + + Donc $V_0 = 4500 - 440 = 4060$ + \item Il faut calculer $V(t)$ pour $t = 4$ donc + \[ + V(4) = 3633.71 + \] + \item Pas de correction pour cette question. + \item Pas de correction pour cette question. + \item Pas de correction pour cette question. + \end{enumerate} + \end{enumerate} +\end{solution} + + +\end{document} + +%%% Local Variables: +%%% mode: latex +%%% TeX-master: "master" +%%% End: diff --git a/TST_sti2d/DM/2102_DM1/corr_19_2102_DM1.tex b/TST_sti2d/DM/2102_DM1/corr_19_2102_DM1.tex new file mode 100644 index 0000000..58f404e --- /dev/null +++ b/TST_sti2d/DM/2102_DM1/corr_19_2102_DM1.tex @@ -0,0 +1,121 @@ +\documentclass[a4paper,10pt]{article} +\usepackage{myXsim} +\usepackage{tasks} + +% Title Page +\title{DM1 \hfill TAVERNIER Joanny} +\tribe{TST sti2d} +\date{\hfillÀ render pour le jeudi 25 février} + +\xsimsetup{ + solution/print = true +} + +\begin{document} +\maketitle + +\begin{exercise}[subtitle={Complexes}] + \begin{enumerate} + \item Mettre le nombre complexe suivant sous forme algébrique $z_1 = \dfrac{10 + 6 i}{-7 + 3 i} $ + \item Mettre le complexe suivante sous forme exponentielle $z_2 = 6 - 6 \sqrt{3} i$ + \item Mettre le complexe suivante sous forme exponentielle $z_3 = 5 \sqrt{3} - 5 i$ + \item Calculer le produit $z_4=z_2\times z_3$ donner le résultat sous forme exponentielle puis algébrique. + \item Calculer le quotient $z_5=\frac{z_2}{z_3}$ donner le résultat sous forme exponentielle puis algébrique. + \end{enumerate} +\end{exercise} + +\begin{solution} + \begin{enumerate} + \item $z_1 = - \frac{26}{29} - \frac{36 i}{29}$ + \item $z_3 = 12 e^{- \frac{i \pi}{3}}$ + \item $z_4 = 120 e^{- \frac{i \pi}{2}} = - 120 i = - 120.0 i$ + \item $z_5 = \frac{6}{5} e^{- \frac{i \pi}{6}} = \frac{3 \sqrt{3}}{5} - \frac{3 i}{5} = 1.04 - 0.6 i$ + \end{enumerate} +\end{solution} + +\begin{exercise}[subtitle={Bassin}] + Le tour d'un bassin au niveau du sol présente deux axes de symétrie : l’axe des abscisses et la droite d’équation $x=4$. Il est obtenu par symétrie de la courbe $\mathcal{C_f}$ sur $\intFF{0}{4}$ où $f$ est la fonction définie par + + \[ + f(x) = \left(- x^{2} + 8.2 x - 3.6\right) e^{- x} + 3.6 + \] + On admet que sur $\intFF{0}{4}$ la fonction $f$ est positive. + \begin{enumerate} + \item Sur un repère, tracer l'allure de la courbe $\mathcal{C}_f$, les axes de symétries puis compléter pour dessiner la forme du bassin. + \item Montrer que la fonction $f$ admet comme primitive sur $\R$ la fonction $F$ définie par + \[ + F(x) = 3.6 x + \left( x^{2} - 6.2 x - 2.6\right) e^{- x} + \] + \item Calculer la quantité $\ds \int_0^4 f(x) \; dx$, vous donnerez le résultat sous forme exacte. Interpréter le résultat et reportez cette quantité sur le graphique. + \item On considère que l'échelle de votre graphique est de 1unité pour 15m. Calculer l'aire du bassin. Vous donnerez un résultat arrondi au $m^2$ près. + \end{enumerate} +\end{exercise} + +\begin{solution} + \begin{enumerate} + \item + \begin{tikzpicture}[baseline=(a.north), xscale=1, yscale=0.5] + \tkzInit[xmin=0,xmax=5,xstep=1, + ymin=0,ymax=10,ystep=1] + \tkzGrid + \tkzAxeXY + \tkzFct[domain=0:10,color=red,very thick]% + { (-x**2 + 8.2*x - 3.6)*exp(-x) + 3.6 }; + \end{tikzpicture} + \item Il faut dériver $F(x)$ et vérifier que $F'(x) = f(x)$. + \item $\ds \int_0^4 f(x) \; dx = F(4) - F(0) = 17.0 - \frac{11.4}{e^{4}}$ + \item La quantité calculée à la question précédente se retrouve 4fois pour former le bassin. Il faut ensuite prendre en compte l'échelle, comme 1unité de longueur correspond à 15m, une unité d'air correspond à $15\times15 = 225m^2$. Ainsi l'aire du bassin est égale à + \[ + (17.0 - \frac{11.4}{e^{4}})\times 4 \times 15^2 = 15112.00000 + \] + + \end{enumerate} +\end{solution} + +\begin{exercise}[subtitle={Bassin}] + Le clinker est un constituant du ciment qui résulte de la cuisson d'un mélange composé de calcaire et d'argile. La fabrication du clinker nécessite des fours à très haute température qui libèrent dans l'air une grande quantité de dioxyde de carbone (CO$_2$). + + Dans une cimenterie, la fabrication du clinker s'effectue de 7 h 30 à 20 h, dans une pièce de volume \np{100000}~dm$^3$. + + À 20 h, après une journée de travail, le taux volumique de CO$_2$ dans la pièce est de 0.8\,\%. + \begin{enumerate} + \item Justifier que le volume de CO$_2$ présent dans cette pièce à 20 h est de \np{800}~dm$^3$ . + \item On modélise le volume de CO$_2$ présent dans la pièce par une fonction du temps $t$ écoulé après 20h (exprimé en minutes) qui pour formule $V(t) = V_0e^{-0.01t} + 560$ + \begin{enumerate} + \item Démontrer que $V_0$ est égale à \np{240}. + \item Quel sera, au dm$^3$ près, le volume de CO$_2$ dans cette pièce à 22 h ? + \item Démontrer que $V'(t) = - 2.4 e^{- 0.01 t}$. + \item Étudier le signe de $V'(t)$ puis en déduire le sens de variation de $V(t)$. + \item Que peut-on dire du volume de CO$_2$ quand $t$ devient grand? + \end{enumerate} + \end{enumerate} +\end{exercise} + +\begin{solution} + \begin{enumerate} + \item Volume à 20h: $100000\times 0.008 = 800$ + \item + \begin{enumerate} + \item $t=0$ correspond à 20h. + + Donc $V(0) = 800 = V_0e^{-0.01\times 0} + 560 = V_0 + 560$ + + Donc $V_0 = 800 - 560 = 240$ + \item Il faut calculer $V(t)$ pour $t = 2$ donc + \[ + V(2) = 795.25 + \] + \item Pas de correction pour cette question. + \item Pas de correction pour cette question. + \item Pas de correction pour cette question. + \end{enumerate} + \end{enumerate} +\end{solution} + + +\end{document} + +%%% Local Variables: +%%% mode: latex +%%% TeX-master: "master" +%%% End: diff --git a/TST_sti2d/DM/2102_DM1/corr_20_2102_DM1.tex b/TST_sti2d/DM/2102_DM1/corr_20_2102_DM1.tex new file mode 100644 index 0000000..c6b194b --- /dev/null +++ b/TST_sti2d/DM/2102_DM1/corr_20_2102_DM1.tex @@ -0,0 +1,121 @@ +\documentclass[a4paper,10pt]{article} +\usepackage{myXsim} +\usepackage{tasks} + +% Title Page +\title{DM1 \hfill ZAHORE Zahiri} +\tribe{TST sti2d} +\date{\hfillÀ render pour le jeudi 25 février} + +\xsimsetup{ + solution/print = true +} + +\begin{document} +\maketitle + +\begin{exercise}[subtitle={Complexes}] + \begin{enumerate} + \item Mettre le nombre complexe suivant sous forme algébrique $z_1 = \dfrac{2 + 4 i}{-4 + 9 i} $ + \item Mettre le complexe suivante sous forme exponentielle $z_2 = - 2 \sqrt{3} - 2 i$ + \item Mettre le complexe suivante sous forme exponentielle $z_3 = 6 + 6 \sqrt{3} i$ + \item Calculer le produit $z_4=z_2\times z_3$ donner le résultat sous forme exponentielle puis algébrique. + \item Calculer le quotient $z_5=\frac{z_2}{z_3}$ donner le résultat sous forme exponentielle puis algébrique. + \end{enumerate} +\end{exercise} + +\begin{solution} + \begin{enumerate} + \item $z_1 = \frac{28}{97} - \frac{34 i}{97}$ + \item $z_3 = 4 e^{- \frac{5 i \pi}{6}}$ + \item $z_4 = 48 e^{- \frac{i \pi}{2}} = - 48 i = - 48.0 i$ + \item $z_5 = \frac{1}{3} e^{- \frac{7 i \pi}{6}} = - \frac{\sqrt{3}}{6} + \frac{i}{6} = -0.289 + 0.167 i$ + \end{enumerate} +\end{solution} + +\begin{exercise}[subtitle={Bassin}] + Le tour d'un bassin au niveau du sol présente deux axes de symétrie : l’axe des abscisses et la droite d’équation $x=4$. Il est obtenu par symétrie de la courbe $\mathcal{C_f}$ sur $\intFF{0}{4}$ où $f$ est la fonction définie par + + \[ + f(x) = \left(- x^{2} + 4.8 x - 3.0\right) e^{- x} + 3.0 + \] + On admet que sur $\intFF{0}{4}$ la fonction $f$ est positive. + \begin{enumerate} + \item Sur un repère, tracer l'allure de la courbe $\mathcal{C}_f$, les axes de symétries puis compléter pour dessiner la forme du bassin. + \item Montrer que la fonction $f$ admet comme primitive sur $\R$ la fonction $F$ définie par + \[ + F(x) = 3.0 x + \left( x^{2} - 2.8 x + 0.2\right) e^{- x} + \] + \item Calculer la quantité $\ds \int_0^4 f(x) \; dx$, vous donnerez le résultat sous forme exacte. Interpréter le résultat et reportez cette quantité sur le graphique. + \item On considère que l'échelle de votre graphique est de 1unité pour 15m. Calculer l'aire du bassin. Vous donnerez un résultat arrondi au $m^2$ près. + \end{enumerate} +\end{exercise} + +\begin{solution} + \begin{enumerate} + \item + \begin{tikzpicture}[baseline=(a.north), xscale=1, yscale=0.5] + \tkzInit[xmin=0,xmax=5,xstep=1, + ymin=0,ymax=10,ystep=1] + \tkzGrid + \tkzAxeXY + \tkzFct[domain=0:10,color=red,very thick]% + { (-x**2 + 4.8*x - 3.0)*exp(-x) + 3.0 }; + \end{tikzpicture} + \item Il faut dériver $F(x)$ et vérifier que $F'(x) = f(x)$. + \item $\ds \int_0^4 f(x) \; dx = F(4) - F(0) = \frac{5.0}{e^{4}} + 11.8$ + \item La quantité calculée à la question précédente se retrouve 4fois pour former le bassin. Il faut ensuite prendre en compte l'échelle, comme 1unité de longueur correspond à 15m, une unité d'air correspond à $15\times15 = 225m^2$. Ainsi l'aire du bassin est égale à + \[ + (\frac{5.0}{e^{4}} + 11.8)\times 4 \times 15^2 = 10702.00000 + \] + + \end{enumerate} +\end{solution} + +\begin{exercise}[subtitle={Bassin}] + Le clinker est un constituant du ciment qui résulte de la cuisson d'un mélange composé de calcaire et d'argile. La fabrication du clinker nécessite des fours à très haute température qui libèrent dans l'air une grande quantité de dioxyde de carbone (CO$_2$). + + Dans une cimenterie, la fabrication du clinker s'effectue de 7 h 30 à 20 h, dans une pièce de volume \np{500000}~dm$^3$. + + À 20 h, après une journée de travail, le taux volumique de CO$_2$ dans la pièce est de 0.8\,\%. + \begin{enumerate} + \item Justifier que le volume de CO$_2$ présent dans cette pièce à 20 h est de \np{4000}~dm$^3$ . + \item On modélise le volume de CO$_2$ présent dans la pièce par une fonction du temps $t$ écoulé après 20h (exprimé en minutes) qui pour formule $V(t) = V_0e^{-0.05t} + 230$ + \begin{enumerate} + \item Démontrer que $V_0$ est égale à \np{3770}. + \item Quel sera, au dm$^3$ près, le volume de CO$_2$ dans cette pièce à 24 h ? + \item Démontrer que $V'(t) = - 188.5 e^{- 0.05 t}$. + \item Étudier le signe de $V'(t)$ puis en déduire le sens de variation de $V(t)$. + \item Que peut-on dire du volume de CO$_2$ quand $t$ devient grand? + \end{enumerate} + \end{enumerate} +\end{exercise} + +\begin{solution} + \begin{enumerate} + \item Volume à 20h: $500000\times 0.008 = 4000$ + \item + \begin{enumerate} + \item $t=0$ correspond à 20h. + + Donc $V(0) = 4000 = V_0e^{-0.05\times 0} + 230 = V_0 + 230$ + + Donc $V_0 = 4000 - 230 = 3770$ + \item Il faut calculer $V(t)$ pour $t = 4$ donc + \[ + V(4) = 3316.61 + \] + \item Pas de correction pour cette question. + \item Pas de correction pour cette question. + \item Pas de correction pour cette question. + \end{enumerate} + \end{enumerate} +\end{solution} + + +\end{document} + +%%% Local Variables: +%%% mode: latex +%%% TeX-master: "master" +%%% End: diff --git a/TST_sti2d/DM/2102_DM1/corr_all_2102_DM1.pdf b/TST_sti2d/DM/2102_DM1/corr_all_2102_DM1.pdf new file mode 100644 index 0000000..5d1f98b Binary files /dev/null and b/TST_sti2d/DM/2102_DM1/corr_all_2102_DM1.pdf differ diff --git a/TST_sti2d/DM/2102_DM1/tpl_2102_DM1.tex b/TST_sti2d/DM/2102_DM1/tpl_2102_DM1.tex new file mode 100644 index 0000000..11a128b --- /dev/null +++ b/TST_sti2d/DM/2102_DM1/tpl_2102_DM1.tex @@ -0,0 +1,153 @@ +\documentclass[a4paper,10pt]{article} +\usepackage{myXsim} +\usepackage{tasks} + +% Title Page +\title{DM1 \hfill \Var{Nom}} +\tribe{TST sti2d} +\date{\hfillÀ render pour le jeudi 25 février} + +\xsimsetup{ + solution/print = false +} + +\begin{document} +\maketitle + +%- set I = sympy.I +%- set latex = sympy.latex +%- set sqrt = sympy.sqrt +%- set exp = sympy.functions.exp +%- set integrate = sympy.integrate + +\begin{exercise}[subtitle={Complexes}] + \begin{enumerate} + %- set z_num = randint(2, 10) + I*randint(2, 10) + %- set z_denom = -randint(2, 10) + I*randint(2, 10) + %- set z1 = z_num / z_denom + \item Mettre le nombre complexe suivant sous forme algébrique $z_1 = \dfrac{\Var{latex(z_num)}}{\Var{latex(z_denom)}} $ + %- set base = choice([(1, sqrt(3)), (sqrt(2), sqrt(2)), (sqrt(3), 1)]) + %- set z2 = randint(1, 10)*(choice([1, -1])*base[0] + choice([1, -1])*base[1]*I) + \item Mettre le complexe suivante sous forme exponentielle $z_2 = \Var{latex(z2)}$ + %- set base = choice([(1, sqrt(3)), (sqrt(2), sqrt(2)), (sqrt(3), 1)]) + %- set z3 = randint(1, 10)*(choice([1, -1])*base[0] + choice([1, -1])*base[1]*I) + \item Mettre le complexe suivante sous forme exponentielle $z_3 = \Var{latex(z3)}$ + %- set z4 = z2*z3 + \item Calculer le produit $z_4=z_2\times z_3$ donner le résultat sous forme exponentielle puis algébrique. + %- set z5 = z2/z3 + \item Calculer le quotient $z_5=\frac{z_2}{z_3}$ donner le résultat sous forme exponentielle puis algébrique. + \end{enumerate} +\end{exercise} + +\begin{solution} + \begin{enumerate} + \item $z_1 = \Var{latex(sympy.re(z1) + sympy.im(z1)*I)}$ + \item $z_3 = \Var{latex(sympy.Abs(z2))} e^{\Var{latex(I*sympy.arg(z2))}}$ + \item $z_4 = \Var{latex(sympy.Abs(z4))} e^{\Var{latex(I*(sympy.arg(z2) + sympy.arg(z3)))}} = \Var{latex(sympy.re(z4) + sympy.im(z4)*I)} = \Var{latex(sympy.N(sympy.re(z4), 3)+ sympy.N(sympy.im(z4), 3)*I)}$ + \item $z_5 = \Var{latex(sympy.Abs(z5))} e^{\Var{latex(I*(sympy.arg(z2) - sympy.arg(z3)))}} = \Var{latex(sympy.re(z5) + sympy.im(z5)*I)} = \Var{latex(sympy.N(sympy.re(z5), 3)+ sympy.N(sympy.im(z5), 3)*I)}$ + \end{enumerate} +\end{solution} + +\begin{exercise}[subtitle={Bassin}] + %- set a = round(random()*10, 1) + %- set b = round(random()*10, 1) + %- set x = sympy.symbols("x") + %- set f = -(x**2 - a*x + b)*exp(-x) + b + %- set F = integrate(f, x) + Le tour d'un bassin au niveau du sol présente deux axes de symétrie : l’axe des abscisses et la droite d’équation $x=4$. Il est obtenu par symétrie de la courbe $\mathcal{C_f}$ sur $\intFF{0}{4}$ où $f$ est la fonction définie par + + \[ + f(x) = \Var{latex(f)} + \] + On admet que sur $\intFF{0}{4}$ la fonction $f$ est positive. + \begin{enumerate} + \item Sur un repère, tracer l'allure de la courbe $\mathcal{C}_f$, les axes de symétries puis compléter pour dessiner la forme du bassin. + \item Montrer que la fonction $f$ admet comme primitive sur $\R$ la fonction $F$ définie par + \[ + F(x) = \Var{latex(F) | replace("1.0", "")} + \] + \item Calculer la quantité $\ds \int_0^4 f(x) \; dx$, vous donnerez le résultat sous forme exacte. Interpréter le résultat et reportez cette quantité sur le graphique. + \item On considère que l'échelle de votre graphique est de 1unité pour 15m. Calculer l'aire du bassin. Vous donnerez un résultat arrondi au $m^2$ près. + \end{enumerate} +\end{exercise} + +\begin{solution} + \begin{enumerate} + \item + \begin{tikzpicture}[baseline=(a.north), xscale=1, yscale=0.5] + \tkzInit[xmin=0,xmax=5,xstep=1, + ymin=0,ymax=10,ystep=1] + \tkzGrid + \tkzAxeXY + \tkzFct[domain=0:10,color=red,very thick]% + { \Var{f} }; + \end{tikzpicture} + \item Il faut dériver $F(x)$ et vérifier que $F'(x) = f(x)$. + %- set surf = integrate(f, (x, 0, 4)) + \item $\ds \int_0^4 f(x) \; dx = F(4) - F(0) = \Var{latex(surf)}$ + \item La quantité calculée à la question précédente se retrouve 4fois pour former le bassin. Il faut ensuite prendre en compte l'échelle, comme 1unité de longueur correspond à 15m, une unité d'air correspond à $15\times15 = 225m^2$. Ainsi l'aire du bassin est égale à + \[ + (\Var{latex(surf)})\times 4 \times 15^2 = \Var{round(sympy.N(surf*4*15**2, 10), 0)} + \] + + \end{enumerate} +\end{solution} + +\begin{exercise}[subtitle={Bassin}] + %- set Vinit = randint(1, 10)*100000 + %- set tx = round((random()+1)/2, 1) + Le clinker est un constituant du ciment qui résulte de la cuisson d'un mélange composé de calcaire et d'argile. La fabrication du clinker nécessite des fours à très haute température qui libèrent dans l'air une grande quantité de dioxyde de carbone (CO$_2$). + + Dans une cimenterie, la fabrication du clinker s'effectue de 7 h 30 à 20 h, dans une pièce de volume \np{\Var{Vinit}}~dm$^3$. + + À 20 h, après une journée de travail, le taux volumique de CO$_2$ dans la pièce est de \Var{tx}\,\%. + \begin{enumerate} + %- set v20 = int(Vinit*tx/100) + \item Justifier que le volume de CO$_2$ présent dans cette pièce à 20 h est de \np{\Var{v20}}~dm$^3$ . + %- set q = round(random()/10, 2) + %- set c = randint(20, 60)*10 + %- set v0 = int(v20 - c) + %- set t = sympy.symbols("t") + %- set V = v0*exp(- q*t) + c + %- set Vp = V.diff() + \item On modélise le volume de CO$_2$ présent dans la pièce par une fonction du temps $t$ écoulé après 20h (exprimé en minutes) qui pour formule $V(t) = V_0e^{-\Var{q}t} + \Var{c}$ + \begin{enumerate} + \item Démontrer que $V_0$ est égale à \np{\Var{v0}}. + %- set decal = randint(1, 4) + \item Quel sera, au dm$^3$ près, le volume de CO$_2$ dans cette pièce à \Var{20+decal} h ? + \item Démontrer que $V'(t) = \Var{latex(Vp)}$. + \item Étudier le signe de $V'(t)$ puis en déduire le sens de variation de $V(t)$. + \item Que peut-on dire du volume de CO$_2$ quand $t$ devient grand? + \end{enumerate} + \end{enumerate} +\end{exercise} + +\begin{solution} + \begin{enumerate} + \item Volume à 20h: $\Var{Vinit}\times \Var{tx/100} = \Var{v20}$ + \item + \begin{enumerate} + \item $t=0$ correspond à 20h. + + Donc $V(0) = \Var{v20} = V_0e^{-\Var{q}\times 0} + \Var{c} = V_0 + \Var{c}$ + + Donc $V_0 = \Var{v20} - \Var{c} = \Var{v0}$ + \item Il faut calculer $V(t)$ pour $t = \Var{decal}$ donc + \[ + V(\Var{decal}) = \Var{round(V.subs(t, str(decal)), 2)} + \] + \item Pas de correction pour cette question. + \item Pas de correction pour cette question. + \item Pas de correction pour cette question. + \end{enumerate} + \end{enumerate} +\end{solution} + + +\end{document} + +%%% Local Variables: +%%% mode: latex +%%% TeX-master: "master" +%%% End: +